Está en la página 1de 91

Pregunta 1

Joven de 25 años, universitario, con múltiples parejas sexuales, refiere un cuadro hiperagudo
de 24 horas de evolución caracterizado por abundante secreción conjuntival purulenta en ojo
derecho, disminución de la visión unilateral y escozor. Llama la atención la severa hiperemia
y edema conjuntival y la opacidad corneal incipiente. El tratante de oftalmología sugiere
solicitar una tinción de Gram. ¿Qué esperaría ver en el reporte de microbiología?
Seleccione una:
a. Bacilos gram positivos
b. Cocos gramnegativos en racimo
c. Diplococos gram negativos.
d. Cocos grampositivos en cadena

Pregunta 2
La retinopatía diabética se caracteriza por:
Seleccione una:
a. Aumento de los pericitos.
b. Microaneurismas vasculares.
c. Disminución de la vascularización.
d. Hemorragia del humor acuoso.

Pregunta 3
Indique. ¿Cuál es el componente de la solución salina?
Seleccione una:
a. Solución salina al 0.9% contiene: 154 mEq/L de sodio.
b. Solución salina 0.9% tiene un pH de 8.
c. Solución salina 0.9% contiene 109 mEq/L de cloro.
d. Solución salina al 0.9% contiene: 4 mEq/L de sodio.

Pregunta 4
La colangitis aguda se caracteriza por una triada clásica (Triada de Charcot). Indique sus
componentes: A) Ictericia. B) Alteración mental. C) Dolor. D) Fiebre. E) Choque séptico.
Seleccione una:
a. a, c, d.
b. c, d, e.
c. b, c, d.
d. a, c, e.

Pregunta 5
Paciente con trauma torácico que presenta signos de hipovolemia e hipoxia, se le realiza una
Radiografía de tórax en la que se evidencia la presencia de líquido en hemitórax izquierdo,
por lo mencionado anteriormente el paciente presenta un hemitórax, para que este sea
considerado como masivo cuanto de sangre debe contener el hemitórax afectado:
Seleccione una:
a. 1000 ml
b. 1500 ml
c. 500 ml
d. 800ml
Pregunta 6
Paciente se sexo femenino de 33 años de edad que acude por dolor en fosa lumbar derecha de
3 horas de evolución irradiado a genitales de carácter intermitente que no calma con el reposo
y se acompaña de nauseas. Presenta puño percusión positiva. Por lo que se piensa en cólico
renal ¿cuál es el mejor método diagnóstico que recomendaría ante esta sospecha?
Seleccione una:
a. Ecografía abdominal
b. TAC simple
c. RMN
d. Radiografía simple de abdomen

Pregunta 7
¿Cuál de las siguientes es una indicación para intubar a un paciente que fue encontrado en
una casa en llamas?
Seleccione una:
a. Facial edema
b. Hollín en vía aérea y pelo nasal chamuscado.
c. Cejas chamuscadas
d. Fuego ocurrió en un espacio cerrado.
e. Paciente incapaz de manejar sus propias secreciones.

Pregunta 8
¿Cuál es el tratamiento más adecuado para una quemadura superficial de espesor parcial en el
antebrazo?
Seleccione una:
a. Un vendaje limpio y seco, como una gasa.
b. Un aderezo que contiene plata disponible comercialmente.
c. B y D
d. Ungüento antibiótico tópico.
e. Antibióticos sistémicos y sulfadiazina de plata.

Pregunta 9
Niña de 4 años de edad. Su madre la trae a consulta por presentar a nivel ocular prurito,
secreción purulenta, ardor, dolor, fotofobia, visión borrosa, lagrimeo y sensación de cuerpo
extraño en los dos ojos, con una evolución de 36 horas. ¿Cuál de las siguientes bacterias no
sería una causa probable?
Seleccione una:
a. Staphylococcus epidermidis
b. Streptococcus pyogenes
c. Streptococcus pneumoniae
d. Staphylococcus aureus

Pregunta 10
Usted con su compañero de guardia llega al diagnóstico de abdomen agudo con un paciente
ingresado a la sala de urgencias a las 4am, con respecto al caso ¿Cuál es el aspecto más
importante para la evaluación del abdomen agudo?
Seleccione una:
a. Una correcta historia clínica y un buen examen físico
b. La valoración de examen de laboratorio
c. La valoración de una TC abdominal
d. La valoración de un ultrasonido

Pregunta 11
Varón de 58 años, padece de diabetes mellitus tipo 2 desde hace 5 años.
Presenta pérdida auditiva, súbita, unilateral derecha, acompañada de
acúfenos y ligera sensación de vértigo. Este cuadro se va intensificando
aproximadamente en tres días sin que se detecte casusa aparente. El examen
otoscópico no revela ningún dato de interés, ¿Cuál sería el tratamiento de
elección?
Seleccione una:
a. Prednisona más antivirales.
b. Antivertiginosos.
c. Corticoides intratimpánicos.
d. Corticoides por vía oral.

Pregunta 12
Dentro de los cánceres de estómago se considera como el cáncer más
frecuente a:
Seleccione una:
a. Carcinoma de células de Hurthle
b. Adenocarcinoma
c. Tumor maligno el estroma gastrointestinal
d. Linfoma

Pregunta 13
Varón de 34 años, consulta por presentar dolor de inicio súbito, intenso,
localizado en fosa lumbar derecha, presenta inquietud, intensa diaforesis y
nauseas. Usted administra ketorolaco 30mg IV pero luego de 30 minutos el
cuadro se mantiene igual. ¿Cuál sería el siguiente paso que daría?
Seleccione una:
a. Repito la dosis
b. Cambiaría a Tramadol
c. Añadiria otro AINE
d. Furosemida

Pregunta 14
Un enfermo de 50 años que acude al servicio de urgencias por presentar dolor
lumbar de 3 meses de evolución, irradiado por glúteo, cara dorsal de muslo y
dorso lateral de la pierna derecha. En la exploración encontramos un pie
derecho caído y cuando le pedimos al paciente que se ponga de puntillas y
logra realizarlo. Con esta información, ¿qué tipo de lesión o síndrome es el más
probable?
Seleccione una:
a. Hernia discal L3-L4.
b. Radículopatía L5 derecha.
c. Radículopatía L5 izquierda
d. Radiculopatia S1.

Pregunta 15
Dentro de las causas de obstrucción del intestino delgado usted considera como la principal a:
Seleccione una:
a. Vólvulos
b. Neoplasias
c. Bridas postquirúrgicas
d. Hernias

Pregunta 16
Paciente varón de 65 años de edad que acude a su control por Hiperplasia
benigna de próstata que ha venido siendo tratada con alfa bloqueante a dosis
plena pero aún presentaba un poco de molestias, por lo que se envió un eco
transrectal, que informa la presencia de una próstata de 40 gramos. ¿ Qué
medida terapeútica emplearía a continuación?
Seleccione una:
a. Subir la dosis del alfa bloqueante
b. Añadir inh. de la 5 alfa reductasa
c. Añadir un AINE
d. Añadir inh. de la 5 fosfodiesterasa

Pregunta 17
Varón de 42 años de edad, con antecedentes de asma en la infancia. Consulta
por presentar un cuadro clínico de 6 meses de evolución. Se caracteriza por
hiposmia, epistaxis ocasionales, congestión nasal, rinorrea y dolor facial. En la
rinoscopia anterior se visualiza tumoraciones con aspecto de uva en ambas
fosas nasales, son blandas, de color gris rosado y translúcidas. ¿Cuál es su
impresión diagnóstica?
Seleccione una:
a. Pólipos nasales.
b. Quistes naso alveolares.
c. Hipertrofia de cornetes.
d. Rinitis vasomotora.

Pregunta 18
Varón de 28 años, acude porque refiere molestias leves y sensación de peso
testicular. En la exploración física el médico encuentra disminución del tamaño
testicular izquierdo y aspecto de gusanos en la bolsa escrotal. ¿Qué será lo que
posiblemente tenga este paciente?
Seleccione una:
a. Hernia inguinal
b. Orquiepididimitis
c. Varicocele
d. Torsión testicular
Pregunta 19
La siguiente es causa benigna de elevación del PSA (Antígeno prostático Especifico), EXCEPTO:
Seleccione una:
a. Trauma perineal.
b. Prostatitis aguda.
c. Retención urinaria.
d. Epididimitis.

Pregunta 20
Una mujer de 52 años de edad, con diabetes mellitus en tratamiento oral
durante varios años. Desde hace diez días presenta rinorrea bilateral,
inicialmente fluida y después amarillenta. Desde hace tres días tiene fiebre y
cefalea frontal. Acude a consulta por dolor periocular izquierdo. Al examen
físico, temperatura bucal 39?C, FC; 110/min, zona periorbitaria izquierda con
edema, eritema y dolor. ¿Cuál es la conducta terapéutica más apropiada en
este caso?
Seleccione una:
a. Sintomático, realizar radiografía de senos de la cara e iniciar antibiótico.
b. Sintomático, iniciar tratamiento antibiótico intravenoso y hospitalizar.
c. Ambulatorio sintomática más ampicilina vía oral y reevaluar en 48 horas.
d. Ambulatorio sintomático, colocar compresas frías y reevaluar en 48 horas.

Pregunta 21
En la hipertension arterial infantil, se considera hipertension cuando los valores
de la tension arterial sistolica o diastolica son mayores al percentil:
Seleccione una:
a. 80
b. Ninguna de las anteriores
c. 90
d. 95

Pregunta 22
Una paciente de 25 años le realiza la siguiente pregunta, en mi cual sería la
presentación más frecuente de hiposecreción hipofisiaria anterior
Seleccione una:
a. Desnutrición y caquexia
b. Disminución de la pigmentación
c. Presencia de mixedema
d. Amenorrea

Pregunta 23
Paciente de 28 años de sexo femenino que ingresa al área de emergencia por
experimentar palpitaciones de forma súbita mientras veía la televisión. A la
exploración física paciente con frecuencia cardiaca de 165. El
electrocardiograma demuestra complejos QRS cortos y no se visualizan ondas
p. El paciente experimenta disnea y posteriormente un episodio sincopal. ¿Cual
es el tratamiento más adecuado en este caso?
Seleccione una:
a. Cardioversión eléctrica sincronizada
b. Dar anti-arrítmicos
c. Observar y hacer EKG periódicos
d. Mejorar la frecuencia cardiaca con betabloqueantes

Pregunta 24
Un hombre mexicoamericano de 35 años de edad acude con su médico
familiar porque su madre que vino de méxico a visitarlo presenta tuberculosis
(TB). El médico familiar hace una prueba del derivado proteínico purificado
(PPD) con resultados negativos, pero recomienda la profilaxis contra TB. ¿Cuál
de los siguientes fármacos está indicado para la profilaxis de TB en pacientes
expuestos?
Seleccione una:
a. Rifampicina
b. Etambutol
c. Isoniacida
d. Pirazinamida

Pregunta 25
En la fibrilación auricular (FA), señale la respuesta INCORRECTA:
Seleccione una:
a. La anticoagulación crónica en la FA solitaria está indicada preferentemente
en pacientes jóvenes (< 60 años) y sin factores de riesgo.
b. En el tratamiento crónico la digoxina habitualmente es insuficiente para el
control de la frecuencia durante el ejercicio
c. La amiodarona es más efectiva en prevenir recaídas en FA y mantener el
ritmo sinusal que los fármacos IA y IC.
d. El riesgo de accidente cerebrovascular embolígeno es similar en la FA
paroxística y en la FA crónica.

Pregunta 26
Paciente de 58 años con antecedentes de hipertensión, tabaquismo y
alcoholismo. Ingresa al departamento de emergencia por disnea en reposo con
presencia de ortopnea. Se evidencia a la exploración edema de miembros
inferiores, ingurgitación yugular y ascitis. En la radiografía demuestra
cardiomegalia importante y datos de redistribución vascular. ¿Cuál de las
siguientes medidas no es conveniente en este paciente?
Seleccione una:
a. Betabloqueantes
b. Diuréticos de asa.
c. Restricción hídrica
d. Antihipertensivos

Pregunta 27
Un hombre de 57 años con aleteo auricular al inicio se trata con quinidina para
controlar la arritmia. Se dio de alta del hospital mientras mejora su estado,
continúan las arritmias esporádicas. ¿Cuál de los siguientes fármacos puede
usarse como coadyuvante de la quinidina en el tratamiento del aleteo o flutter
auricular?
Seleccione una:
a. Procainamida
b. Digoxina
c. Lidocaína
d. Propranolol

Pregunta 28
Un hombre de 54 años de edad con antecedentes de abuso de tabaco de 75
cajetillas/año y de alcohol, presenta carcinoma de laringe. Su tratamiento
incluye cisplatino a dosis alta concomitante con la radioterapia. Ha presentado
náuseas y vómito significativos. ¿Cuál sería el mejor agente para tratar estos
efectos secundarios?
Seleccione una:
a. Loperamida
b. Metoclopramida
c. Prometacina
d. Ondansetrón

Pregunta 29
Enunciado de la pregunta
Su profesor de fisiología y fisiopatología le realiza la siguiente pregunta ¿Cuál de
los siguientes mecanismos describe mejor el efecto de los iones del calcio sobre
el miocardio?
Seleccione una:
a. Cronotropismo positivo
b. Cronotropismo negativo
c. Inotropismo negativo
d. Inotropismo positivo

Pregunta 30
Una mujer de 42 años fue expulsada de su vehículo durante una colisión
vehicular. Camino al departamento de emergencia personal paramédico
reporta: frecuencia cardiaca de 130/min, presión arterial de 90/45 mmHg y
frecuencia respiratoria de 34/min. Presenta una herida en extremidad inferior
derecha a la que se realiza presión directa para controlar el sangrado. Al
momento la paciente se encuentra ansiosa y confusa. Seleccione el grado de
hemorragia según pérdida sanguínea estimada y elija el tipo de restitución de
líquidos.
Seleccione una:
a. Hemorragia grado III, necesita cristaloides.
b. Hemorragia grado IV, necesita cristaloides y sangre.
c. Hemorragia grado III, necesita cristaloides y sangre.
d. Hemorragia grado II, necesita cristaloides.

Pregunta 31
Un hombre de 66 años de edad acude a su consultorio por el antecedente de 5
meses de tos seca. Niega otros síntomas. Sus antecedentes médicos incluyen
infarto de miocardio(IM) reciente, después del cual se le prescribieron varios
medicamentos. No fuma, no ha tenido antecedentes de asma. Usted decide
que los efectos secundarios del medicamento son la causa más probable de
sus síntomas ¿Qué medicamento sería ese?
Seleccione una:
a. Nitroglicerina
b. Quinidina
c. Digoxina
d. Lisinopril

Pregunta 32
Enunciado de la pregunta
Un estudiante universitario de 20 años de edad participa en varios programas
atléticos intramurales, pero menciona que su asma, que usted ha estado
tratando con glucocorticoides inhalados durante 5 años, está empeorando. En
el último mes ha utilizado su inhalador de albuterol al menos 20 veces después
de jugar béisbol, pero no ha estado caminando mucho por las noche. ¿Cuál de
los siguientes sería mejor opción de cambio para este paciente?
Seleccione una:
a. Triamcinolona oral
b. Zileutón
c. Etanercept
d. Salmeterol

Pregunta 33
Un paciente de 16 años de edad entra a su clínica de dermatología y
manifiesta un exantema. No toma medicamentos, viste y se arregla bien. Usted
diagnostica un caso leve de acné vulgar y nota que su piel y cabello parecen
grasas de forma inusual
Seleccione una:
a. Bexaroteno
b. Dexametasona tópica
c. Isotretinoína
d. Calcipotrieno

Pregunta 34
Marta, una paciente de 27 años, manifiesta dificultad para deglutir, dolor de la
garganta e inflamación con hipersensibilidad en el cuello, durante la semana
pasada ha padecido de fiebres intermitentes y le diagnosticaron infección de
las vías respiratorias altas. En la exploración física que le realiza usted nota un
bocio doloroso al tacto, en los laboratorios que le trae de destaca leucocitos de
13800 sin desviación, VSG de 53 mm/hr , valores de TSH suprimida y anticuerpos
antiroideos negativos. ¿Cuál considera es el diagnóstico presuntivo más
acertado?
Seleccione una:
a. Angina de Ludwig
b. Hipotiroidismo autoinmunitario
c. Tiroiditis subaguda
d. Enfermedad de graves

Pregunta 35
Un hombre de 39 años ha experimentado un empeoramiento de la fiebre, la tos
y la disnea en las últimas 2 semanas por lo cual acude a la emergencia del
hospital donde usted labora como médico residente de medicina interna. En la
exploración se detectan estertores y disminución del murmullo vesicular en la
auscultación torácica. Una radiografía de tórax muestra infiltrados diseminados
en ambos pulmones. La prueba de la tuberculina da lugar a 6 mm de
induración. Una muestra de esputo es negativa, aunque el lavado
broncoalveolar es positivo para bacilos acidorresistentes. El recuento leucocítico
es de 4.600/mm3, con un recuento diferencial de un 80% de neutrófilos, un 10%
de linfocitos y un 10% de monocitos. ¿Cuál de los siguientes es el factor de riesgo
más probable para el desarrollo de esta enfermedad?
Seleccione una:
a. Consumo excesivo de alcohol
b. Escorbuto
c. Diabetes mellitus
d. Infección por el VIH

Pregunta 36
Un varón de 61 años de edad obeso con antecedentes de hipertensión y
dislipidemia mal controlada se presenta al hospital de milagro presentando un
infarto agudo de miocardio sin elevación del ST e ingresa en la unidad de
cuidados coronarios. al día siguientes desarrolla bradicardia asintomática. su
presión arterial es de 126/86 mmHg, el pulso 50/min y en la exploración física sus
tonos cardíacos son normales, sin ningún ruido o roce adicional. su ECG ha
cambiado. ¿Cuál de los siguientes hallazgos del ECG es la mejor indicación
para implantar un marcapasos?
Seleccione una:
a. Bloqueo AV de primer grado
b. Hemibloqueo ventricular izquierdo (LBBB) y bloqueo AV de segundo grado
Mobitz II
c. Bradicardia persistente
d. Bloqueo AV de segundo grado Mobitz I
e. Nuevo bloque de la rama derecha

Pregunta 37
Enunciado de la pregunta
Acude al hospital de Guaranda un varón de 67 que presenta un infarto agudo
de miocardio (MI) en cara anterior y recibe tratamiento trombolítico. tres días
después, nota dolor torácico que empeora al acostarse, con una exploración
física normal excepto por la presencia de roce por fricción. su ECG muestra los
cambios correspondientes al infarto de cara anterior, pero aparece ahora
depresión del segmento PR y una elevación de 1mm del segmento ST en las
derivaciones de las extremidades. ¿Cuál de los siguientes diagnósticos es el más
probable?
Seleccione una:
a. Reinfarto de miocardio
b. Embolia pulmonar
c. Aneurisma disecante de aorta
d. Pericarditis postinfarto

Pregunta 38
Enunciado de la pregunta
Paciente femenina de 25 años de edad acude a consulta por poliuria y
polidipsia. Hasta el momento, las investigaciones excluyen causas psicógenas y
de hiperglicemia. Se le realiza una prueba de la sed, al final de la misma la
osmolaridad urinaria es de 240 y el nivel de la hormona antidiurética se
encuentra elevado. ¿Cuál de los siguientes diagnósticos es el más probable?
Seleccione una:
a. Diabetes insípida central
b. Defecto de la corteza suprarrenal
c. Enfermedad de Addison
d. Diabetes insípida nefrogena

Pregunta 39
En una brigada médica en un cantón rural le presentan a Elsa de 27 años que
presenta un cuadro clínico de pérdida de peso, cansancio y debilidad también
ha experimentado náuseas y vómito, a la exploración física usted nota
hiperpigmentación de la piel, su sodio sérico está bajo y su potasio está bien
elevado, con estos datos clínicos y de laboratorio la paciente que otra cosa
puede presentar:
Seleccione una:
a. Los esteroides en orina esta aumentados
b. El calcio sérico esta aumentado
c. La diuresis aumentada
d. La piel brillante y pálida

Pregunta 40
Paciente que dentro de las mediciones rutinaria de presión arterial presenta una
media de 160/105 mmHg. Según este valor, ¿dentro de que grado de
hipertensión se encuentra?
Seleccione una:
a. Hipertensión etapa 2
b. Prehipertensión
c. Presión arterial normal
d. Hipertensión etapa 1

Pregunta 41
Señale en qué circunstancia la firma de aceptación del consentimiento
informado debe ser notarizada:
Seleccione una:
a. Terapias nuevas de baja complejidad.
b. Procedimientos diagnósticos.
c. Intervenciones quirúrgicas.
d. Trasplante de órganos.

Pregunta 42
Pedro de 72 años, con Enfermedad Renal Crónica acude al hospital por su cita
de hemodiálisis trisemanal. Durante diálisis se torna inquieto e intenta irse. Afirma
que el equipo médico esta violando sus derechos y debe ir a trabajar. 15
minutos después se torna violento y es fijado. Tiene además historial de Diabetes
Mellitus tipo 2, Hipertensión y Depresión Mayor la cual está en remisión hace 15
años. Su temperatura es 38.3ºC, tensión 112/63 mm Hg, pulso 93/min. Se
escucha un soplo en arteria carótida derecha y catéter de diálisis en yugular
interna izquierda. Durante la evaluación neurológica, es confuso pero
cooperativo sin signos focales. ¿Cuál de las siguientes opciones es la más
probable explicación para el comportamiento de este paciente?
Seleccione una:
a. Demencia Vascular
b. Desorden Psicotico Breve
c. Depresión mayor con características psicóticas
d. Delirio

Pregunta 43
Las personas que tienen derecho a solicitar copia certificada de la Historia
Clínica en un establecimiento de salud público o privado son los siguientes,
EXCEPTO:
Seleccione una:
a. Los usuarios de los servicios del establecimiento de salud público o privado.
b. Los representantes legales o apoderado(a) para menores de edad.
c. Las instituciones que representan las autoridades judiciales.
d. Familiares de segundo y tercer grado de consanguinidad para fallecidos.

Pregunta 44
Dentro de la figura de la eutanasia, ¿cuándo se permite su aplicación en el
territorio nacional ecuatoriano?
Seleccione una:
a. Cuando el paciente lo solicite, siempre que cuente con una valoración
psiquiátrica.
b. En caso de muerte encefálica, diagnosticada y certificada por medio del
protocolo del Ministerio de Salud Pública.
c. Nunca.
d. En caso de pacientes terminales y que no controlen sus síntomas con
cuidados paliativos.

Pregunta 45
¿Cuál es la escala que se utiliza con más frecuencia para la evaluación de la
intensidad del dolor en pacientes terminales?
Seleccione una:
a. Test de Folstein
b. Índice de Barthel
c. Índice de Katz
d. Escala Visual Análoga (EVA)

Pregunta 46
Daniela, de 26 años no tiene antecedentes médicos previos. Fue traída desde el
terminal terrestre porque "no podía respirar". Actualmente luce tranquila
manifiesta que desea ver a un médico por las dudas y luego regresar a trabajar.
Durante el episodio de disnea, tenia una saturación el 100%,
electrocardiograma y examen físico no mostraban signos relevantes, y a su
llegada al hospital en el triage no mostraba anormalidades en los signos vitales.
Teniendo en cuenta el caso presentado ¿Cuál de las siguientes opciones es la
más apropiada?
Seleccione una:
a. Indicar fluoxetina y consulta posterior con psiquiatría
b. Dímero D para descartar embolismo pulmonar
c. Observación y electrocardiograma seriado para descartar infarto miocardio
d. Interconsulta a psiquiatría antes del alta.

Pregunta 47
Mauricio de 18 años, es traído a la clínica por su madre por incremento de
comportamiento bizarro en los últimos 2 meses. Recientemente dejo el colegio y
no socializa. El paciente pasa la mayoría de tiempo solo en su cuarto y come
poco, porque cree que la comida esta envenenada. No usa drogas ilícitas y el
examen físico no es remarcable. Su cabello esta muy sucio y su ropa
desarreglada. Realiza poco contacto visual y mumura solo como respondiendo
a personas que le este hablando. ¿Cuál de los siguientes es el diagnostico más
probable?
Seleccione una:
a. Esquizofrenia
b. Desorden psicótico breve
c. Delirio
d. Desorden Esquizofreniforme

Pregunta 48
Paciente de 9 años con síndrome de Laron que ha sido seleccionado para
entrar en un ensayo clínico multicéntrico para probar un nuevo IGF-1
recombinante humano. El niño y sus padres firman el consentimiento informado.
¿Qué principio bioético está ejerciendo el paciente?
Seleccione una:
a. Autonomía.
b. Justicia.
c. Beneficencia.
d. Privacidad.

Pregunta 49
Mujer de 32 años de edad, 2 hijos de 3 y 5 años, antecedentes de un episodio
depresivo a los de edad. Al llegar su esposo le entrega una nota en la que se
despida y le encarga "velar por sus hijos", luego de lo cual se encierra en su
habitación. Luego de un largo tiempo el esposo logra ingresar a la habitación y
encuentra a la paciente llorosa, angustiada y con una apreciable cantidad de
medicamentes que han sido sacados de sus envases, al parecer listos para ser
ingeridos. El médico de urgencias es llamado, realizando una valoración
mediante la escala de Patterson y obtiene un puntaje de 7, ¿Cuál sería su
decisión?
Seleccione una:
a. Sugerir consulta con psiquiatra
b. Sugerir consulta con psicólogo
c. Observación en su domicilio
d. Traslado a un hospital

Pregunta 50
Un hombre de 56 años ha sufrido recientemente un infarto de miocardio (IM). Se
encuentra en tratamiento con diversos fármacos,muchos de los cuales se
metabolizan por el sistema del citocromo P450. Actualmente,acude al
psiquiatra por un trastorno del sueño y pérdida de apetito,y explica que no
disfruta del golf como solía hacerlo. Reconociendo que la depresión es habitual
en pacientes que han sufrido recientemente un IM,el psiquiatra decide iniciar
tratamiento con inhibidores de la recaptación de serotonina (ISRS). Cómo tomar
muchos fármacos,¿qué ISRS debería evitarse?
Seleccione una:
a. Fluoxetina
b. Sertralina
c. Escitalopram
d. Fenelzina

Pregunta 51
Señale lo correcto con respecto al Protocolo para el Diagnóstico y Certificación
de Muerte Encefálica del Ministerio de Salud Pública:
Seleccione una:
a. La certificación debe estar firmada por dos médicos, de los cuales al menos
uno debe ser un neurólogo o un psiquiatra
b. Los médicos que certifican la muerte encefálica deben ser parte del equipo
de trasplantes
c. En adultos, el examen neurológico debe ser repetido luego de al menos seis
horas
d. No es necesario que el paciente esté estable hemodinámicamente para
hacer el examen neurológico

Pregunta 52
Elija el término correcto para la siguiente definición: Los síntomas somáticos
pueden manifestarse de forma brusca por una descarga súbita y aislada del
sistema nervioso vegetativo junto a un miedo intenso a morir o a perder el
control. La duración de los síntomas es de aproximadamente entre 15 y 30
minutos y su expresión máxima es en los primeros 10 minutos.
Seleccione una:
a. Ansiedad.
b. Angustia.
c. Manía.
d. Euforia.

Pregunta 53
Fausto es un joven de 19 años de edad, quien escondido de sus padres tomó el
vehículo familiar para ir de fiesta con sus amigos. Al regresar, Fausto es
embestido por un camión que transitaba de forma ilegal por la carretera de
regreso. En el hospital y, luego de cumplirse estrictamente con el protocolo de
diagnóstico y certificación de muerte encefálica del MSP, se confirma Muerte
Encefálica en Fausto. Sus familiares se niegan a donar sus órganos, pese a haber
Fausto declarado en vida su voluntad de hacerlo (Registro Civil). ¿Qué debe
hacer usted, como médico de urgencias?
Seleccione una:
a. Decirle a la familia que la opinión de ellos no importa en nada y que, a
diferencia de ellos, Fausto sí quería ayudar con su órganos y tejidos.
b. Ser empático con el duelo de la familia, pero comunicar que ese era el
deseo de Fausto y que debe continuar con el protocolo de la INDOT.
c. Llamar al equipo de seguridad del hospital y evacuar a los familiares.
d. Esperar a que acuda el abogado de la familia y dialogar con él.

Pregunta 54
María es una paciente de 85 años de edad quien actualmente está
hospitalizada por una neumonía grave de la comunidad. Está desaturando y
usted le plantea la próxima probable necesidad de uso de ventilación
mecánica invasiva. María se niega de forma verbal y explícita al uso de esta
medida, pese a comprender que de esa decisión puede derivarse su muerte.
Está consciente y es capaz y competente para tomar decisiones. Usted cree
que el ventilador sería momentáneo para su recuperación. ¿Qué debe hacer
usted en este caso?
Seleccione una:
a. No intubar a María sería cometer eutanasia, por lo que usted está obligado a
colocarle el ventilador.
b. Como su médico, usted debe seguir el principio de no maleficencia y debe
proceder a la intubación cuando María lo necesite.
c. Debe respetar la decisión autónoma de María y hacer que ella firme su
voluntad en un documento de consentimiento informado.
d. Debe declarar incapaz a María, alegando que por la edad y la infección no
puede decidir de forma correcta.

Pregunta 55
Un residente de primer año de cirugía ha rotado por diversos servicios de cirugía
durante ese año, incluyendo cirugía general, cirugía
cardiotorácica,urología,cirugía oncológica,traumatología y cirugía colorrectal.
Ha conseguido acostumbrarse a recetar generosamente morfina para el
control del dolor. Sin embargo,¿cuál de las siguientes es una contraindicación
absoluta para el uso de opioides?
Seleccione una:
a. Cólico renal
b. Cólico biliar
c. Traumatismo craneoencefálico cerrado
d. Infarto de miocardio

Pregunta 56
Un varón de 23 años es trasladado a urgencias tras encontrarle caminando
desnudo por la calle y proclamándose a sí mismo el hijo de Dios . El análisis de
tóxicos en orinales negativo para drogas y alcohol. Durante la entrevista con el
psiquiatra de guardia,el paciente muestra una fuga de ideas de un tema a otro.
El médico recomienda empezar tratamiento con litio para la manía aguda.
¿Cuál delas siguientes afirmaciones se asocia al tratamiento con litio?
Seleccione una:
a. Pérdida de peso
b. Temblor fino
c. Hipertiroidismo
d. Retención de orina

Pregunta 57
En un paciente con riesgo suicida. ¿Cuál de los siguientes enunciados es un
criterio para hospitalización inmediata?
Seleccione una:
a. No tener ninguna comorbilidad.
b. El alto grado de letalidad del intento suicida.
c. Sexo femenino.
d. Tener una sólida red de apoyo psicosocial.

Pregunta 58
Una joven de 27 años de edad, es abandonada súbitamente en la puerta del
hospital, con múltiples contusiones, estupor e hipotensión severa. Usted realiza
un eco FAST y comprueba que la paciente presenta hemorragia abdominal
masiva por estallido esplénico. Su secuencia de pasos será la siguiente:
Seleccione una:
a. Enviar inmediatamente a la paciente a quirófano, realizando al mismo
tiempo medidas de estabilización y pruebas de diagnóstico (incluyendo
pruebas cruzadas y tipo de sangre).
b. Colocar una vía periférica, administrar cristaloides y 2 U de GRC.
c. Preguntar a la seguridad del hospital si vieron quién abandonó a la paciente.
d. Solicitar pruebas cruzadas, tipo de sangre y programar cirugía cuando la
paciente pueda firmar el consentimiento informado.

Pregunta 59
Un paciente con diagnóstico de enfermedad bipolar tipo 1, se encuentra en un
episodio de manía psicótica. Señale la respuesta correcta:
Seleccione una:
a. El paciente debe dar al menos su autorización verbal para poder ingresarse.
b. El paciente debe dar su consentimiento para poder ingresarse al hospital.
c. Podremos tratar al paciente de forma ambulatoria hasta que él decida
ingresar, por sus propios medios.
d. En este caso, se puede prescindir del consentimiento informado, debido a la
falta de conciencia de la enfermedad.

Pregunta 60
¿Qué grupo farmacológico se recomienda en pacientes adultos con depresión
mayor que presentan ideación suicida?
Seleccione una:
a. Antipsicóticos atípicos.
b. Anticonvulsivantes.
c. Benzodiazepinas.
d. Inhibidores selectivos de la recaptación de serotonina.

Pregunta 61
Paciente primigesta de 39 semanas de gestación, que se encuentra en labor de
parto. Al ingreso al centro obstétrico tiene actividad uterina de 4 contracciones
en 10 minutos, de 50 segundos de duración y 80 mm Hg de intensidad. A la
exploración vaginal: 3 cm de dilatación y 100% de borramiento, con
membranas integras y presentación cefálica en primer plano, proporción
cefalopélvica adecuada. Durante 8 horas ha mantenido la actividad uterina y
los cambios cervicales han alcanzado dilatación de 5 cm y 100% de
borramiento. Frecuencia cardiaca fetal basal de 152 por minuto, ¿Cuál es el
siguiente paso a seguir?
Seleccione una:
a. Continuar con el control y evolución espontanea.
b. Terminación del embarazo por vía alta.
c. Administración de oxitocina a dosis respuesta.
d. Rotura artificial de membranas.

Pregunta 62
Complete el siguiente enunciado: Una de las contraindicaciones para el uso
del dispositivo intrauterino de liberación de levonorgestrel (LNG-IUS) es ______
presentando una tasa de expulsión de _____
Seleccione una:
a. Metrorragia de origen desconocido, 20 %.
b. La existencia de leiomiomas que deformen la cavidad del útero, 10 %.
c. La existencia de leiomiomas que no deformen la cavidad uterina, 30 %.
d. Mujeres infectadas por el virus de inmunodeficiencia humana (VIH), 50%.

Pregunta 63
María de 22 años cursa su primer embarazo sin complicaciones. Al momento:
38.5 semanas de gestación, actividad uterina leve, y refiere pérdida de tapón
mucoso hace una hora. Razón por la cual acude a un centro médico particular,
donde se realiza ecografía que reporta: feto único, posición cefálica, LCF 130,
ILA 7. ¿Cuál es la conducta obstétrica más adecuada a seguir?
Seleccione una:
a. Indica cesárea de urgencia por sufrimiento fetal.
b. Esperar la evolución normal del parto.
c. Induce el trabajo de parto.
d. Ingreso hospitalario por oligohidramnios.
Retroalimentación

Pregunta 64
Paciente femenino diagnosticada con verrugas genitales externas. ¿Cuál es el
tratamiento aplicado por el médico?
Seleccione una:
a. Imiquimod 5 % cada noche por 5 semanas.
b. Sinecatequina, al 15% 3 veces a la semana.
c. Podofilox 5 % una vez al mes.
d. Resina de podofilina al 10 – 25 % cada semana.

Pregunta 65
Paciente de 28 años, primigesta, con embarazo de 39 semanas, al tacto vaginal
presenta cuello uterino de consistencia blanda, posición anterior, con dilatación
de 3 cm, borramiento del 60% y la presentación del feto OIIA se encuentra en el
III plano de Hodge (localización -1). ¿A qué índice de Bishop corresponde e
indique si es favorable o desfavorable para inducción de trabajo de parto?
Seleccione una:
a. 8 puntos, desfavorable.
b. 3 puntos, desfavorable.
c. 2 puntos, favorable.
d. 10 puntos, favorable.

Pregunta 66
La velocidad de ganancia de peso, promedio, por semana (kg/sem), en una
embarazada durante el segundo trimestre con un IMC pregestacional normal es
de:
Seleccione una:
a. 0.51Kg/sem.
b. 0.28 Kg/sem.
c. 0.22 Kg/sem.
d. 0.42 Kg/sem.

Pregunta 67
A su consulta llega Jacquie de 32 años de edad, que cursa su tercer embarazo.
Es su primera consulta, pero como ya sabe el procedimiento trae exámenes de
laboratorio que reportan: Hb 10mg/dl, Hto de 34%, VCM 78fl, según su
diagnóstico ¿Cuál sería la conducta más adecuada para esta paciente?
Seleccione una:
a. Suplementa con hierro oral 120mg + acido fólico.
b. Trasfunde 1 unidad de GRC.
c. Le pide cambios alimenticios.
d. Suplementa con hierro parenteral +acido fólico.

Pregunta 68
El enunciado correcto en referencia al parto pretérmino es:
Seleccione una:
a. La proteína C reactiva en frotis vaginal previene el riesgo de parto
pretérmino.
b. El incremento del pH vaginal previene el riesgo de parto pretérmino.
c. Las modificaciones cervicales valoradas por ecografía, son un marcador
predictor.
d. El uso de progesterona aumenta la incidencia de parto pretérmino.

Pregunta 69
Durante el Control prenatal específico en mujeres con DM uno de los exámenes
a solicitarse con su debida frecuencia es la ecografía. De la tabla expuesta a
continuación, relacione el período del embarazo con el objetivo para el cual se
solicita la ecografía.
Semana gestacional Objetivo de la ecografía
a. Realizar ecografía para valorar crecimiento fetal, perímetro
abdominal fetal y determinación de percentil para descartar
1. Semanas 11 a 14. macrosomía fetal incipiente.
2. Semanas 20 a 22. b. Realizar ecografía para valorar crecimiento fetal y el
volumen del
3. Semanas 28 a 30. líquido amniótico.
4. Semana 36. c. Realizar ecografía para determinar marcadores ecográficos
De AC
d. Realizar una ecografía morfológica fetal (detalle anatómico

Seleccione una:
a. 1a, 2b, 3c, 4d.
b. 1d, 2a, 3b, 4c.
c. 1c, 2d, 3a, 4b.
d. 1b, 2c, 3d, 4a.

Pregunta 70
Una vez que la hemorragia postparto inmediata ha sido identificada, indique el
tratamiento farmacológico de primera opción y la dosis correcta.
Seleccione una:
a. Oxitocina 10Ul/mL lM (o 5 Ul lV lento), o 20-40 Ul en 1000 mL de solución
cristaloide en infusión IV a 250 mL/h.
b. Acido tranexámico 500mg intravenoso a pasar diluido en 200 ml de solución
fisiológica y luego 500 mg intramuscular cada 8 horas.
c. Misoprostol, 200 mcg VO, 1er minuto posparto, 300 mcg cada hora por 6
horas.
d. Ergonovina 0,2 mg IM, 1er minuto, Ergonovina repetir cada 2 a 4 horas,
máximo 5 dosis (1 mg) en un período de 24 horas.

Pregunta 71
Niño de 5 años de edad acude a emergencias con fiebre de 48 horas de
evolución. El examen físico es normal. El residente solicita exámenes y prescribe
acetaminofén. ¿Cuál es la dosis adecuada para este paciente?
Seleccione una:
a. 15 mg/ kg dosis cada 6 horas.
b. 5 mg/Kg dosis cada 4 horas.
c. 30 miligramos/ Kilo / día.
d. 15 miligramos cada 6 horas.

Pregunta 72
Un recién nacido de 32 semanas de edad gestacional, sin antecedentes
prenatales de importancia, nace por parto céfalo vaginal luego de amniorexis
instrumental. Presenta desde el nacimiento quejido, aleteo nasal, retracciones y
cianosis. En el examen físico se encuentra una frecuencia respiratoria de 80 lpm,
crepitaciones bilaterales y mala entrada de aire. La radiografía de tórax revela
un volumen pulmonar disminuido, infiltrado retículo granular bilateral, ¿Cuál de
los siguientes factores mejoraría el pronóstico?
Seleccione una:
a. Realizar cesárea electiva
b. Aplicar corticoides prenatales
c. Paciente de sexo masculino
d. Prescribir prostaglandina E2

Pregunta 73
Una niña de 12 años presenta desde hace 2 días temperatura de 40 grados y
odinofagia, al examen físico se encuentra exudado purulento en amígdalas,
lengua saburral y petequias en el paladar, que complicaciones podría
presentar esta paciente.
Seleccione una:
a. Glomerulonefritis
b. Artritis febriles
c. Convulsiones febriles
d. Encefalitis

Pregunta 74
Un niño a término, de 4 días de vida acude hoy al centro de salud de sucumbíos
para recibir las vacunas correspondientes a su edad. Hasta el momento no ha
recibido ninguna vacuna, ¿qué vacunas le corresponderán el día de hoy?
Seleccione una:
a. Pentavalente, rotavirus
b. BCG, pentavalente
c. Hepatitis a, neumococo
d. Hepatitis B, BCG

Pregunta 75
La administración de zinc a un niño con diarrea aguda, según las
recomendaciones de la OMS, tiene las siguientes normas y ventajas. EXCEPTO:
Seleccione una:
a. Disminuye el riesgo de diarrea, en los 2 o 3 meses siguientes.
b. Restituye parte de los electrolitos perdidos durante la diarrea.
c. Deberá realizarse desde el comienzo de una diarrea aguda.
d. Reduce la duración, la gravedad y el riesgo de deshidratación.
Pregunta 76
Un lactante nacido a término presenta índices de Apgar de 7 y 10 a 1 y 5 min.
En la auscultación torácica se identifica un soplo cardíaco. Se observan
hepatoesplenomegalia y cataratas en el cristalino. El niño está en el percentil 30
de talla y peso. La ecografía muestra un conducto arterioso persistente. ¿Cuál
de los siguientes factores de riesgo es más probable en este lactante?
Seleccione una:
a. Infección congénita por rubéola2
b. Carencia de folato en la dieta
c. Dispermia en la concepción
d. Eritroblastosis fetal

Pregunta 77
Una niña de 8 años sin antecedentes de importancia acude a emergencia con
una crisis asmática aguda leve, ¿Cuál de los siguientes fármacos es de primera
elección en este cuadro?
Seleccione una:
a. Salmeterol
b. Trofilina
c. Sulfato de magnesio
d. Salbutamol

Pregunta 78
Un niño de 10 meses de edad se atoró con la llanta de un juguete. Usted debe
realizar la maniobra de Heimlich. ¿Cómo realizaría la misma a esta edad?
Seleccione una:
a. Con el niño en posición de Tredelemburg realizar una compresión en el
cartílago cricoides.
b. Posicionar al niño boca abajo sobre el antebrazo del reanimador y dar 5
palmadas entre los omóplatos.
c. Sostener al niño sentado en sus piernas y aplicarle una compresión con el
puño de su mano en el epigastrio.
d. Colocar al niño boca arriba y realizar con el puño dos compresiones debajo
del apéndice xifoides.

Pregunta 79
Una paciente de 5 años es ingresada con diagnóstico de desnutrición grave de
origen primario, ¿Con cuál de estas complicaciones está asociada la
mortalidad en las primeras horas de ingreso?
Seleccione una:
a. Anemia y desnutrición.
b. Hipotermia e Hipoglicemia.
c. Insuficiencia hepática y sepsis.
d. Insuficiencia cardiaca y renal.

Pregunta 80
Un neonato a término presenta a las 3 horas de vida hipoglicemia, de entre la
siguiente lista. ¿cuál de los siguientes signos permitiría sospechar un
hiperinsulinismo?
Seleccione una:
a. Macrosomia
b. Peso bajo
c. Temblores
d. Visceromegalias

Pregunta 81
Joven de 25 años, universitario, con múltiples parejas sexuales, refiere un cuadro hiperagudo de
24 horas de evolución caracterizado por abundante secreción conjuntival purulenta en ojo
derecho, disminución de la visión unilateral y escozor. Llama la atención la severa hiperemia y
edema conjuntival y la opacidad corneal incipiente. El tratante de oftalmología sugiere solicitar
una tinción de Gram. ¿Qué esperaría ver en el reporte de microbiología?
Seleccione una:
a. Bacilos gram positivos
b. Cocos gramnegativos en racimo
c. Diplococos gram negativos.
d. Cocos grampositivos en cadena

Pregunta 82
La retinopatía diabética se caracteriza por:
Seleccione una:
a. Aumento de los pericitos.
b. Microaneurismas vasculares.
c. Disminución de la vascularización.
d. Hemorragia del humor acuoso.

Pregunta 83
Indique. ¿Cuál es el componente de la solución salina?
Seleccione una:
a. Solución salina al 0.9% contiene: 154 mEq/L de sodio.
b. Solución salina 0.9% tiene un pH de 8.
c. Solución salina 0.9% contiene 109 mEq/L de cloro.
d. Solución salina al 0.9% contiene: 4 mEq/L de sodio.

Pregunta 84
La colangitis aguda se caracteriza por una triada clásica (Triada de Charcot). Indique sus
componentes: A) Ictericia. B) Alteración mental. C) Dolor. D) Fiebre. E) Choque séptico.
Seleccione una:
a. a, c, d.
b. c, d, e.
c. b, c, d.
d. a, c, e.

Pregunta 85
Paciente con trauma torácico que presenta signos de hipovolemia e hipoxia, se le realiza una
Radiografía de tórax en la que se evidencia la presencia de líquido en hemitórax izquierdo, por
lo mencionado anteriormente el paciente presenta un hemitórax, para que este sea considerado
como masivo cuanto de sangre debe contener el hemitórax afectado:
Seleccione una:
a. 1000 ml
b. 1500 ml
c. 500 ml
d. 800ml

Pregunta 86
Paciente se sexo femenino de 33 años de edad que acude por dolor en fosa lumbar derecha de
3 horas de evolución irradiado a genitales de carácter intermitente que no calma con el reposo
y se acompaña de nauseas. Presenta puño percusión positiva. Por lo que se piensa en cólico
renal ¿cuál es el mejor método diagnóstico que recomendaría ante esta sospecha?
Seleccione una:
a. Ecografía abdominal
b. TAC simple
c. RMN
d. Radiografía simple de abdomen

Pregunta 87
¿Cuál de las siguientes es una indicación para intubar a un paciente que fue encontrado en una
casa en llamas?
Seleccione una:
a. Facial edema
b. Hollín en vía aérea y pelo nasal chamuscado.
c. Cejas chamuscadas
d. Fuego ocurrió en un espacio cerrado.
e. Paciente incapaz de manejar sus propias secreciones.

Pregunta 88
¿Cuál es el tratamiento más adecuado para una quemadura superficial de espesor parcial en el
antebrazo?
Seleccione una:
a. Un vendaje limpio y seco, como una gasa.
b. Un aderezo que contiene plata disponible comercialmente.
c. B y D
d. Ungüento antibiótico tópico.
e. Antibióticos sistémicos y sulfadiazina de plata.

Pregunta 89
Niña de 4 años de edad. Su madre la trae a consulta por presentar a nivel ocular prurito,
secreción purulenta, ardor, dolor, fotofobia, visión borrosa, lagrimeo y sensación de cuerpo
extraño en los dos ojos, con una evolución de 36 horas. ¿Cuál de las siguientes bacterias no sería
una causa probable?
Seleccione una:
a. Staphylococcus epidermidis
b. Streptococcus pyogenes
c. Streptococcus pneumoniae
d. Staphylococcus aureus

Pregunta 90
Usted con su compañero de guardia llega al diagnóstico de abdomen agudo con un paciente
ingresado a la sala de urgencias a las 4am, con respecto al caso ¿Cuál es el aspecto más
importante para la evaluación del abdomen agudo?
Seleccione una:
a. Una correcta historia clínica y un buen examen físico
b. La valoración de examen de laboratorio
c. La valoración de una TC abdominal
d. La valoración de un ultrasonido

Pregunta 91
Varón de 58 años, padece de diabetes mellitus tipo 2 desde hace 5 años. Presenta pérdida
auditiva, súbita, unilateral derecha, acompañada de acúfenos y ligera sensación de vértigo. Este
cuadro se va intensificando aproximadamente en tres días sin que se detecte casusa aparente.
El examen otoscópico no revela ningún dato de interés, ¿Cuál sería el tratamiento de elección?
Seleccione una:
a. Prednisona más antivirales.
b. Antivertiginosos.
c. Corticoides intratimpánicos.
d. Corticoides por vía oral.

Pregunta 92
Dentro de los cánceres de estómago se considera como el cáncer más frecuente a:
Seleccione una:
a. Carcinoma de células de Hurthle
b. Adenocarcinoma
c. Tumor maligno el estroma gastrointestinal
d. Linfoma

Pregunta 93
Varón de 34 años, consulta por presentar dolor de inicio súbito, intenso, localizado en fosa
lumbar derecha, presenta inquietud, intensa diaforesis y nauseas. Usted administra ketorolaco
30mg IV pero luego de 30 minutos el cuadro se mantiene igual. ¿Cuál sería el siguiente paso que
daría?
Seleccione una:
a. Repito la dosis
b. Cambiaría a Tramadol
c. Añadiria otro AINE
d. Furosemida

Pregunta 94
Un enfermo de 50 años que acude al servicio de urgencias por presentar dolor lumbar de 3
meses de evolución, irradiado por glúteo, cara dorsal de muslo y dorso lateral de la pierna
derecha. En la exploración encontramos un pie derecho caído y cuando le pedimos al paciente
que se ponga de puntillas y logra realizarlo. Con esta información, ¿qué tipo de lesión o síndrome
es el más probable?
Seleccione una:
a. Hernia discal L3-L4.
b. Radículopatía L5 derecha.
c. Radículopatía L5 izquierda
d. Radiculopatia S1.
Retroalimentación

Pregunta 95
Dentro de las causas de obstrucción del intestino delgado usted considera como la principal a:
Seleccione una:
a. Vólvulos
b. Neoplasias
c. Bridas postquirúrgicas
d. Hernias

Pregunta 96
Paciente varón de 65 años de edad que acude a su control por Hiperplasia benigna de próstata
que ha venido siendo tratada con alfa bloqueante a dosis plena pero aún presentaba un poco
de molestias, por lo que se envió un eco transrectal, que informa la presencia de una próstata
de 40 gramos. ¿ Qué medida terapeútica emplearía a continuación?
Seleccione una:
a. Subir la dosis del alfa bloqueante
b. Añadir inh. de la 5 alfa reductasa
c. Añadir un AINE
d. Añadir inh. de la 5 fosfodiesterasa

Pregunta 97
Varón de 42 años de edad, con antecedentes de asma en la infancia. Consulta por presentar un
cuadro clínico de 6 meses de evolución. Se caracteriza por hiposmia, epistaxis ocasionales,
congestión nasal, rinorrea y dolor facial. En la rinoscopia anterior se visualiza tumoraciones con
aspecto de uva en ambas fosas nasales, son blandas, de color gris rosado y translúcidas. ¿Cuál
es su impresión diagnóstica?
Seleccione una:
a. Pólipos nasales.
b. Quistes naso alveolares.
c. Hipertrofia de cornetes.
d. Rinitis vasomotora.

Pregunta 98
Varón de 28 años, acude porque refiere molestias leves y sensación de peso testicular. En la
exploración física el médico encuentra disminución del tamaño testicular izquierdo y aspecto de
gusanos en la bolsa escrotal. ¿Qué será lo que posiblemente tenga este paciente?
Seleccione una:
a. Hernia inguinal
b. Orquiepididimitis
c. Varicocele
d. Torsión testicular
Retroalimentación

Pregunta 99
La siguiente es causa benigna de elevación del PSA (Antígeno prostático Especifico), EXCEPTO:
Seleccione una:
a. Trauma perineal.
b. Prostatitis aguda.
c. Retención urinaria.
d. Epididimitis.

Pregunta 100
Una mujer de 52 años de edad, con diabetes mellitus en tratamiento oral durante varios años.
Desde hace diez días presenta rinorrea bilateral, inicialmente fluida y después amarillenta.
Desde hace tres días tiene fiebre y cefalea frontal. Acude a consulta por dolor periocular
izquierdo. Al examen físico, temperatura bucal 39?C, FC; 110/min, zona periorbitaria izquierda
con edema, eritema y dolor. ¿Cuál es la conducta terapéutica más apropiada en este caso?
Seleccione una:
a. Sintomático, realizar radiografía de senos de la cara e iniciar antibiótico.
b. Sintomático, iniciar tratamiento antibiótico intravenoso y hospitalizar.
c. Ambulatorio sintomática más ampicilina vía oral y reevaluar en 48 horas.
d. Ambulatorio sintomático, colocar compresas frías y reevaluar en 48 horas.

Pregunta 101
Paciente mujer de 74 años de edad, que ingresó en el hospital por presentar disnea progresiva,
hasta hacerse de reposo, y ortopnea. Era portadora de una prótesis valvular mecánica aórtica
implantada hacía 20 años; en una ecocardiografía realizada 6 meses antes del ingreso la prótesis
era normofuncionante. En la exploración física destacaban los hallazgos siguientes: palidez de
piel, moderada ictericia conjuntival, ritmo cardíaco regular a una frecuencia de 85 lat/min, ruido
protésico valvular, soplo protodiastólico en foco de Erb, estertores crepitantes en ambas bases
pulmonares. Entre las exploraciones complementarias iniciales, los resultados más relevantes
fueron: Hb 10,2 g/dl, Htc 32,3%, VCM 75 fl, HCM 24 pg, CCMH 26,3 g/ dl, amplitud de distribución
eritrocitaria (ADE) 18,8%, leucocitos 8.300/μl, plaquetas 196.000/μl; parámetros bioquímicos en
sangre: glucosa 86 mg/dl, urea 42 mg/ dl, creatinina 1,01 mg/dl, Cl 91 mmol/l, Na 136 mmol/l,
K 3,8 mmol/l, bilirrubina total 3,9 mg/dl, bilirrubina conjugada 1,2 mg/dl, AST 34 U/l, ALT 22 U/l,
fosfatasa alcalina 95 U/l, LDH 824 U/l; ECG: ritmo sinusal, 87 lat/min, signos de hipertrofia
ventricular izquierda; radiografía de tórax: cardiomegalia, prominencia de los hilios pulmonares,
redistribución vascular pulmonar, patrón alveolar difuso en ambas bases pulmonares. Con los
datos analíticos expuestos,
¿cuál de las siguientes opciones constituiría su orientación diagnóstica inicial del caso?
Seleccione una:
a. Hemólisis.
b. Anemia por enfermedad crónica
c. Anemia inflamatoria.
d. Déficit de cobalamina o folato.
Pregunta 102
Su profesor de fisiología y fisiopatología le realiza la siguiente pregunta ¿Cuál de los siguientes
mecanismos describe mejor el efecto de los iones del calcio sobre el miocardio?
Seleccione una:
a. Desacoplamiento de la excitación-contracción
b. Inotropismo negativo
c. Cronotropismo negativo
d. Cronotropismo positivo
e. Inotropismo positivo

Pregunta 104
Enunciado de la pregunta
Un varón de 47 años presenta disnea y dolor torácico siete días después de una colecistectomía
abierta. Su presión arterial 145/86 mmHg, el pulso de 124/min, frecuencia respiratoria de
26/min y una saturación de oxígeno de 97 porciento. Clínicamente se sospecha embolia de
pulmón. ¿Cuál de los siguientes hallazgos en el ECG es más frecuente en la embolia de pulmón?
Seleccione una:
a. Desviación del eje a la derecha en derivaciones precordiales.
b. Descenso del ST en la derivación DI y DII
c. Onda Q prominente en derivación DI
d. Taquicardia sinusal
e. Onda S profunda de derivación DI

Pregunta 105
Un hombre de 26 años originario sur de Perú que vive actualmente en Huaquillas, presenta una
lesión en el pene que ha aumentado de tamaño en los últimos 4 meses. En la exploración física
se aprecia una lesión papulosa indolora de 2 cm en el dorso del pene que ha evolucionado hasta
formar una úlcera expansiva carnosa roja, que sangra con facilidad. No hay adenopatías
inguinales. Se obtiene una biopsia de la lesión, en cuyo examen microscópico se aprecian
hiperplasia pseudoepiteliomatosa e infiltrado inflamatorio mixto. La tinción de Giemsa revela
presencia de cocobacilos en vacuolas dentro de macrófagos. En la anamnesis social el paciente
informa de que ha mantenido relaciones con múltiples parejas sexuales. ¿Cuál de los siguientes
diagnósticos es más probable?
Seleccione una:
a. Chancroide
b. Balanopostitis
c. Linfogranuloma venéreo
d. Granuloma inguinal

Pregunta 105
Entre los casos de falsos negativos de la prueba de Intradermorracción de Mantoux, se
encuentran todos, excepto:
Seleccione una:
a. Pacientes con VIH
b. Paciente con tuberculosis miliar
c. Paciente vacunado previamente con BCG
d. Adultos mayores
Retroalimentación
Pregunta 106
Una niña de 8 años ha desarrollado una enfermedad febril leve con dolor de garganta a lo largo
de los últimos 2 días. Como antecedentes la madre refiere que ha llevado un peso control en
cuanto a las vacunaciones de la niña, ella ni sabe que le han puesto ni cuándo le ha puesto ya
que antes vivía la niña con sus abuelos. En la exploración física su temperatura es de 38,4 °C y
padece faringitis leve. Los síntomas remiten sin tratamiento después de 1 semana. Durante los
2 meses siguientes la niña padece parálisis facial del lado derecho con incapacidad para cerrar
el ojo de ese lado. ¿Cuál de los siguientes agentes infecciosos es más probable que cause estos
síntomas?
Seleccione una:
a. a) Cryptococcus neoformans
b. c) Listeria monocytogenes
c. d) Poliovirus
d. e) Toxoplasma gondii
e. b) Citomegalovirus

Pregunta 106
PACIENTE MASCULINO DE 7 AÑOS DE EDAD, ACUDE A CONSULTA POR PRESENTAR PLACA
ERITEMATOSA, SUPURATIVA, CON ALOPECIA AÑADIDA. TIENE APARENCIA DE PANAL DE ABEJAS.
PACIENTE REFIERE MUCHO DOLOR Y AL PRESIONAR LA LESION HAY DRENAJE DE CONTENIDO
PURULENTO.DE ACUERDO A LA IMPRESION DIAGNÓSTICA, CUAL ES EL TRATAMIENTO DE
ELECCIÓN PARA ESTE PACIENTE.
Seleccione una:
a. GRISEOFULVINA 10-20MG/KG CADA DÍA 8-12 SEMANAS
b. CLOTRIMAZOL TOPICO
c. TERBINAFINA 250MG CADA DIA POR 12 SEMANAS
d. ITRACONAZOL 200MG CADA DIA POR 12 SEMANAS

Pregunta 107
¿Según las últimas guías de la HTA, cuál de los siguientes fármacos no corresponde al
tratamiento de primera línea?
Seleccione una:
a. Hidroclorotiazida
b. Espironolactona
c. ARA II
d. IECAS

Pregunta 108
Usted se encuentra trabajando en atención primaria de salud y ve a un paciente de sexo
masculino de 43 años de que pesa 85kg y que mide 1,8 metros. En el cálculo de índice de masa
corporal es de 23 Kg/m2. De entre las opciones siguientes ¿En cuál se encuentra mayor riesgo
de desarrollar?
Seleccione una:
a. Hiperlipoproteinemia
b. Diabetes mellitus
c. Riesgo de mortalidad no incrementada
d. Ateroesclerosis
Pregunta 109
Un varón de 67 años procedente de la provincia de Manabí con antecedentes de arteriopatía
coronaria (CAD) se presenta a la emergencia del hospital donde usted trabaja con un síncope.
La exploración física es normal, a excepción de bradicardia (55 latidos/minuto) y un impulso
irregular. El electrocardiograma (ECG) muestra un bloqueo auriculoventricular (AV) tipo
Wenckebach. ¿Cuál de las siguientes alteraciones es más probable encontrar en el ECG?
Seleccione una:
a. Taquicardia
b. Alargamiento progresivo del intervalo PR
c. Ausencia de un latido después de la prolongación del PR
d. Acortamiento progresivo

Pregunta 110
En la hipertension arterial infantil, se considera hipertension cuando los valores de la tension
arterial sistolica o diastolica son mayores al percentil:
Seleccione una:
a. 80
b. Ninguna de las anteriores
c. 90
d. 95

Pregunta 111
Una paciente de 25 años le realiza la siguiente pregunta, en mi cual sería la presentación más
frecuente de hiposecreción hipofisiaria anterior
Seleccione una:
a. Desnutrición y caquexia
b. Disminución de la pigmentación
c. Presencia de mixedema
d. Amenorrea
Retroalimentación

Pregunta 112
Paciente de 28 años de sexo femenino que ingresa al área de emergencia por experimentar
palpitaciones de forma súbita mientras veía la televisión. A la exploración física paciente con
frecuencia cardiaca de 165. El electrocardiograma demuestra complejos QRS cortos y no se
visualizan ondas p. El paciente experimenta disnea y posteriormente un episodio sincopal. ¿Cual
es el tratamiento más adecuado en este caso?
Seleccione una:
a. Cardioversión eléctrica sincronizada
b. Dar anti-arrítmicos
c. Observar y hacer EKG periódicos
d. Mejorar la frecuencia cardiaca con betabloqueantes

Pregunta 113
Un hombre mexicoamericano de 35 años de edad acude con su médico familiar porque su madre
que vino de méxico a visitarlo presenta tuberculosis (TB). El médico familiar hace una prueba
del derivado proteínico purificado (PPD) con resultados negativos, pero recomienda la profilaxis
contra TB. ¿Cuál de los siguientes fármacos está indicado para la profilaxis de TB en pacientes
expuestos?
Seleccione una:
a. Rifampicina
b. Etambutol
c. Isoniacida
d. Pirazinamida

Pregunta 114
En la fibrilación auricular (FA), señale la respuesta INCORRECTA:
Seleccione una:
a. La anticoagulación crónica en la FA solitaria está indicada preferentemente en pacientes
jóvenes (< 60 años) y sin factores de riesgo.
b. En el tratamiento crónico la digoxina habitualmente es insuficiente para el control de la
frecuencia durante el ejercicio
c. La amiodarona es más efectiva en prevenir recaídas en FA y mantener el ritmo sinusal que los
fármacos IA y IC.
d. El riesgo de accidente cerebrovascular embolígeno es similar en la FA paroxística y en la FA
crónica.

Pregunta 115
Paciente de 58 años con antecedentes de hipertensión, tabaquismo y alcoholismo. Ingresa al
departamento de emergencia por disnea en reposo con presencia de ortopnea. Se evidencia a
la exploración edema de miembros inferiores, ingurgitación yugular y ascitis. En la radiografía
demuestra cardiomegalia importante y datos de redistribución vascular. ¿Cuál de las siguientes
medidas no es conveniente en este paciente?
Seleccione una:
a. Betabloqueantes
b. Diuréticos de asa.
c. Restricción hídrica
d. Antihipertensivos

Pregunta 116
Un hombre de 57 años con aleteo auricular al inicio se trata con quinidina para controlar la
arritmia. Se dio de alta del hospital mientras mejora su estado, continúan las arritmias
esporádicas. ¿Cuál de los siguientes fármacos puede usarse como coadyuvante de la quinidina
en el tratamiento del aleteo o flutter auricular?
Seleccione una:
a. Procainamida
b. Digoxina
c. Lidocaína
d. Propranolol

Pregunta 117
Un hombre de 54 años de edad con antecedentes de abuso de tabaco de 75 cajetillas/año y de
alcohol, presenta carcinoma de laringe. Su tratamiento incluye cisplatino a dosis alta
concomitante con la radioterapia. Ha presentado náuseas y vómito significativos. ¿Cuál sería el
mejor agente para tratar estos efectos secundarios?
Seleccione una:
a. Loperamida
b. Metoclopramida
c. Prometacina
d. Ondansetrón

Pregunta 118
Su profesor de fisiología y fisiopatología le realiza la siguiente pregunta ¿Cuál de los siguientes
mecanismos describe mejor el efecto de los iones del calcio sobre el miocardio?
Seleccione una:
a. Cronotropismo positivo
b. Cronotropismo negativo
c. Inotropismo negativo
d. Inotropismo positivo

Pregunta 119
Una mujer de 42 años fue expulsada de su vehículo durante una colisión vehicular. Camino al
departamento de emergencia personal paramédico reporta: frecuencia cardiaca de 130/min,
presión arterial de 90/45 mmHg y frecuencia respiratoria de 34/min. Presenta una herida en
extremidad inferior derecha a la que se realiza presión directa para controlar el sangrado. Al
momento la paciente se encuentra ansiosa y confusa. Seleccione el grado de hemorragia según
pérdida sanguínea estimada y elija el tipo de restitución de líquidos.
Seleccione una:
a. Hemorragia grado III, necesita cristaloides.
b. Hemorragia grado IV, necesita cristaloides y sangre.
c. Hemorragia grado III, necesita cristaloides y sangre.
d. Hemorragia grado II, necesita cristaloides.

Pregunta 120
Un hombre de 66 años de edad acude a su consultorio por el antecedente de 5
meses de tos seca. Niega otros síntomas. Sus antecedentes médicos incluyen
infarto de miocardio(IM) reciente, después del cual se le prescribieron varios
medicamentos. No fuma, no ha tenido antecedentes de asma. Usted decide
que los efectos secundarios del medicamento son la causa más probable de
sus síntomas ¿Qué medicamento sería ese?
Seleccione una:
a. Nitroglicerina
b. Quinidina
c. Digoxina
d. Lisinopril

Pregunta 121
Un estudiante universitario de 20 años de edad participa en varios programas
atléticos intramurales, pero menciona que su asma, que usted ha estado
tratando con glucocorticoides inhalados durante 5 años, está empeorando. En
el último mes ha utilizado su inhalador de albuterol al menos 20 veces después
de jugar béisbol, pero no ha estado caminando mucho por las noche. ¿Cuál de
los siguientes sería mejor opción de cambio para este paciente?
Seleccione una:
a. Triamcinolona oral
b. Zileutón
c. Etanercept
d. Salmeterol

Pregunta 122
Un paciente de 16 años de edad entra a su clínica de dermatología y
manifiesta un exantema. No toma medicamentos, viste y se arregla bien. Usted
diagnostica un caso leve de acné vulgar y nota que su piel y cabello parecen
grasas de forma inusual
Seleccione una:
a. Bexaroteno
b. Dexametasona tópica
c. Isotretinoína
d. Calcipotrieno

Pregunta 123
Marta, una paciente de 27 años, manifiesta dificultad para deglutir, dolor de la
garganta e inflamación con hipersensibilidad en el cuello, durante la semana
pasada ha padecido de fiebres intermitentes y le diagnosticaron infección de
las vías respiratorias altas. En la exploración física que le realiza usted nota un
bocio doloroso al tacto, en los laboratorios que le trae de destaca leucocitos de
13800 sin desviación, VSG de 53 mm/hr , valores de TSH suprimida y anticuerpos
antiroideos negativos. ¿Cuál considera es el diagnóstico presuntivo más
acertado?
Seleccione una:
a. Angina de Ludwig
b. Hipotiroidismo autoinmunitario
c. Tiroiditis subaguda
d. Enfermedad de graves

Pregunta 124
Un hombre de 39 años ha experimentado un empeoramiento de la fiebre, la tos
y la disnea en las últimas 2 semanas por lo cual acude a la emergencia del
hospital donde usted labora como médico residente de medicina interna. En la
exploración se detectan estertores y disminución del murmullo vesicular en la
auscultación torácica. Una radiografía de tórax muestra infiltrados diseminados
en ambos pulmones. La prueba de la tuberculina da lugar a 6 mm de
induración. Una muestra de esputo es negativa, aunque el lavado
broncoalveolar es positivo para bacilos acidorresistentes. El recuento leucocítico
es de 4.600/mm3, con un recuento diferencial de un 80% de neutrófilos, un 10%
de linfocitos y un 10% de monocitos. ¿Cuál de los siguientes es el factor de riesgo
más probable para el desarrollo de esta enfermedad?
Seleccione una:
a. Consumo excesivo de alcohol
b. Escorbuto
c. Diabetes mellitus
d. Infección por el VIH

Pregunta 125
Un varón de 61 años de edad obeso con antecedentes de hipertensión y
dislipidemia mal controlada se presenta al hospital de milagro presentando un
infarto agudo de miocardio sin elevación del ST e ingresa en la unidad de
cuidados coronarios. al día siguientes desarrolla bradicardia asintomática. su
presión arterial es de 126/86 mmHg, el pulso 50/min y en la exploración física sus
tonos cardíacos son normales, sin ningún ruido o roce adicional. su ECG ha
cambiado. ¿Cuál de los siguientes hallazgos del ECG es la mejor indicación
para implantar un marcapasos?
Seleccione una:
a. Bloqueo AV de primer grado
b. Hemibloqueo ventricular izquierdo (LBBB) y bloqueo AV de segundo grado
Mobitz II
c. Bradicardia persistente
d. Bloqueo AV de segundo grado Mobitz I
e. Nuevo bloque de la rama derecha

Pregunta 126
Acude al hospital de Guaranda un varón de 67 que presenta un infarto agudo
de miocardio (MI) en cara anterior y recibe tratamiento trombolítico. tres días
después, nota dolor torácico que empeora al acostarse, con una exploración
física normal excepto por la presencia de roce por fricción. su ECG muestra los
cambios correspondientes al infarto de cara anterior, pero aparece ahora
depresión del segmento PR y una elevación de 1mm del segmento ST en las
derivaciones de las extremidades. ¿Cuál de los siguientes diagnósticos es el más
probable?
Seleccione una:
a. Reinfarto de miocardio
b. Embolia pulmonar
c. Aneurisma disecante de aorta
d. Pericarditis postinfarto

Pregunta 127
Paciente femenina de 25 años de edad acude a consulta por poliuria y
polidipsia. Hasta el momento, las investigaciones excluyen causas psicógenas y
de hiperglicemia. Se le realiza una prueba de la sed, al final de la misma la
osmolaridad urinaria es de 240 y el nivel de la hormona antidiurética se
encuentra elevado. ¿Cuál de los siguientes diagnósticos es el más probable?
Seleccione una:
a. Diabetes insípida central
b. Defecto de la corteza suprarrenal
c. Enfermedad de Addison
d. Diabetes insípida nefrogena
Pregunta 128
En una brigada médica en un cantón rural le presentan a Elsa de 27 años que
presenta un cuadro clínico de pérdida de peso, cansancio y debilidad también
ha experimentado náuseas y vómito, a la exploración física usted nota
hiperpigmentación de la piel, su sodio sérico está bajo y su potasio está bien
elevado, con estos datos clínicos y de laboratorio la paciente que otra cosa
puede presentar:
Seleccione una:
a. Los esteroides en orina esta aumentados
b. El calcio sérico esta aumentado
c. La diuresis aumentada
d. La piel brillante y pálida

Pregunta 129
Paciente que dentro de las mediciones rutinaria de presión arterial presenta una
media de 160/105 mmHg. Según este valor, ¿dentro de que grado de
hipertensión se encuentra?
Seleccione una:
a. Hipertensión etapa 2
b. Prehipertensión
c. Presión arterial normal
d. Hipertensión etapa 1

Pregunta 130
Señale en qué circunstancia la firma de aceptación del consentimiento informado debe ser
notarizada:
Seleccione una:
a. Terapias nuevas de baja complejidad.
b. Procedimientos diagnósticos.
c. Intervenciones quirúrgicas.
d. Trasplante de órganos.

Pregunta 131
Pedro de 72 años, con Enfermedad Renal Crónica acude al hospital por su cita de hemodiálisis
trisemanal. Durante diálisis se torna inquieto e intenta irse. Afirma que el equipo médico esta
violando sus derechos y debe ir a trabajar. 15 minutos después se torna violento y es fijado.
Tiene además historial de Diabetes Mellitus tipo 2, Hipertensión y Depresión Mayor la cual está
en remisión hace 15 años. Su temperatura es 38.3ºC, tensión 112/63 mm Hg, pulso 93/min. Se
escucha un soplo en arteria carótida derecha y catéter de diálisis en yugular interna izquierda.
Durante la evaluación neurológica, es confuso pero cooperativo sin signos focales. ¿Cuál de las
siguientes opciones es la más probable explicación para el comportamiento de este paciente?
Seleccione una:
a. Demencia Vascular
b. Desorden Psicotico Breve
c. Depresión mayor con características psicóticas
d. Delirio
Pregunta 132
Las personas que tienen derecho a solicitar copia certificada de la Historia Clínica en un
establecimiento de salud público o privado son los siguientes, EXCEPTO:
Seleccione una:
a. Los usuarios de los servicios del establecimiento de salud público o privado.
b. Los representantes legales o apoderado(a) para menores de edad.
c. Las instituciones que representan las autoridades judiciales.
d. Familiares de segundo y tercer grado de consanguinidad para fallecidos.

Pregunta 133
Dentro de la figura de la eutanasia, ¿cuándo se permite su aplicación en el territorio nacional
ecuatoriano?
Seleccione una:
a. Cuando el paciente lo solicite, siempre que cuente con una valoración psiquiátrica.
b. En caso de muerte encefálica, diagnosticada y certificada por medio del protocolo del
Ministerio
de Salud Pública.
c. Nunca.
d. En caso de pacientes terminales y que no controlen sus síntomas con cuidados paliativos.

Pregunta 134
¿Cuál es la escala que se utiliza con más frecuencia para la evaluación de la intensidad del dolor
en pacientes terminales?
Seleccione una:
a. Test de Folstein
b. Índice de Barthel
c. Índice de Katz
d. Escala Visual Análoga (EVA)

Pregunta 135
Daniela, de 26 años no tiene antecedentes médicos previos. Fue traída desde el terminal
terrestre porque "no podía respirar". Actualmente luce tranquila manifiesta que desea ver a un
médico por las dudas y luego regresar a trabajar. Durante el episodio de disnea, tenia una
saturación el 100%, electrocardiograma y examen físico no mostraban signos relevantes, y a su
llegada al hospital en el triage no mostraba anormalidades en los signos vitales. Teniendo en
cuenta el caso presentado
¿Cuál de las siguientes opciones es la más apropiada?
Seleccione una:
a. Indicar fluoxetina y consulta posterior con psiquiatría
b. Dímero D para descartar embolismo pulmonar
c. Observación y electrocardiograma seriado para descartar infarto miocardio
d. Interconsulta a psiquiatría antes del alta.

Pregunta 136
Mauricio de 18 años, es traído a la clínica por su madre por incremento de comportamiento
bizarro en los últimos 2 meses. Recientemente dejo el colegio y no socializa. El paciente pasa la
mayoría de tiempo solo en su cuarto y come poco, porque cree que la comida esta envenenada.
No usa drogas ilícitas y el examen físico no es remarcable. Su cabello esta muy sucio y su ropa
desarreglada. Realiza poco contacto visual y mumura solo como respondiendo a personas que
le este hablando. ¿Cuál de los siguientes es el diagnostico más probable?
Seleccione una:
a. Esquizofrenia
b. Desorden psicótico breve
c. Delirio
d. Desorden Esquizofreniforme

Pregunta 137
Paciente de 9 años con síndrome de Laron que ha sido seleccionado para entrar en un ensayo
clínico multicéntrico para probar un nuevo IGF-1 recombinante humano. El niño y sus padres
firman el consentimiento informado. ¿Qué principio bioético está ejerciendo el paciente?
Seleccione una:
a. Autonomía.
b. Justicia.
c. Beneficencia.
d. Privacidad.

Pregunta 138
Mujer de 32 años de edad, 2 hijos de 3 y 5 años, antecedentes de un episodio depresivo a los de
edad. Al llegar su esposo le entrega una nota en la que se despida y le encarga "velar por sus
hijos", luego de lo cual se encierra en su habitación. Luego de un largo tiempo el esposo logra
ingresar a la habitación y encuentra a la paciente llorosa, angustiada y con una apreciable
cantidad de medicamentes que han sido sacados de sus envases, al parecer listos para ser
ingeridos. El médico de urgencias es llamado, realizando una valoración mediante la escala de
Patterson y obtiene un puntaje de 7, ¿Cuál sería su decisión?
Seleccione una:
a. Sugerir consulta con psiquiatra
b. Sugerir consulta con psicólogo
c. Observación en su domicilio
d. Traslado a un hospital

Pregunta 139
Un hombre de 56 años ha sufrido recientemente un infarto de miocardio (IM). Se encuentra
en tratamiento con diversos fármacos,muchos de los cuales se metabolizan por el sistema del
citocromo P450. Actualmente,acude al psiquiatra por un trastorno del sueño y pérdida de
apetito,y explica que no disfruta del golf como solía hacerlo. Reconociendo que la depresión
es habitual en pacientes que han sufrido recientemente un IM,el psiquiatra decide iniciar
tratamiento con inhibidores de la recaptación de serotonina (ISRS). Cómo tomar muchos
fármacos,¿qué ISRS debería evitarse?
Seleccione una:
a. Fluoxetina
b. Sertralina
c. Escitalopram
d. Fenelzina

Pregunta 140
Señale lo correcto con respecto al Protocolo para el Diagnóstico y Certificación
de Muerte Encefálica del Ministerio de Salud Pública:
Seleccione una:
a. La certificación debe estar firmada por dos médicos, de los cuales al menos
uno debe ser un neurólogo o un psiquiatra
b. Los médicos que certifican la muerte encefálica deben ser parte del equipo
de trasplantes
c. En adultos, el examen neurológico debe ser repetido luego de al menos seis
horas
d. No es necesario que el paciente esté estable hemodinámicamente para
hacer el examen neurológico

Pregunta 141
Elija el término correcto para la siguiente definición: Los síntomas somáticos
pueden manifestarse de forma brusca por una descarga súbita y aislada del
sistema nervioso vegetativo junto a un miedo intenso a morir o a perder el
control. La duración de los síntomas es de aproximadamente entre 15 y 30
minutos y su expresión máxima es en los primeros 10 minutos.
Seleccione una:
a. Ansiedad.
b. Angustia.
c. Manía.
d. Euforia.

Pregunta 142
Fausto es un joven de 19 años de edad, quien escondido de sus padres tomó el
vehículo familiar para ir de fiesta con sus amigos. Al regresar, Fausto es
embestido por un camión que transitaba de forma ilegal por la carretera de
regreso. En el hospital y, luego de cumplirse estrictamente con el protocolo de
diagnóstico y certificación de muerte encefálica del MSP, se confirma Muerte
Encefálica en Fausto. Sus familiares se niegan a donar sus órganos, pese a haber
Fausto declarado en vida su voluntad de hacerlo (Registro Civil). ¿Qué debe
hacer usted, como médico de urgencias?
Seleccione una:
a. Decirle a la familia que la opinión de ellos no importa en nada y que, a
diferencia de ellos, Fausto sí quería ayudar con su órganos y tejidos.
b. Ser empático con el duelo de la familia, pero comunicar que ese era el
deseo de Fausto y que debe continuar con el protocolo de la INDOT.
c. Llamar al equipo de seguridad del hospital y evacuar a los familiares.
d. Esperar a que acuda el abogado de la familia y dialogar con él.

Pregunta 143
María es una paciente de 85 años de edad quien actualmente está
hospitalizada por una neumonía grave de la comunidad. Está desaturando y
usted le plantea la próxima probable necesidad de uso de ventilación
mecánica invasiva. María se niega de forma verbal y explícita al uso de esta
medida, pese a comprender que de esa decisión puede derivarse su muerte.
Está consciente y es capaz y competente para tomar decisiones. Usted cree
que el ventilador sería momentáneo para su recuperación. ¿Qué debe hacer
usted en este caso?
Seleccione una:
a. No intubar a María sería cometer eutanasia, por lo que usted está obligado a
colocarle el ventilador.
b. Como su médico, usted debe seguir el principio de no maleficencia y debe
proceder a la intubación cuando María lo necesite.
c. Debe respetar la decisión autónoma de María y hacer que ella firme su
voluntad en un documento de consentimiento informado.
d. Debe declarar incapaz a María, alegando que por la edad y la infección no
puede decidir de forma correcta.

Pregunta 144
Un residente de primer año de cirugía ha rotado por diversos servicios de cirugía
durante ese año, incluyendo cirugía general, cirugía
cardiotorácica,urología,cirugía oncológica,traumatología y cirugía colorrectal.
Ha conseguido acostumbrarse a recetar generosamente morfina para el
control del dolor. Sin embargo,¿cuál de las siguientes es una contraindicación
absoluta para el uso de opioides?
Seleccione una:
a. Cólico renal
b. Cólico biliar
c. Traumatismo craneoencefálico cerrado
d. Infarto de miocardio

Pregunta 145
Un varón de 23 años es trasladado a urgencias tras encontrarle caminando
desnudo por la calle y proclamándose a sí mismo el hijo de Dios . El análisis de
tóxicos en orinales negativo para drogas y alcohol. Durante la entrevista con el
psiquiatra de guardia,el paciente muestra una fuga de ideas de un tema a otro.
El médico recomienda empezar tratamiento con litio para la manía aguda.
¿Cuál delas siguientes afirmaciones se asocia al tratamiento con litio?
Seleccione una:
a. Pérdida de peso
b. Temblor fino
c. Hipertiroidismo
d. Retención de orina

Pregunta 146
En un paciente con riesgo suicida. ¿Cuál de los siguientes enunciados es un
criterio para hospitalización inmediata?
Seleccione una:
a. No tener ninguna comorbilidad.
b. El alto grado de letalidad del intento suicida.
c. Sexo femenino.
d. Tener una sólida red de apoyo psicosocial.
Pregunta 147
Una joven de 27 años de edad, es abandonada súbitamente en la puerta del
hospital, con múltiples contusiones, estupor e hipotensión severa. Usted realiza
un eco FAST y comprueba que la paciente presenta hemorragia abdominal
masiva por estallido esplénico. Su secuencia de pasos será la siguiente:
Seleccione una:
a. Enviar inmediatamente a la paciente a quirófano, realizando al mismo
tiempo medidas de estabilización y pruebas de diagnóstico (incluyendo
pruebas cruzadas y tipo de sangre).
b. Colocar una vía periférica, administrar cristaloides y 2 U de GRC.
c. Preguntar a la seguridad del hospital si vieron quién abandonó a la paciente.
d. Solicitar pruebas cruzadas, tipo de sangre y programar cirugía cuando la
paciente pueda firmar el consentimiento informado.

Pregunta 148
Correcta
Puntúa 1,00 sobre 1,00
Marcar pregunta
Enunciado de la pregunta
Un paciente con diagnóstico de enfermedad bipolar tipo 1, se encuentra en un
episodio de manía psicótica. Señale la respuesta correcta:
Seleccione una:
a. El paciente debe dar al menos su autorización verbal para poder ingresarse.
b. El paciente debe dar su consentimiento para poder ingresarse al hospital.
c. Podremos tratar al paciente de forma ambulatoria hasta que él decida
ingresar, por sus propios medios.
d. En este caso, se puede prescindir del consentimiento informado, debido a la
falta de conciencia de la enfermedad.

Pregunta 149
¿Qué grupo farmacológico se recomienda en pacientes adultos con depresión
mayor que presentan ideación suicida?
Seleccione una:
a. Antipsicóticos atípicos.
b. Anticonvulsivantes.
c. Benzodiazepinas.
d. Inhibidores selectivos de la recaptación de serotonina.

Pregunta 150
Paciente primigesta de 39 semanas de gestación, que se encuentra en labor de parto. Al
ingreso al centro obstétrico tiene actividad uterina de 4 contracciones en 10 minutos, de 50
segundos de duración y 80 mm Hg de intensidad. A la exploración vaginal: 3 cm de
dilatación y 100% de borramiento, con membranas integras y presentación cefálica en primer
plano, proporción cefalopélvica adecuada. Durante 8 horas ha mantenido la actividad uterina
y los cambios cervicales han alcanzado dilatación de 5 cm y 100% de borramiento.
Frecuencia cardiaca fetal basal de 152 por minuto, ¿Cuál es el siguiente paso a seguir?
Seleccione una:
a. Continuar con el control y evolución espontanea.
b. Terminación del embarazo por vía alta.
c. Administración de oxitocina a dosis respuesta.
d. Rotura artificial de membranas.

Pregunta 151
Complete el siguiente enunciado: Una de las contraindicaciones para el uso del dispositivo
intrauterino de liberación de levonorgestrel (LNG-IUS) es ______ presentando una
tasa de expulsión de _____
Seleccione una:
a. Metrorragia de origen desconocido, 20 %.
b. La existencia de leiomiomas que deformen la cavidad del útero, 10 %.
c. La existencia de leiomiomas que no deformen la cavidad uterina, 30 %.
d. Mujeres infectadas por el virus de inmunodeficiencia humana (VIH), 50%.

Pregunta 152
María de 22 años cursa su primer embarazo sin complicaciones. Al momento: 38.5 semanas
de gestación, actividad uterina leve, y refiere pérdida de tapón mucoso hace una hora. Razón
por la cual acude a un centro médico particular, donde se realiza ecografía que reporta: feto
único, posición cefálica, LCF 130, ILA 7. ¿Cuál es la conducta obstétrica más adecuada a
seguir?
Seleccione una:
a. Indica cesárea de urgencia por sufrimiento fetal.
b. Esperar la evolución normal del parto.
c. Induce el trabajo de parto.
d. Ingreso hospitalario por oligohidramnios.

Pregunta 153
Paciente femenino diagnosticada con verrugas genitales externas. ¿Cuál es el tratamiento
aplicado por el médico?
Seleccione una:
a. Imiquimod 5 % cada noche por 5 semanas.
b. Sinecatequina, al 15% 3 veces a la semana.
c. Podofilox 5 % una vez al mes.
d. Resina de podofilina al 10 – 25 % cada semana.

Pregunta 154
Paciente de 28 años, primigesta, con embarazo de 39 semanas, al tacto vaginal presenta
cuello uterino de consistencia blanda, posición anterior, con dilatación de 3 cm, borramiento
del 60% y la presentación del feto OIIA se encuentra en el III plano de Hodge (localización -
1). ¿A qué índice de Bishop corresponde e indique si es favorable o desfavorable para
inducción de trabajo de parto?
Seleccione una:
a. 8 puntos, desfavorable.
b. 3 puntos, desfavorable.
c. 2 puntos, favorable.
d. 10 puntos, favorable.
Pregunta 155
La velocidad de ganancia de peso, promedio, por semana (kg/sem), en una embarazada
durante el segundo trimestre con un IMC pregestacional normal es de:
Seleccione una:
a. 0.51Kg/sem.
b. 0.28 Kg/sem.
c. 0.22 Kg/sem.
d. 0.42 Kg/sem.

Pregunta 156
A su consulta llega Jacquie de 32 años de edad, que cursa su tercer embarazo. Es su primera
consulta, pero como ya sabe el procedimiento trae exámenes de laboratorio que reportan: Hb
10mg/dl, Hto de 34%, VCM 78fl, según su diagnóstico ¿Cuál sería la conducta más adecuada
para esta paciente?
Seleccione una:
a. Suplementa con hierro oral 120mg + acido fólico.
b. Trasfunde 1 unidad de GRC.
c. Le pide cambios alimenticios.
d. Suplementa con hierro parenteral +acido fólico.

Pregunta 157
El enunciado correcto en referencia al parto pretérmino es:
Seleccione una:
a. La proteína C reactiva en frotis vaginal previene el riesgo de parto pretérmino.
b. El incremento del pH vaginal previene el riesgo de parto pretérmino.
c. Las modificaciones cervicales valoradas por ecografía, son un marcador predictor.
d. El uso de progesterona aumenta la incidencia de parto pretérmino.

Pregunta 158
Durante el Control prenatal específico en mujeres con DM uno de los exámenes a solicitarse
con su debida frecuencia es la ecografía. De la tabla expuesta a continuación, relacione el
período del embarazo con el objetivo para el cual se solicita la ecografía.
Semana gestacional Objetivo de la ecografía
a. Realizar ecografía para valorar crecimiento fetal, perímetro
abdominal fetal y determinación de percentil para descartar
1. Semanas 11 a 14. macrosomía fetal incipiente.
2. Semanas 20 a 22. b. Realizar ecografía para valorar crecimiento fetal y el
volumen del
3. Semanas 28 a 30. líquido amniótico.
4. Semana 36. c. Realizar ecografía para determinar marcadores ecográficos de AC
d. Realizar una ecografía morfológica fetal (detalle anatómico)
Seleccione una:
a. 1a, 2b, 3c, 4d.
b. 1d, 2a, 3b, 4c.
c. 1c, 2d, 3a, 4b.
d. 1b, 2c, 3d, 4a.
Pregunta 169
Una vez que la hemorragia postparto inmediata ha sido identificada, indique el tratamiento
farmacológico de primera opción y la dosis correcta.
Seleccione una:
a. Oxitocina 10Ul/mL lM (o 5 Ul lV lento), o 20-40 Ul en 1000 mL de solución cristaloide en
infusión IV a 250 mL/h.
b. Acido tranexámico 500mg intravenoso a pasar diluido en 200 ml de solución fisiológica y
luego 500 mg intramuscular cada 8 horas.
c. Misoprostol, 200 mcg VO, 1er minuto posparto, 300 mcg cada hora por 6 horas.
d. Ergonovina 0,2 mg IM, 1er minuto, Ergonovina repetir cada 2 a 4 horas, máximo 5 dosis
(1 mg) en un período de 24 horas.

Pregunta 170
Niño de 5 años de edad acude a emergencias con fiebre de 48 horas de evolución. El examen
físico es normal. El residente solicita exámenes y prescribe acetaminofén. ¿Cuál es la dosis
adecuada para este paciente?
Seleccione una:
a. 15 mg/ kg dosis cada 6 horas.
b. 5 mg/Kg dosis cada 4 horas.
c. 30 miligramos/ Kilo / día.
d. 15 miligramos cada 6 horas.

Pregunta 171
Un recién nacido de 32 semanas de edad gestacional, sin antecedentes prenatales de
importancia, nace por parto céfalo vaginal luego de amniorexis instrumental. Presenta desde
el nacimiento quejido, aleteo nasal, retracciones y cianosis. En el examen físico se encuentra
una frecuencia respiratoria de 80 lpm, crepitaciones bilaterales y mala entrada de aire. La
radiografía de tórax revela un volumen pulmonar disminuido, infiltrado retículo granular
bilateral, ¿Cuál de los siguientes factores mejoraría el pronóstico?
Seleccione una:
a. Realizar cesárea electiva
b. Aplicar corticoides prenatales
c. Paciente de sexo masculino
d. Prescribir prostaglandina E2

Pregunta 172
Una niña de 12 años presenta desde hace 2 días temperatura de 40 grados y odinofagia, al
examen físico se encuentra exudado purulento en amígdalas, lengua saburral y petequias en el
paladar, que complicaciones podría presentar esta paciente.
Seleccione una:
a. Glomerulonefritis
b. Artritis febriles
c. Convulsiones febriles
d. Encefalitis
Pregunta 173
Un niño a término, de 4 días de vida acude hoy al centro de salud de sucumbíos para recibir
las vacunas correspondientes a su edad. Hasta el momento no ha recibido ninguna vacuna,
¿qué vacunas le corresponderán el día de hoy?
Seleccione una:
a. Pentavalente, rotavirus
b. BCG, pentavalente
c. Hepatitis a, neumococo
d. Hepatitis B, BCG

Pregunta 174
La administración de zinc a un niño con diarrea aguda, según las recomendaciones de la
OMS, tiene las siguientes normas y ventajas. EXCEPTO:
Seleccione una:
a. Disminuye el riesgo de diarrea, en los 2 o 3 meses siguientes.
b. Restituye parte de los electrolitos perdidos durante la diarrea.
c. Deberá realizarse desde el comienzo de una diarrea aguda.
d. Reduce la duración, la gravedad y el riesgo de deshidratación.

Pregunta 175
Un lactante nacido a término presenta índices de Apgar de 7 y 10 a 1 y 5 min. En la
auscultación torácica se identifica un soplo cardíaco. Se observan hepatoesplenomegalia y
cataratas en el cristalino. El niño está en el percentil 30 de talla y peso. La ecografía muestra
un conducto arterioso persistente. ¿Cuál de los siguientes factores de riesgo es más probable
en este lactante?
Seleccione una:
a. Infección congénita por rubéola2
b. Carencia de folato en la dieta
c. Dispermia en la concepción
d. Eritroblastosis fetal

Pregunta 176
Correcta
Puntúa 1,00 sobre 1,00
Marcar pregunta
Enunciado de la pregunta
Una niña de 8 años sin antecedentes de importancia acude a emergencia con una crisis
asmática aguda leve, ¿Cuál de los siguientes fármacos es de primera elección en este cuadro?
Seleccione una:
a. Salmeterol
b. Trofilina
c. Sulfato de magnesio
d. Salbutamol

Pregunta 177
Un niño de 10 meses de edad se atoró con la llanta de un juguete. Usted debe realizar la
maniobra de Heimlich. ¿Cómo realizaría la misma a esta edad?
Seleccione una:
a. Con el niño en posición de Tredelemburg realizar una compresión en el cartílago cricoides.
b. Posicionar al niño boca abajo sobre el antebrazo del reanimador y dar 5 palmadas entre los
omóplatos.
c. Sostener al niño sentado en sus piernas y aplicarle una compresión con el puño de su mano
en el epigastrio.
d. Colocar al niño boca arriba y realizar con el puño dos compresiones debajo del apéndice
xifoides.

Pregunta 178
Una paciente de 5 años es ingresada con diagnóstico de desnutrición grave de origen
primario, ¿Con cuál de estas complicaciones está asociada la mortalidad en las primeras horas
de ingreso?
Seleccione una:
a. Anemia y desnutrición.
b. Hipotermia e Hipoglicemia.
c. Insuficiencia hepática y sepsis.
d. Insuficiencia cardiaca y renal.

Pregunta 179
Un neonato a término presenta a las 3 horas de vida hipoglicemia, de entre la siguiente lista.
¿cuál de los siguientes signos permitiría sospechar un hiperinsulinismo?
Seleccione una:
a. Macrosomia
b. Peso bajo
c. Temblores
d. Visceromegalias

Pregunta 180
El tratamiento de cetoacidosis diabética es:
a. Administración de bicarbonato.
b. Tratar la alcalosis metabólica.
c. Venoclisis con solución salina.
d. Colocar insulina de acción prolongada

Pregunta 181
Un niño de 5 años de edad acude por primera vez a su consulta, para control de niño sano. Es
un niño activo, comunicativo. Su peso y talla se encuentran en el percentil 50. Juega fútbol y es
amiguero. Sin embargo, a la auscultación cardíaca presenta soplo sistólico mesocárdico, grado
II/VI, sin una irradiación definida y que, con los cambios de posición del niño, disminuye
notablemente de intensidad. Seleccione el literal correspondiente a esta descripción:
a. Comunicación interventricular (CIV).
b. Comunicación interauricular (CIA).
c. Persistencia del conducto arterioso (PCA).
d. Soplo funcional o inocente.

Pregunta 182
¿Cuál es la indicación para referir a un paciente a un centro especializado para la atención de
quemados?
a. Quemaduras de espesor parcial con un compromiso de 5% TBSA.
b. Quemaduras en pacientes sin comorbilidades.
c. Quemadura de segundo grado en cualquier edad.
d. Quemaduras químicas, de cualquier mecanismo de acción.

Pregunta 183
En el embarazo los cambios fisiológicos como sensación de ardor estomacal y la pirosis se
atribuyen a:
a. Presiones intraesofágicas más altas e intragástricas más bajas.
b. Hipotonía del esfínter esofágico inferior.
c. Mayor velocidad de vaciamiento gástrico.
d. Cambio de posición del estómago en el primer trimestre.

Pregunta 184
Una paciente nuligesta presenta ciclos menstruales regulares con FUM hace 7 días, al
examen físico tiene dolor anexial bilateral, la movilización del cuello uterino y palpación
directa de abdomen son dolorosas, adicionalmente tiene 38.5 grados C y leucocitosis de 12
000 / ml. ¿Cuál es el diagnóstico?
a. Infección de vías urinarias.
b. Ruptura de quiste del cuerpo lúteo.
c. Endometriosis pélvica.
d. Enfermedad pélvica inflamatoria.

Pregunta 185
Respecto a las claves obstétricas, la toma de muestra de lactato sérico corresponde a:
a. CLAVE NARANJA para manejo de traumatismo obstétrico.
b. CLAVE ROJA para manejo de hemorragia obstétrica.
c. CLAVE AMARILLA para manejo de choque séptico obstétrico.
d. CLAVE AZUL para manejo trastornos hipertensivos obstétricos.

Pregunta 186
Paciente de 32 años de edad, sin antecedentes personales de importancia, G4 P3, acude al
Hospital Gíneco-Obstétrico Isidro Ayora con un embarazo de 32 semanas, cefalea, TA de
150/120,
edema en piernas, cara y manos, ROTS 3/5. Sus indicaciones de ingreso a la paciente
incluyen los
siguientes medicamentos. EXCEPTO:
a. Betametasona 12 mg IM 2 dosis cada 24 horas.
b. Nifedipina 10 mg VO cada 20 minutos.
c. Sulfato de magnesio IV 6 g de carga y 1 g/hora mantenimiento
d. Hidralazina IV bolo de 5 mg stat.

Pregunta 187
¿Qué mecanismo fisiológico en el embarazo produce incremento de la excreción urinaria de
proteínas?
a. Disminución de la tasa de filtración glomerular.
b. Disfunción endotelial glomerular.
c. Disminución de la reabsorción tubular de las proteínas filtradas.
d. Alteración del sistema renina-angiotensina-aldosterona.
Pregunta 189
Algunas parasitosis intestinales pueden causar la pérdida constante de sangre dentro del
huésped. Seleccione cual parasitosis intestinal causa anemia ferropénica en casos de
infestación moderada a severa:
a. Amibiasis.
b. Giardiasis
c. Uncinariasis.
d. Balantidiasis.

Pregunta 190
Un hombre de 44 años de edad llega por primera vez a su consulta. En la sala de espera se le
solicita que llene varios formularios, con datos de su historial médico familiar y personal, ante
lo cual el paciente asiente amablemente. En la consulta, el médico constata que los
documentos están vacíos, a pesar de que el paciente ha tenido el tiempo necesario para
llenarlos. Al preguntarle ¿por qué no llenó los documentos? el paciente dice en tono
sarcástico: "estoy seguro que un médico con
su fama y educación puede hacer una historia clínica". El médico le pregunta al paciente si
todo está bien, con lo que el paciente responde: “Si, todo está de maravilla, no hay ningún
problema”; Indique. ¿Cuál de las siguientes opciones describe la actitud del paciente?
a. Proyección.
b. Agresión pasiva.
c. Desplazamiento.
d. Comportamiento impulsivo.

Pregunta 191

Respecto al protocolo de manejo en la Clave Roja obstétrica, todas las acciones son correctas.
EXCEPTO:
a. Asegurar vía aérea y oxígeno suplementario para conseguir saturación mayor a 85%.
b. Realizar examen físico completo que incluya signos vitales completos y nivel de conciencia.
c. Asegurar dos accesos venosos con catéter No. 16 e infusión rápida de líquidos calientes.
d. Vaciar vejiga y colocar sonda vesical a drenaje con bolsa de recolección

Pregunta 192
En un niño con diarrea aguda viral y que presenta manifestaciones de deshidratación
moderada, se deberá administrar, en las primeras 4 horas, la solución de rehidratación oral de
la OMS, a la dosis de:

a. 30ml/kg.
b. 75 ml/kg.
c. 90 ml/kg.
d. 200 ml/kg.

Pregunta 193
Un niño de 4 años de edad, con 16 kg de peso, presenta desde hace 4 días un proceso
respiratorio caracterizado por tos productiva, con esputo café amarillento y fiebre. En las
últimas 24 horas se encuentra con mayor astenia, respiración agitada y anorexia. Toma muy
pocos líquidos. T: 39,2 grado C; FC: 116 x 1; FR: 36 x 1; Sus labios están secos y respira con la
boca entreabierta. Se observa tiraje intercostal y subcostal moderado. A la auscultación existe
una notable disminución del ingreso del aire en la base derecha. Biometría: leucocitos 18
000/mm3; Polimorfonucleares: 78%; Linfocitos 17%; Eosinófilos: 2%: Monocitos: 3%. La Rx
demuestra consolidación del parénquima pulmonar en la base derecha. Se diagnostica
Neumonía. Seleccionar el germen productor de la neumonía y el
antimicrobiano más adecuado para su tratamiento:
a. Streptococcus pyogenes, que debe ser tratado con cefotaxima IV.
b Staphylococcus aureus, debe tratarse con ampicilina IV.
c. Streptococcus pneumoniae, el tratamiento indicado es penicilina IV.
d. Haemophilus influenzae, debe ser tratada con penicilina benzatínica.

Pregunta 194
La presencia de los siguientes elementos en un niño de 4 años de edad, sugiere maltrato
infantil. EXCEPTO:
a. Estreñimiento a petición.
b. Numerosas cicatrices en la piel.
c. Niño o niña con manifiesta timidez.
d. Antecedentes de fracturas repetidas.

Pregunta 195
Seleccione los parámetros radiográficos evaluados para diagnosticar displasia del desarrollo
de cadera.
1. Acetabular.
2. Barlow.
3. Perkins.
4. Hilgenreiner.
5. Shenton.
6. Ortolani.
a. 1, 3, 4, 5
b. 2, 3, 5, 6
c. 1, 2, 4, 5
d. 1, 3, 5, 6

Pregunta 196
El principal síntoma o signo predictor de prescripción antibiótica en Bronquitis aguda es:
a. Disnea.
b. Fiebre.
c. Esputo blanquecino.
d. Tos.

Pregunta 197
La regla de los 9 puede utilizarse como una referencia rápida para calcular en un paciente
quemado, el área de superficie corporal comprometida. ¿Cuál es el porcentaje
comprometido en un paciente con una quemadura de todo el tronco anterior y el miembro
superior derecho?
a. 27%.
b. 18%.
c. 36%.
d. 19%.

Pregunta 198
Los siguientes enunciados en relación a Herpes Zóster. EXCEPTO:
a. Si la función inmunitaria disminuye luego de la infección primaria, el virus se reactiva en los
ganglios
sensitivos, desciende por los nervios y se multiplica.
b. Se caracteriza por disestesias unilaterales, erupción vesicular en la piel, en el trayecto del
dermatoma o
los dermatomas inervados por el ganglio sensitivo correspondiente.
c. Durante la infección primaria, el virus de varicela-zóster infecta los ganglios sensitivos y
persiste en éstos,
en forma latente, durante el resto de la vida.
d. La administración de la vacuna de virus vivo atenuado contra el virus Herpes zóster (VZV),
favorece el
desarrollo de Herpes zóster durante la tercera edad.

Pregunta 199
Señale la afirmación correcta relacionada a los signos y síntomas de la apendicitis aguda:
a. La falta de apetito temporal es un síntoma poco frecuente en cuadros apendiculares.
b. El dolor durante el examen rectal es característico en los apéndices inflamados de
localización paracecal ascendente.
c. El signo de Rovsing es el dolor que se presenta en el cuadrante inferior derecho al ejercer
presión en el cuadrante inferior izquierdo.
d. El signo de Blumberg es positivo cuando hay dolor en fosa ilíaca izquierda al comprimir fosa
ilíaca derecha.

Pregunta 200
RN a término, Peso al nacimiento: 2,3 kg; Talla: 49 cm; PC: 35 cm; APGAR: 7 / 10; es reactivo
al manejo, su respiración es normal y presenta buen reflejo de succión. Se lo mantiene en
observación. A las 4 horas de edad presenta glucemia de 50 mg/dl. Aún no se lo ha puesto en
contacto con su madre. Determine el diagnóstico y terapéutica:
a. Retardo de crecimiento intrauterino; lactancia materna a libre demanda + controles de
glucemia.
b. Retardo de crecimiento intrauterino; dextrosa al 5% en AD + controles de glucemia.
c. Retardo de crecimiento intrauterino; dextrosa al 10% en AD + controles de glucemia.
d. Control insuficiente en el embarazo; dextrosa al 10% en AD + controles de glucemia.

Pregunta 201
Usted va a empezar a realizar una investigación observacional analítica para buscar la
relación que existe entre la obesidad y los videos juegos en estudiantes de secundaria,
mayores de 13 años y menores de 18 años. Para la expresión de voluntariedad y ejercicio de
autonomía. ¿Qué documentos deben firmar los participantes?
a.Consentimiento informado del participante y de sus padres o representante legal.
b. Consentimiento informado firmado por el participante y la carta de aprobación del director
escolar.
c. Consentimiento informado firmado por los padres y la carta de aprobación del director
escolar.
d. Carta de aprobación del director(a) del colegio considerando que es un estudio
observacional analítico.

Pregunta 202
La tos crónica con imagen radiográfica normal, se debe a la administración de:
a. Amitriptilina.
b. Captopril.
c. Dextrometorfán.
d. Hidrocodona.

Pregunta 203
La celulitis en diabéticos ancianos es ocasionada por:
a. Estafilococo.
b. Haemophilus influenzae.
c. Pasteurella multocida.
d Streptococcus agalactiae.

Pregunta 204
De los siguientes casos, seleccione aquel que requiere la firma del consentimiento
informado.
a. Situaciones de emergencia.
b. Tratamientos exigidos por la ley como en pandemias.
c. Papanicolaou para despistaje de lesiones pre-invasoras.
d. Procedimientos de reproducción asistida.

Pregunta 205
51. Los siguientes son síntomas asociados a hipotiroidismo en edad escolar. EXCEPTO:
a. Disminución del volumen tiroideo.
b. Anemia por deficiencia de hierro.
c. Aumento del colesterol y triglicéridos.
d. Déficit sutil del coeficiente intelectual.

Pregunta 206
A su consulta acude un paciente de 3 años de edad por presentar tos desde hace 4 días. Al
examen físico el paciente se encuentra febril (38 grados C), frecuencia cardiaca de 95 por
minuto, frecuencia respiratoria de 52 por minuto y saturación de oxígeno 93%. No se
evidencia estridor y a la auscultación pulmonar no presenta disminución de murmullo
vesicular ni sibilancias. El diagnóstico del paciente es:
a. Bronquitis aguda.
b. Neumonía.
c. Resfriado común.
d. Crup viral.

Pregunta 207
Seleccione el literal que describe la disentería en niños en menores de 5 años:
a. Vómito persistente, inicio súbito de deposiciones diarreicas líquidas, abundantes, muy
frecuentes y acompañadas de deshidratación progresiva y rápida, con colapso vascular siendo
esta infección producida
por la bacteria Vibrio Cholerae.
b. Vómito, fiebre, cefalea, diarreas líquidas, abundantes, mucosas, con mal olor, dolor
abdominal y persistencia de las diarreas durante 5 a 7 días. Puede haber somnolencia y
convulsiones en algunos pacientes y esta infección es producida por la bacteria Salmonella
spp.
c. Dolor abdominal tipo cólico, tenesmo, fiebre alta y en ocasiones convulsiones febriles,
diarrea mucopurulenta y sanguinolenta, de cantidad variable. El paciente puede presentar
somnolencia y es producida por la bacteria Shigella spp.
d. Dolor abdominal tipo cólico postprandial, luego diarreas líquidas, abundantes, explosivas,
blanquecinas, espumosas, mal olientes, con restos alimentarios. Hay flatulencia intensa y es
producida por Giardia lamblia
Pregunta 208
¿Cuál de los siguientes fármacos usados para el tratamiento de diabetes disminuye la
producción hepática de glucosa?
a. Nateglidina.
b. Rosiglitazona.
c. Sulfanilureas.
d. Biguanidas.

Pregunta 209
En revisión secundaria, indique las lesiones potencialmente fatales en trauma de tórax.
1. Hemotórax masivo.
2. Neumotórax simple.
3. Lesión cardiaca cerrada.
4. Taponamiento cardiaco.
5. Lesión cerrada de esófago.
a. 1,2,3.
b. 1,3,5.
c. 2,3,4.
d. 2,3,5.

Pregunta 210
Paciente varón de 65 años, bebedor habitual, que se presenta con intenso dolor epigástrico
después de una comida abundante, describe al dolor como “puñalada”, con irradiación a la
espalda y que se alivia cuando se inclina hacia adelante. Refiere náusea y vómito
abundantes. Al examen físico presenta FC: 112 latidos por minuto, TA: 90/60 mmHg,
temperatura: 38,8 grados C; el abdomen se observa distendido, con defensa involuntaria y
dolor al palpar todo el hemiabdomen superior, los
ruidos hidroaéreos están disminuidos. En los exámenes de laboratorio se encuentra
hemoconcentración, hiperglucemia, hiperazoemia e hiperamilasemia. Se realizó ecografía
que no dio información útil por la presencia de abundantes gases intestinales. Se solicitó TAC
que se muestra en la imagen. ¿Cuál es el diagnóstico a plantear en este paciente?
a. Pancreatitis aguda edematosa leve.
b. Úlcera péptica gastroduodenal perforada.
c. Apendicitis aguda complicada.
d. Pancreatitis aguda necrosante grave.

Pregunta 211
De los siguientes criterios. ¿Cuál está asociado con la presencia de una pancreatitis aguda
grave?
a. Escala de APACHE II mayor a 8.
b. Escala de BISAP de 1.
c. Escala de RANSON de 1.
d. Presencia de 1 criterio de Ranson

Pregunta 212
El tratamiento de infección extensa por tiña es:
a. Caspofungina.
b. Amfotericina.
c. Micafungina.
d. Terbinafina.
Pregunta 213
Indique el enunciado perjudicial en pacientes con diabetes:
a. Consumo de edulcorantes sin calorías.
b. Dieta hipocalórica baja en carbohidratos
c. Consumo de antioxidantes u oligoelementos.
d. Consumo mínimo de grasas trans.

Pregunta 214
Relacione las causas de bocio endémico con sus características:
Causas Características
1. Alteraciones del metabolismo del yodo.
2. Bociógenos.
a) Interfieren en la captación del yodo.
b) Aclaramiento renal de yodo aumentado.
c) Déficit en el aporte de yodo.
d) Interfieren en la producción hormonal.
a. 1bc, 2ad.
b. 1ab, 2cd.
c. 1bd, 2ac.
d. 1ac, 2bd.
Pregunta 215
Un síntoma de la anemia por deficiencia de complejo B es:
a) Síndrome de mano-pie.
b) Priapismo.
c) Abombamiento craneal.
d) Glositis.

Pregunta 216
Las benzodiacepinas son utilizadas en el tratamiento sintomático de los trastornos de
pánico.
¿Cuál es el mecanismo farmacológico por el que logra su efecto ansiolítico?
a) La acción del fármaco sobre el receptor NMDA.
b) La acción del fármaco sobre el receptor GABA-A.
c) La acción del fármaco sobre el receptor 5HT1.
d) La acción del fármaco sobre el receptor D2.

Pregunta 217
La madre lleva a la consulta a su niño de 5 años de edad por cuanto presenta lesiones de piel
que aparentemente se exacerban con el consumo de alimentos. ¿Cuál diagnóstico está
relacionado con la dieta?
a) Dermatitis atópica.
b) Dermatitis herpetiforme.
c) Dermatitis de contacto.
d) Acné vulgaris.

Pregunta 218
El paludismo se caracteriza por la presencia de crisis febriles que tienen una determinada
periodicidad, la misma que orienta sobre el tipo de Plasmodium infectante. Los episodios
febriles se presentan con intervalos variables y conocidos, que en la mayoría de ellos son
conocidos por su periodicidad. Seleccione el tipo de Plasmodium que presenta estos
episodios clínicos de fiebre con el mayor intervalo de tiempo:
a. P. ovale.
b. P. malariae.
c. P. vivax.
d. P. falciparum.

Pregunta 219
La radiografía de caderas de una niña de 3 meses de edad presenta signos de luxación
congénita de su cadera izquierda. Identificar el literal que describe en mejor forma las
características radiográficas de esta patología:
a. Hipoplasia del núcleo femoral izquierdo, ángulo acetabular izquierdo de 26 grados, línea de
Perkins alineada desde el extremo lateral del techo acetabular izquierdo con la epífisis femoral.
b. Presencia del núcleo femoral izquierdo, ángulo acetabular izquierdo de 28 grados, línea de
Perkins desplazada hacia afuera del techo acetabular izquierdo.
c. Hipoplasia del núcleo femoral izquierdo, ángulo acetabular izquierdo de 36 grados, línea de
Perkins desplazada hacia afuera del techo acetabular izquierdo.
d. Presencia del núcleo femoral izquierdo, ángulo acetabular izquierdo inferior de 27 grados,
línea de Perkins alineada desde el extremo lateral del techo acetabular izquierdo con la epífisis
femoral.

Pregunta 220
Las siguientes afirmaciones son correctas en relación a la rubéola. EXCEPTO:
a. La fiebre de la rubéola es alta, dura 3 días y coincide con el exantema generalizado.
b. Presenta una erupción máculo papular eritematosa, leve, acompañada de linfadenopatías.
c. En adolescentes y adultos, presenta con frecuencia poliartralgias y poliartritis transitorias.
d. Es una enfermedad usualmente benigna, excepto para el embrión durante el embarazo.

Pregunta 221
A su consulta es llevado un paciente masculino de 6 meses de edad por sus padres indicando
que no sienten el testículo del lado derecho del niño. Ellos indican que hace 3 meses
acudieron a un médico quien les indico que el testículo descendería posteriormente. El
examen físico revela que el testículo derecho se encuentra en el canal inguinal. Señale cual
es el manejo en este tipo de paciente.
a. Esperar hasta el año de edad para que el testículo descienda.
b. Realizar una ecografía diagnóstica.
c. Indicar que el hallazgo es normal y que en la infancia se corregirá.
d. Referirlo a urología pediátrica.

Pregunta 222
Paciente masculino de 25 años de edad, con antecedentes familiares de acné quístico grave,
presenta lesiones que iniciaron con comedones abiertos y cerrados, que se transformaron en
lesiones pápulo-pustulosas, con aumento de tamaño y difusión, lo cual es característico del
acné noduloquístico. Las lesiones son muy dolorosas y presentan una distribución simétrica
en la cara.
¿Cuál es la gravedad del acné y su tratamiento?
a. Acné leve. Manejo con antibióticos vía oral, minociclina, 50 a 100 mg/día o doxiciclina, 50 a
100 mg cada 12h.
b Acné moderado. Manejo con retinoides tópicos (ácido retinoico, adapaleno, tazaroteno) .
c. Acné grave. Manejar con tratamiento tópico y tratamiento sistémico con isotretinoina.
d. Acné leve. Manejo con antibióticos tópicos (clindamicina y eritromicina) y geles con
peróxido benzoico.

Pregunta 223
El tratamiento de la Esporotricosis linfocutánea es:
a. Fluocitosina.
b. La Terbinafina.
c. Itraconazol.
d. Anfotericina B.

Pregunta 224
Un lactante mayor de 24 meses de edad tiene un peso de 10kg, una talla de 70 cm y un
perímetro cefálico de 43 cm. El peso está bajo el percentil 5, la talla en el percentil 5 y el
perímetro cefálico en el percentil 25. Al examen físico presenta astenia, reducción del tejido
celular subcutáneo y de las masas musculares. Abdomen algo prominente. Piel seca, pelo
escaso, descolorido, sin brillo y fácilmente desprendible. No presenta visceromegalias. Si el
percentil 50 del peso para esta edad es
de 12 kg, su pérdida de peso es de 16.6%. Seleccione el diagnóstico más probable para este
niño:
a. Desnutrición calórica proteica de primer grado.
b. Desnutrición calórica proteica de tercer grado.
c. Desnutrición calórica proteica de segundo grado.
d. Desnutrición proteica calórica o kwashiorkor.

Pregunta 225
¿En qué grupo poblacional se recomienda tratar siempre una bacteriuria asintomática?
a. Pacientes pediátricos.
b. En todos los grupos poblacionales sin excepción.
c. Tercera edad.
d. Embarazadas.

Pregunta 226
Los exámenes indicados determinan el diagnóstico etiológico de la neumonía. EXCEPTO:
a. Cultivo de esputo.
b. Hemocultivo.
c. Proteína C reactiva.
d. Prueba urinaria con antígeno.

Pregunta 227
Niño de 9 años, acude a consulta con tos leve con expectoración mucosa, disnea espiratoria,
el cuadro se ha presentado por algunas ocasiones. Al examen físico se encuentra algo
cianótico, se auscultan sibilancias e hipertimpanismo. Una de las sospechas diagnósticas es
el asma, con la finalidad de confirmar o descartar el diagnóstico usted solicita algunos
estudios funcionales pulmonares. Los siguientes son resultados de pruebas funcionales
pulmonares normales.
EXCEPTO:
a. FEV 1/ FVC = 90%.
b. FEF 25-75% = 86%.
c. FVC menor a 68%.
d. FEV 1 89%.
Pregunta 228
A un paciente con un primer episodio psicótico. ¿Qué se aplica una vez instaurado el
tratamiento
farmacológico y de apoyo terapéutico?
a. Tratamiento de desintoxicación.
b. Plan de actividad física y nutricional.
c. Internación psiquiátrica forzosa.
d. Tratamiento en hospital del día.

Pregunta 229
Paciente adulto con antecedente de traumatismo hace dos semanas que presenta Hb de 16
g/dl, albúmina menor a 2.8 g /100 ml, linfocitopenia de 1500/μl, con deficiente cicatrización
de heridas, desprendimiento fácil del cabello y edema, estatura normal, ¿Cuál es el
diagnóstico?
a. Kwashiorkor.
b. Marasmo.
c. Cretinismo.
d. Anemia.

Pregunta 230
Niña de 1 año 6 meses que durante su control de salud presenta la siguiente curva de
crecimiento:

¿Cómo se interpreta la siguiente curva del gráfico anterior?


a. La niña presenta una tendencia de crecimiento normal baja para la edad.
b. La niña no está subiendo de peso y tiene una señal de peligro.
c. La niña no ha subido mucho de peso, pero a su edad este comportamiento es normal.
d. La niña ha subido poco de peso y esto nos alerta de algún proceso patológico.

Pregunta 231
Paciente femenina con enfermedad catastrófica, solicita a su médico le ayude a terminar con
su vida, este le administra una sustancia específica a sabiendas que causará la muerte de la
paciente. Según el Código Integral Penal del Ecuador. ¿Qué tipo de delito comete el médico?
a. Abandono de personal.
b. Sicariato.
c. Femicidio.
d. Asesinato.

Pregunta 232
Un paciente varón, de 55 años de edad, presenta los siguientes datos clínicos: presión
arterial, 150/90 mm Hg; perímetro abdominal, 90 cm; glucemia en ayunas, 150 mg/dl;
triglicéridos, 179 mg/dl; colesterol HDL, 25 mg/dl. De acuerdo con los datos expuestos. ¿Cuál
de las siguientes opciones corresponde al diagnóstico de síndrome metabólico?
a. Presenta 4 de los 5 criterios diagnósticos, por tanto, tiene síndrome metabólico.
b. Presenta 2 de los 5 criterios diagnósticos, por tanto, tiene síndrome metabólico.
c. Presenta 4 de los 5 criterios diagnósticos, por tanto, no tiene síndrome metabólico.
d. Presenta 3 de los 5 criterios diagnósticos, por tanto, no tiene síndrome metabólico.

Pregunta 233
94. Las siguientes son indicaciones de laparotomía en pacientes con lesión penetrante de
abdomen. EXCEPTO:
a. Signos de irritación peritoneal.
b. Alteración hemodinámica.
c. Signos de penetración de la fascia.
d. Distención abdominal.

Pregunta 234
Seleccione la respuesta correcta sobre profilaxis antibiótica:
a. La profilaxis antibiótica administrada correctamente previene infecciones nosocomiales.
b. La profilaxis antibiótica está indicada para la mayoría de las cirugías sucias.
c. La profilaxis antibiótica se utiliza sobre todo para tratar la infección del sitio quirúrgico.
d. La profilaxis antibiótica administrada incorrectamente es ineficaz e incluso perjudicial.

Pregunta 235
El régimen antibiótico de erradicación de cepas resistentes de Helycobacter pylori es:
a. Amoxicilina + levofloxacina + tinidazol.
b. Furazolidina + claritromicina + tetraciclina
c. Rifabutina + Tinidazol + claritromicina.
d. Metronidazol + rifabutina + tinidazol

Pregunta 236
Indique en cuál de las siguientes patologías hipertensivas se utiliza fentolamina:
a. Encefalopatía hipertensiva.
b. Hipertensión arterial maligna.
c. Feocromocitoma.
d. Disección aórtica.

Pregunta 237
El tratamiento de una criptorquidia verdadera en un lactante de 9 meses de edad es:
a. Inducción hormonal para descenso testicular.
b. Tratamiento expectante hasta el año de edad.
c. Orquidopexia.
d. Tratamiento expectante hasta los 3 años de edad.

Pregunta 238
Los siguientes enunciados sobre prevención o tratamiento de las complicaciones en
pacientes diabéticos, son correctos. EXCEPTO:
a. Aspirina para cadioprotección está contraindicada en presencia de retinopatía diabética.
b. Duloxetina es uno de los tratamientos iniciales para el dolor neuropático.
c. Aspirina está indicada para cardioprotección en mayores de 50 años con riesgo
cardiovascular.
d. Pregabalina es uno de los tratamientos iniciales para el dolor neuropático.

Pregunta 239
En los cuadros de artropatía psoriásica. ¿Cuál de los siguientes parámetros clínicos o de
laboratorio es más común entre las mujeres en relación a los hombres?
a. Tener artritis de afectación solo distal.
b. Tener artritis deformante.
c. Ser seropositivo para factor reumatoide.
d. Tener artritis de afectación simétrica.
Pregunta 240
¿Cuál es una contraindicación absoluta para el uso de anticonceptivos orales combinados?
a. Historia de trombosis.
b. Historia de dismenorrea.
c. Historia de cáncer de cérvix.
d. Historia de cefalea tensional.

Pregunta 241
Síntomas y signos de neumotórax a tensión. EXCEPTO:
a. Desviación traqueal.
b. Dificultad respiratoria.
c. Ruidos cardiacos apagados.
d. Dolor torácico.

Pregunta 242
En relación a neurocisticercosis, las siguientes afirmaciones son correctas. EXCEPTO:
a. La neurocisticercosis se trata con antiparasitarios como metronidazol o tinidazol sumado a
AINES para manejo del dolor.
b. Los cisticercos y sus quistes, corresponden a la fase larvaria de las tenias del cerdo, como es
la Taenia solium.
c. Se produce por ingestión de huevos y proglótides a través de alimentos contaminados con
cisticercos.
d. Los cisticercos, después de morir, pueden producir sintomatología inflamatoria significativa
y convulsiones.
Pregunta 243
Llega a la consulta una mujer de 27 años con un retraso en su menstruación de 2 meses. La
anamnesis revela un aumento de 5 kg durante 6 meses anteriores (IMC 27), sequedad de
piel, intolerancia al frío e irritabilidad. Durante el examen físico no se evidencias signos
alarmantes. Una prueba de BhCG es negativa. Con su posible diagnóstico según la clínica.
¿Cuál es el tratamiento?
a. Bromocriptina.
b. Levotiroxina.
c. Pastillas anticonceptivas orales.
d. Metformina.

Pregunta 244
¿Cuál de los siguientes signos ecográficos corresponde a colecistitis aguda?
a. Presencia de litos al interior de la vesícula.
b. Murphy ecográfico.
c. Pared vesicular de 2 mm de diámetro.
d. Vesícula biliar con un volumen de 30 ml.

Pregunta 245
A su consulta acude un paciente masculino de 67 años de edad por presentar pirosis desde
hace 4 semanas relacionado con la ingesta alimentaria. El paciente no presenta historia
personal o familiar de cáncer gastrointestinal. Niega consumo de cigarrillo y consume alcohol
ocasionalmente. Niega dificultad para deglutir, refiere sensación de plenitud gástrica,
melenas o pérdida de peso. El examen físico del paciente es normal, excepto por leve dolor a
nivel epigástrico a la palpación. ¿Cuál de los siguientes procedimientos se recomienda en el
paciente?
a. Investigación de sangre oculta en heces.
b. Pruebas funcionales hepáticas.
c. Estudios de imagen.
d. Endoscopia digestiva alta.

Pregunta 246
¿Cuál de los siguientes signos ecográficos corresponde a colecistitis aguda?
a. Presencia de litos al interior de la vesícula.
b. Murphy ecográfico.
c. Pared vesicular de 2 mm de diámetro.
d. Vesícula biliar con un volumen de 30 ml.

Pregunta 247
A su consulta acude un paciente masculino de 67 años de edad por presentar pirosis desde
hace 4 semanas relacionado con la ingesta alimentaria. El paciente no presenta historia
personal o familiar de cáncer gastrointestinal. Niega consumo de cigarrillo y consume alcohol
ocasionalmente. Niega dificultad para deglutir, refiere sensación de plenitud gástrica,
melenas o pérdida de peso. El examen físico del paciente es normal, excepto por leve dolor a
nivel epigástrico a la palpación. ¿Cuál
de los siguientes procedimientos se recomienda en el paciente?
a. Investigación de sangre oculta en heces.
b. Pruebas funcionales hepáticas.
c. Estudios de imagen.
d. Endoscopia digestiva alta.

Pregunta 248
Los siguientes son factores de riesgo, en mayor o menor grado, para desarrollar enfermedad
pulmonar obstructiva crónica (EPOC). EXCEPTO:
a. Ser fumador pasivo por largo tiempo.
b. Exposición al polvo de algodón y carbón.
c. Neumonía o bronquitis.
d. Tabaquismo prolongado.

Pregunta 249
En una evaluación nutricional se reporta que Rosario de 20 años tiene una ingesta diaria de
50 gramos de proteína, 100 gramos de grasa y 400 gramos de hidratos de carbono. ¿Cuál
sería la ingesta
calórica total?
a. 4200 calorías/día.
b. 2700 calorías/día.
c. 2450 calorías/día.
d. 2200 calorías/día.

Pregunta 250
Una mujer de 36 años, casada, empleada pública, presenta desde hace 7 meses episodios de
taquicardia, falta de aire, temblor y mareo, que se repiten dos veces al mes. Define a estos
episodios como que “siente que va a morir”. Los estudios de ECG y de hormonas tiroideas,
son normales. Adicionalmente, tiene miedo a salir de casa y evita ciertas situaciones
específica. Cuando está en compañía o se convence de que no pasará nada supera ese
miedo. Al examen psiquiátrico la paciente se manifiesta muy intranquila, pide ayuda al
médico constantemente, está orientada, no se objetivan alteraciones de la sensopercepción,
presenta ideas fijas con respecto a su padecimiento somático, tiene una leve hipertimia
displacentera y se encuentra eubúlica. Le administraron una benzodiacepina y el episodio
cede, aunque persiste el miedo a que se repita. ¿Cuál es el diagnóstico de esta paciente?
a. Trastorno de ansiedad social.
b. Trastorno de ansiedad generalizada.
c. Trastorno de pánico.
d. Fobia específica.

Pregunta 251
Paciente acude con su hijo de 12 meses a vacunación en el centro de salud. Le corresponden
vacunas de sarampión, rubeola, parotiditis (SRP) y fiebre amarilla. La paciente indica a la
enfermera que le administre solamente la vacuna de SRP. ¿Cuál es la conducta a seguir en
esta situación?
a Se le solicita que firme el consentimiento informado donde rechaza el procedimiento y no se
le aplica ninguna vacuna porque el esquema debe completarse estrictamente.
b. Brindar información sobre los beneficios y riesgos de la vacuna de fiebre amarilla, y firmar el
consentimiento informado donde rechaza el procedimiento y se aplica sólo SRP.
c. Se le sugiere que acuda a un centro privado de vacunación para que le apliquen vacuna
contra la fiebre amarilla y que firme el consentimiento informado que rechaza el
procedimiento.
d. Se le brinda información sobre los beneficios y riesgos de la vacuna de fiebre amarilla y se le
solicita que regrese otro día una vez que reflexione sobre la información brindada
Pregunta 252
María es una niña que al examen para evaluar el desarrollo psicomotor demuestra que
puede comer galletas sola, imita palabras y dice de forma inespecífica papa, mama, pasa un
cubo de una mano a otra, está sentada con apoyo y es tímida con extraños. Escoja la opción
que corresponde a la edad aproximada de la niña.
a. 6 meses.
b. 9 meses.
c. 12 meses.
d. 3 meses.

Pregunta 253
En la mujer con diabetes gestacional es importante la identificación de los factores de riesgo.
Los siguientes son factores de riesgo alto. EXCEPTO:
a. Síndrome de ovario poliquístico (SOP).
b. Sobrepeso (IMC mayor a 25 kg/m2) antes del embarazo.
c. Partos con productos macrosómicos de más de 4 kilos.
d. Trastorno del metabolismo de los carbohidratos.

Pregunta 254
¿En qué etapa de la sífilis se observa la perforación del paladar blando?
a. Sífilis primaria.
b. Sífilis prenatal (congénita).
c. Sífilis terciaria.
d. Sífilis secundaria.

Pregunta 255
En relación con el secreto profesional, todos los enunciados son verdaderos. EXCEPTO:
a. No hay obligación de los profesionales de la salud de establecimientos públicos o privados,
de denunciar un presunto delito cuando el conocimiento de los hechos esté amparado por el
secreto profesional.
b. Existen casos donde la ruptura del secreto profesional o el denunciar, no causan daño al
paciente, sino lo protegen de situaciones de vulnerabilidad e indefensión.
c. El secreto médico es la categoría que se asigna a toda información que es revelada por un/a
usuario/a al profesional que le brinda la atención de salud.
d. Bajo cualquier circunstancia es obligación de los profesionales de la salud de
establecimientos públicos o privados presentar una denuncia, al conocer la comisión de un
presunto delito.
Pregunta 256
William tiene 44 años, está desempleado, divorciado, consume bebidas alcohólicas todos los
fines de semana, se siente triste, duerme mucho, ha bajado de peso, no puede concentrarse,
siente que piensa lento, por dos ocasiones se ha cortado las venas y ha decidido lanzarse de
un puente. Escoja el plan de tratamiento a seguir en el manejo clínico de William.
a. Manejo y apoyo familiar para la guía conductual e inicio de ansiolíticos.
b. Manejo por consulta externa de psiquiatría semanal.
c. Hospitalización en el servicio de psiquiatría e inicio de psicofármacos.
d. Se respeta la autonomía del paciente porque es una voluntad anticipada.

Pregunta 257
Son factores de riesgo para la demencia tipo Alzheimer. EXCEPTO:
a. Edad avanzada.
b. Tabaquismo.
c. Sexo masculino.
d. Hipertensión arterial.

258
La hipertensión portal es una de las primeras manifestaciones de una patología cirrótica,
pero también otras patologías prehepáticas e intrahepáticas pueden causarla. En el
tratamiento profiláctico para evitar hemorragia variceal indica el uso de los
betabloqueadores no selectivos (BBNS). Señale cual es un efecto secundario al uso de estos
medicamentos:
a. Bloqueo auriculo ventricular de II grado.
b. Impotencia.
c. Insuficiencia cardiaca.
d. Enfermedad pulmonar obstructiva grave

Pregunta 259
La lactancia materna está contraindicada en las siguientes situaciones. EXCEPTO:
a. VIH y carga viral alta.
b. Fenilcetonuria.
c. Mastitis Aguda.
d. Galactosemia.

Pregunta 260
El medicamento que disminuye la hemorragia en el sangrado digestivo alto del paciente
cirrótico
es:
a. Ceftriaxona.
b. Eritromicina.
c. Octeotride.
d. Prostaciclina.

Pregunta 261
Los siguientes son cambios hemodinámicos fisiológicos del embarazo. EXCEPTO:
a. Reducción de la resistencia vascular sistémica.
b. Disminución del volumen sanguíneo.
c. Aumento del gasto cardiaco.
d. Reducción de la presión arterial.

Pregunta 262
El hallazgo clínico más habitual en los casos de neumonía infantil es:
a. La presencia de estridor.
b. La presencia de taquipnea.
c. La presencia de auscultación pulmonar anormal.
d. La presencia de tos.

Pregunta 263
Un recién nacido, producto de la primera gesta de una madre de 23 años que tuvo una labor
de parto de 12 horas, ruptura de membranas intraparto, líquido amniótico claro y grumoso.
El RN. Nació de parto normal en un servicio hospitalario. Al nacer presentó un Peso de: 2.9
kg; Talla: 49 cm; PC: 34.5 cm; APGAR: 7/9; A las 24 horas de edad presenta ictericia en cara y
tronco. La bilirrubina indirecta es de 7 mg/dl. La bilirrubina directa es de 0.9 mg/dl. Se
encuentra activo, tiene buena reactividad al manejo. Toma seno materno exclusivo, con
succión adecuada, presentó regurgitación por dos ocasiones. El grupo sanguíneo de su
madre es A+; el RN es 0+ y Coombs directo es negativo. Seleccione el diagnóstico, así como la
decisión terapéutica correcta para su ictericia:
a. Septicemia; hemocultivo y otros exámenes relativos + antibioticoterapia.
b. Hepatitis neonatal; solicitar estudios correlativos y vigilar bilirrubinas.
c. Ictericia por incompatibilidad ABO; fototerapia y realizar control de bilirrubinas.
d. Ictericia fisiológica; observar evolución y realizar control de bilirrubinas.

Pregunta 264
Paciente de 21 años con embarazo de 28 semanas por FUM, refiere fiebre, dolor
costovertebral, náuseas y vómito de 48 horas de evolución. El EMO presenta piuria y
hematuria. ¿Qué diagnóstico tiene la paciente y qué estudio debe solicitar?
a. Bacteriuria asintomática, solicitar hemocultivo.
b. Cistitis aguda, solicitar ultrasonografía renal.
c. Pielonefritis, solicitar cultivo y antibiograma de orina.
d. Litiasis renal, solicitar urografía con contraste.

Pregunta 265
Paciente que presenta un accidente de tránsito, es llevado a la sala de emergencia con
disnea, dolor torácico, taquicardia y ausencia de ruidos respiratorios en hemitórax derecho,
usted sospecha de un neumotórax a tensión. ¿Cuál es el sitio en el que se debe colocar la
aguja de descompresión en esta patología?
a. Sexto espacio intercostal, línea axilar anterior del hemitórax afectado.
b. Segundo espacio intercostal, línea medio clavicular del hemitórax afectado.
c. Cuarto espacio intercostal, línea medio clavicular del hemitórax afectado.
d. Segundo espacio intercostal, línea axilar anterior del hemitórax afectado.

Pregunta 266
¿Cómo se denomina la maniobra de valoración de displasia del desarrollo de cadera, en la
cual la cabeza femoral dislocada se reduce al acetábulo?
a) Galeazzi.
b) Barlow.
c) Ortolani.
d) Allis.

Pregunta 267
Según las curvas de crecimiento aceptadas en el Ecuador, para un varón de 7 años de edad,
los percentiles 10, 50 y 95, para su antropometría, son los siguientes: Peso: 16, 22 y 27 kg;
Talla: 108,119 y 128 cm; IMC: 13.7; 15 y 18 kg/m2; Un niño de 7 años de edad presenta los
siguientes parámetros antropométricos: Peso: 30 kg; Talla: 118 cm; IMC: 21.5 kg/m2. De
acuerdo con estos datos, seleccione el diagnóstico nutricional:
a. Obesidad grado I.
b. Obesidad Grado II.
c. Obesidad mórbida.
d. Sobrepeso.

Pregunta 268
En la clasificación TNM para cáncer de próstata un estadio T3b se refiere a:
a. El tumor es un hallazgo histológico fortuito en más del 5 % del tejido resecado.
b. Extensión extracapsular incluida la afectación microscópica del cuello de la vejiga.
c. El tumor afecta a más de la mitad de un lóbulo, pero no a los dos lóbulos.
d. El tumor invade una o ambas vesículas seminales.

Pregunta 269
Una mujer de 16 años llega a la consulta por un chequeo rutinario. La anamnesis revela
amenorrea primaria, ella dice que ha aumentado la cantidad de ejercicio que realiza.
Durante el examen físico se encuentra un IMC de 21, desarrollo mamario Tanner estadio 1 y
FSH elevado. La prueba de BhCG fue negativa. ¿Cuál es el siguiente paso?
a. Realizar un cariotipo.
b. Medir niveles de TSH.
c. Pedirle que disminuya las horas de ejercicio que realiza y que regrese en 2 meses.
d. Prescribir anticonceptivos orales.

Pregunta 270
En un paciente con cetoacidosis diabética, a más de la corrección de la hiperglucemia, se
deberán realizar las siguientes acciones. EXCEPTO:
a. Medición y reposición de potasio.
b. Medición y reposición de fosfato.
c. Medición y reposición de bicarbonato.
d. Medición y reposición de calcio.

Pregunta 271
José tiene 39 años, se encuentra desempleado, su hijo nació con una malformación de la
oreja y es motivo de discusiones en su hogar, ha estado triste por más de 4 meses; ya no sale
con sus amigos que era algo que le gustaba mucho, acude a la consulta solicitando ayuda
porque ha pensado que sería mejor si estuviera muerto, sabe que su pensamiento es malo,
pero esa idea es recurrente. Seleccione la severidad de riesgo suicida en este caso:
a. Moderado.
b. Grave.
c. Leve .
d. Extremo.

Pregunta 272
Paciente de 54 años con fractura distal de radio izquierdo extraarticular tratada mediante
inmovilización con yeso. A los 2 días presente: aumento de perímetro del antebrazo
Izquierdo, parestesias, edema, palidez distal e intenso dolor que aumenta con el
estiramiento pasivo de la mano izquierda ¿Cuál es el diagnostico?
a. Trombosis venosa profunda.
b. Síndrome compartimental agudo
c. Desplazamiento de la fractura.
d. Celulitis del antebrazo izquierdo

Pregunta 273
Seleccione el tratamiento inicial de asma en niños mayores de 5 años:
a. Agonista Beta 2 de larga duración, como el salmeterol. inhalado
b. Agonista Beta 2 de corta duración como el salbutamol. inhalado.
c. Corticoide, inhalado como fluticasona.
d. Antagonista de receptores de los leucotrienos, como montetukast

Pregunta 274
Paciente de 4 años, presenta desde hace 24 horas evacuaciones liquidas, abundantes y
frecuentes; tiene vómitos continuos e intolerancia oral: presenta signo del pliegue bien
definido y la piel regresa muy lentamente a su lugar. Indique el diagnóstico y tratamiento
correcto para este paciente:
a. Deshidratación leve. Administrar por vía oral sales de rehidratación oral 1000 ml en las
primeras 4 horas.
b. Deshidratación grave. Administrar por vía intravenosa. Ringer lactato 50 ml/kg en la primera
hora. luego 20mI/kg en la 2da y 3ra horas.
c. Deshidratación grave. Administrar por via oral: sales de rehidratación oral 1000 ml en las
primeras 4 horas.
d. Deshidratación moderada. Administrar, via intravenosa: Ringer lactato 50 ml/kg en la
primera hora, luego 20 ml/kg en le 2da y 3ra hora.

Pregunta 275
La letra D del acrónimo ABCDE de la revisión primaria en trauma corresponde a:
a. Valoración neurológica.
b. Valoración circulatoria.
c. Valoración de vía aérea.
d. Valoración respiratoria.

Pregunta 276
Seleccione la patología que presenta Incremento en el análisis cuantitativo de Beta-hCG
(human chorionic gonadotropint valorado cada 48 horas en una embarazada:
a. Embarazo molar.
b. Retención fetal.
e. Amenaza de aborto.
d. Desprendimiento placentario

Presencia 277
En el servicio de emergencia tiene un paciente pediátrico con una crisis convulsiva tonico
clónica generalizada. Determine el orden de la prioridad, de las siguientes acciones, para su
tratamiento inmediato:
1. Canalizar vía venosa.
2. Abrir-Posicionar la vía aérea y evaluar el estado cardiovascular.
3. Administrar diazepam intravenoso 0.3 mg/kg peso.
4. Administrar oxígeno.
a) 1,2,3,4.
b )4.1,3.2.
c) 3,1.2,4.
D) 2 4.1.3.

Pregunta 278
La característica más común de la disfunción autonómica de la diabetes:
a. Bradicardia en reposo
b. Anhidrosis de Piernas
c. Aumenta del Vaciamiento Gástrico.
d. Hiperglicemia.

Pregunta 279
13. Señale qué medicamento de los usados en el tratamiento del acné (enfocándose en la
seguridad de su uso durante el embarazo le corresponde a Categoria X:
a. Cotrimoxazol.
b. lsotrenitoina.
c. Doxiciclina.
d. Adapaleno.

Pregunta 280
Son causas de delirium. EXCEPTO
a. Intoxicación por sustancias.
b. Exposición a eventos traumáticos.
c. Abstinencia de sustancias.
d. Trastornos metabólicos

Pregunta 281
¿Cuántos grados hay que rotar en un parto céfalo-vaginal de una variedad de presentación
Occipito Ilíaca Izquierda Anterior, en la rotación interna que coloca a la presentación en
directa?
a. 135 grados en contra de las manecillas del reloj.
b. 90 grados en sentido de las manecillas del reloj.
c. 25 grados en sentido de las manecillas del reloj.
d. 45 grados en contra de las manecillas del reloj.

Pregunta 282
Julia es una lactante de 18 meses da edad que acude a su consulta porque hace 3 dias
presenta abundante rinorrea, fiebre, malestar general, hiporexia, tos esporádica y
congestión nasal que no le permite dormir bien en la noche porque le falta el aire. Al
examen físico presenta temperatura de 38°C. congestión nasal orofaringe ligeramente
eritematosa, el examen de oído y pulmones esta normal. Escoja la opción de tratamiento
adecuada para la paciente.
a. Acetaminofen, cetirizina mas peudoefedrina.
b. Acetaminofén peudoefedrina y miel de abeja.
c. Acetaminofén y lavado nasal con Solución salina.
d. Vitamina C, dextrometorfano y loratadina.

Pregunta 283
El uso de lsotretinoína produce:
a. Aumenta la producción de sebo.
b. Sequedad extrema de la piel y queilitis.
c. Aumenta la queratinización de la piel.
d. Inflamación de la piel

Pregunta 284
¿Qué medicamento se utiliza en el tratamiento del acné vulgar leve?
a. Tetraciclina.
b. Clobetasol.
c. Isotretinoína.
d. Metronidazol.

Pregunta 285
La angustia patológica so distingue por tina de las siguientes manifestaciones:
a. Ser fantasmagórica, representación imaginara de un conflicto real y consciente.
b. Ser estereotipada, repetitiva, sin estar afincada en eI carácter del sujeto.
c. Es adaptativa para el sujeto especialmente en las crisis de angustia.
d. Ser anacrónica, pues lleva a revivir situaciones pasadas

Pregunta 286
Las siguientes son indicaciones de referencia al oftalmólogo ante un caso de conjuntivitis
aguda. EXCEPTO:
a. Pérdida de la visión.
b. Inmunosupresión.
c. Epifora o lagrimeo continuo.
d. Episodios recurrentes.

Pregunta 287
¿Qué signo al examen físico caracteriza a la anorexia nerviosa?
a. Signo de Russell.
b. Borborigmos.
c. Lanugo en la cara.
d. Fractura y pérdida dental

Pregunta 288
El diagnóstico de muerte por cese de las funciones cardiorrespiratorias de forma irreversible,
se establece en la comprobación de forma inequívoca de:
a. Ausencia de latido cardiaco y de respiración espontánea durante un periodo no inferior a 5
minutos, tras adecuado período de maniobras de reanimación.
b. En casos de temperatura corporal inferior a 28°C se deberá recalentar el cuerpo antes de
poder establecer la irreversibilidad de la parada.
c. La irreversibilidad del cese cardiorrespiratorio se deberá constatar tras maniobras de
reanimación cardiopulmonar avanzada no menor a 8 minutos.
d. Ausencia trazado electrocardiográfico, de ausencia de respiración espontánea, ambas cosas
durante un periodo no inferior a 10 minutos.

Pregunta 289
¿Cuál es el diagnóstico más probable, de un paciente de 70 años que presenta marcado
malestar general y ortostatismo, que inició hace 1 hora? Al examen fisico se objetiva pulso
regular, FC: 31 pulsaciones por minuto y PA: 80/50 mm Hg.
a. Síndrome de Wolf-Parkinson-White
b. Bradicardia sinusal
c. Enfermedad del nodo sinusal.
d. Bloqueo AV completo

Pregunta 290
Paciente sin cardiopatía, refiere episodios anteriores al presente de palpitaciones, ansiedad,
dolor torácico, disnea, que ceden espontáneamente. Presenta el siguiente trazo
electrocardiográfico indique el algoritmo de tratamiento a seguir.
a. Verapamilo + amiodarona + adenosina.
b. Maniobras vagales + adenosina+ procainamida.
c. Verapamilo + maniobras vagales + procainamida.
d. Adenosina + amiodarona + maniobras vagales.

Pregunta 291
Seleccione la opción correcta para el tratamiento de un paciente de 4 años. Con 17 kg, en
cuyo coproparasitario se evidencia quistes y trofozoitos de Giardia lamblia y huevos de
Ascaris lumbricoides:
a. Metronidazol de 125 mg / 5 ml en suspensión, 3.5 ml TID por 7 días: al día siguiente
Albendazol 400 mg PO una sola loma
b. Metromdazol 3.5 ml de suspensión de 250 mg / 5 ml. TID durante 7 días; al día siguiente
albendazol 400 mg PO una vez
c. Albendazol 400 mg PO el 1er día, al día siguiente metronidazol de 125 mg / 5 mi en 3.5 ml
TID por 7 días
d. albendazol 400 mg PO el 1er día. al día siguiente metronidazol de 250 mg / 5 ml en
suspensión: 4 mi TID por 5 días.

Pregunta 292
Angelita tiene 8 años, hace una semana presento alza térmica de 38 grados centígrados,
dolor de cabeza, rinorrea y malestar general. Hace 24 horas presentó eritema facial que
tienen el aspecto de 'mejillas abofeteadas.' y ahora, el exantema se extendió al tronco y
extremidades corno eritema macular difuso respetando palmas y plantas. Escoja la opción
correcta con relación al diagnóstico y al tratamiento.
a. Parvovirus 519 - control por consulta externa.
b. Rubivirus - antipiréticos y analgésicos.
c. Paramyxoviridae - medidas de soporte.
d. Herpes 6 - tratamiento sintomático.

Pregunta 293
Se recomienda el uso rutinario de corticoides para el tratamiento del síndrome de HELLP.
b. Se recomienda el intercambio de plasma o plasmaféresis para el tratamiento de síndrome
de HELLP, en especial durante los primeros 4 días posparto
c. Según le clasificación de Mississipi el síndrome de Hellp, es de Clase I, cuando las plaquetas
son menores a 50.000. AST o ALT mayor a 70 Ul/L. y LDL mayor a 600 Ul/L.
d. Cuando el conteo de plaquetas se encuentra comprende. entre 20000 y 49000 pL con
síndrome de HELLP, no se debe transfundir plaquetas previo o la cesárea.

Pregunta 294
Paciente de 25 años con antecedente de parto reciente presenta palpitaciones, disnea, dolor
pleurítico, hemoptisis: al examen físico se evidencia fiebre de 38° C. cianosis, taquipnea,
taquicardia, estertores pulmonares. En exámenes solicitados: gasometría arterial normal,
EKG reporta taquicardia sinusal, Rx de tórax reporta atelectasias basales con elevación del
hemidiafragrna, ensanchamiento de los hilios pulmonares bilateralmente, Infiltrado alveolar
basal borde inferior convexo (joroba de Hampton), afilamiento de imágenes vasculares.
¿Cuál es el tratamiento anticoagulante que debe administrar al ingreso, antes de referir a
UCI?
a. Heparina.
b. Estreptocinasa.
c. Warfarina.
d. Clopidrogrel.

Pregunta 295
Relacione los conceptos con sus cáracteristicas:
CONCEPTOS CARACTERISTICAS
1) ÉTICA a) Conjunto de normas que vienen del Interior.
2) MORAL b) Vinculada estrechamente a las costumbres.
c) Tiene una base de reflexión personal.
d) Hace referencia a las normas en una sociedad.
a. 1ab. 2cd
b. 1bc. 2ad
c. 1 ad. 2bc
d. 1 ac. 2bd

Pregunta 295
Un varón de 23 años de edad, previamente sano, acude a su consulta por presentar desde hace
24 horas fiebre, escalofrío, malestar general, dolor perineal, disuria y polaquiuria. En el examen
físico presión arterial: 12/80 mmHg, frecuencia cardiaca: 100 lpm, frecuencia respiratoria: 18
rpm, temperatura bucal 39,5 0C, ruidos cardio-respiratorios y examen abdominal son normales.
Al tacto rectal siente intenso dolor interno y el examen de orina revela abundantes leucocitos.
Junto con medidas de hidratación y analgesia. ¿Qué tipo de antibiótico es el recomendado como
primera línea de tratamiento?
a. Amoxicilina clavulanato.
b. Aminoglicósidos.
c. Fluoroquinolonas.
d. Cefalosporinas

Pregunta 296
Varón de 37 años que llega a la emergencia del hospital al poco tiempo de un accidente de
tránsito. El examen físico constata: presión arterial 88/42 mmHg, taquicardia de 118 lpm,
abdomen ligeramente distendido y doloroso. Usted sospecha de trauma abdominal cerrado,
¿Cuál de los siguientes procedimientos efectuaría primero?
a. Punción abdominal diagnóstica
b. Radiografía abdominal simple
c. Ecografía al pie de la cama
d. Tomografía computarizada

Pregunta 297
Paciente masculino de 25 años y 50 kg de peso, sufre quemadura que compromete las dos
extremidades superiores y la extremidad inferior izquierda. Las lesiones son: de apariencia roja
con formación de ampollas. Seguún el porcentaje de superficie corporal quemada y su
profundidad, indique el volumen estimado en el manejo de líquidos.
a. Lactato Ringer 3.600 cc en las primeras 8 horas y 3.600 cc en las siguientes 16 horas
b. Lactato Ringer 3.600 cc administrado inmediatamente y 3.600 cc en las siguientes 16 horas
c. Dextrosa en solución salina 3.600 cc en las primeras 8 horas y 3.600 cc en las siguientes 16
horas
d. Lactato Ringer 3.600 cc hasta conseguir diuresis y 3.600 cc en las siguientes 16 horas

Pregunta 298
Mujer de 45 años, acude a emergencias por presentar desde hace dos días fiebre de 38,5 0C y
dolor en hipocondrio derecho, al inicio tipo cólico, ahora constante, que se irradia a región
torácica derecha posterior. Tambien refiere nausea y vomito. La causa aparente es la ingesta
de comida abundante que incluyó carne de chancho y papa con mayonesa. El examen físico
revela un índice de masa corporal de 28,5, temperatura de 38,7 0C, frecuencia cardiaca 110
por minuto. Ruidos hidroaéreos presentes pero disminuidos. Hemiabdomen superior doloroso,
especialmente bajo borde costal derecho a la inspiración profunda. La mejor combinación de
examenes que usted podría utilizar para precisar el diagnóstico, es?
a. Resonancia magnética y colangio-pancreatografia retrograda endoscópica.
b. Determinación de amilasa y lipasa pancreática
c. Endoscopia digestiva alta y biopsia gastroduodenal.
d. Solicitar ecografía abdominal y biometría hemática

Pregunta 299
Varon de 22 años, se presenta a un examen medico de rutina. En la exploración física el
medico encuentra disminución del tamaño testicular izquierdo y aspecto de gusanos en la
bolsa escrotal. El paciente ante este hallazgo comenta que ocasionalmente ha sentido dolor
testicular, sensación de masa y aumento de peso escrotal, pero que estos síntomas duran
pocos días y no les ha prestado mayor atención. Cuál es el diagnostico mas probable?
a. Orquitis
b. Torsion testicular
c. Varicocele
d. Hernia inguinal

Pregunta 300
Varon de 58 años de edad, acude por presentar un dolor a nivel inguinal izquierdo que según
relata se ha presentado hace pocos días luego de un esfuerzo físico realizado en la mudanza de
casa. Su medico diagnostica una hernia inguinal Grado II, y recomienda coreccion quirúrgica, la
cual se efectua en la semana siguiente. En exámenes de laboratorio efectuados previa a la
cirugía, destaca glicemia en ayunas, de 118 mg. El acto quirurgico se lleva a cabo sin
complicaciones. En el post operatorio inmediato. ¿Que conducta seria la mas adecuada
respecto al empleo de antibiótico?
a. Continuar con el antibiotico indicado en el preoperatorio.
b. Cefazolina 1 gramo por vía intravenosa, dosis única
c. Amoxicilina mas acido clavulánico, 2 g I.V cada 8 horas
d. No esta indicada profilaxis antibiótica post operatoria.

Pregunta 301
Un varón de 23 años de edad, previamente sano, acude a su consulta por presentar desde hace
24 horas fiebre, escalofrío, malestar general, dolor perineal, disuria y polaquiuria. En el examen
físico presión arterial: 12/80 mmHg, frecuencia cardiaca: 100 lpm, frecuencia respiratoria: 18
rpm, temperatura bucal 39,5 0C, ruidos cardio-respiratorios y examen abdominal son normales.
Al tacto rectal siente intenso dolor interno y el examen de orina revela abundantes leucocitos.
Junto con medidas de hidratación y analgesia. ¿Qué tipo de antibiótico es el recomendado como
primera línea de tratamiento?
a. Amoxicilina clavulanato.
b. Aminoglicósidos.
c. Fluoroquinolonas.
d. Cefalosporinas

Pregunta 302
Varón de 37 años que llega a la emergencia del hospital al poco tiempo de un accidente de
tránsito. El examen físico constata: presión arterial 88/42 mmHg, taquicardia de 118 lpm,
abdomen ligeramente distendido y doloroso. Usted sospecha de trauma abdominal cerrado,
¿Cuál de los siguientes procedimientos efectuaría primero?
a. Punción abdominal diagnóstica
b. Radiografía abdominal simple
c. Ecografía al pie de la cama
d. Tomografía computarizada

Pregunta 303
Paciente masculino de 25 años y 50 kg de peso, sufre quemadura que compromete las dos
extremidades superiores y la extremidad inferior izquierda. Las lesiones son: de apariencia roja
con formación de ampollas. Seguún el porcentaje de superficie corporal quemada y su
profundidad, indique el volumen estimado en el manejo de líquidos.
a. Lactato Ringer 3.600 cc en las primeras 8 horas y 3.600 cc en las siguientes 16 horas
b. Lactato Ringer 3.600 cc administrado inmediatamente y 3.600 cc en las siguientes 16
horas
c. Dextrosa en solución salina 3.600 cc en las primeras 8 horas y 3.600 cc en las
siguientes 16 horas
d. Lactato Ringer 3.600 cc hasta conseguir diuresis y 3.600 cc en las siguientes 16 horas

Pregunta 304
Mujer de 45 años, acude a emergencias por presentar desde hace dos días fiebre de 38,5 0C y
dolor en hipocondrio derecho, al inicio tipo cólico, ahora constante, que se irradia a región
torácica derecha posterior. Tambien refiere nausea y vomito. La causa aparente es la ingesta
de comida abundante que incluyó carne de chancho y papa con mayonesa. El examen físico
revela un índice de masa corporal de 28,5, temperatura de 38,7 0C, frecuencia cardiaca 110
por minuto. Ruidos hidroaéreos presentes pero disminuidos. Hemiabdomen superior doloroso,
especialmente bajo borde costal derecho a la inspiración profunda. La mejor combinación de
examenes que usted podría utilizar para precisar el diagnóstico, es?
a. Resonancia magnética y colangio-pancreatografia retrograda endoscópica.
b. Determinación de amilasa y lipasa pancreática
c. Endoscopia digestiva alta y biopsia gastroduodenal.
d. Solicitar ecografía abdominal y biometría hemática

Pregunta 305
Varon de 22 años, se presenta a un examen medico de rutina. En la exploración física el
medico encuentra disminución del tamaño testicular izquierdo y aspecto de gusanos en la
bolsa escrotal. El paciente ante este hallazgo comenta que ocasionalmente ha sentido dolor
testicular, sensación de masa y aumento de peso escrotal, pero que estos síntomas duran
pocos días y no les ha prestado mayor atención. Cuál es el diagnostico mas probable?
a. Orquitis
b. Torsion testicular
c. Varicocele
d. Hernia inguinal

Pregunta 306
Varon de 58 años de edad, acude por presentar un dolor a nivel inguinal izquierdo que según
relata se ha presentado hace pocos días luego de un esfuerzo físico realizado en la mudanza de
casa. Su medico diagnostica una hernia inguinal Grado II, y recomienda coreccion quirúrgica, la
cual se efectua en la semana siguiente. En exámenes de laboratorio efectuados previa a la
cirugía, destaca glicemia en ayunas, de 118 mg. El acto quirurgico se lleva a cabo sin
complicaciones. En el post operatorio inmediato. ¿Que conducta seria la mas adecuada
respecto al empleo de antibiótico?

a. Continuar con el antibiotico indicado en el preoperatorio.


b. Cefazolina 1 gramo por vía intravenosa, dosis única
c. Amoxicilina mas acido clavulánico, 2 g I.V cada 8 horas
d. No esta indicada profilaxis antibiótica post operatoria.

Pregunta 307
Una mujer de 46 años, presenta desde hace 26 horas un cuadro de dolor epigástrico irradiado
hacia la espalda, se acompaña de nausea y vomito. Los niveles de amilasa y lipasa están diez
veces elevado sobre su valor normal. En este contexto, de entre los siguientes, ¿Cuál NO es un
marcador de severidad?
a. Niveles séricos de lipasa
b. Niveles séricos de creatinina
c. Niveles séricos de lactodeshidrogenasa
d. Niveles séricos de aminotranferasas

Pregunta 308
Un varón de 23 años es llevado a emergencia luego de haber sufrido un accidente
automovilistico de alto impacto, se queja de dolor abdominal severo irradiado a escapula
izquierda. Al examen físico: presión arterial 70/50 mmHg, saturación de oxigeno 88% con el
ambiente, dolor abdominal generalizado con signo de rebote en cuadrante superior izquierdo
de abdomen, dolor a la palpación en región subcostal izquierda. La ecografía evidencia
hematoma rodeando el bazo y fluido peritoneal. De las siguientes medidas terapéuticas,
señale la mas importante a implementarse en este paciente.
a. Administración de cristaloides
b. Esplenectomía de emergencia
c. Esplenorrafia si es posible
d. Administración de oxigeno

Pregunta 309
Un varón de 56 años de edad, sin antecedentes relevantes, tiene el diagnostico de hipertrofia
prostática benigna y recibe tamsulosina. ¿Cual de los siguientes efectos adversos aparece con
relativa frecuencia cuando se realiza este fármaco?
a. Caída del cabello
b. Hipotension ortostática
c. Insuficiencia hepática
d. Hipoglicemia nocturna
310
Un varón de 19 años, presenta dolor abdominal de 12 horas de evolución. Al momento el dolor
se localiza en fosa iliaca derecha, se acompaña de nauseas, escaso vomito y no ha realizado la
deposicion. Al examen físico el abdomen es suave, doloroso a la palpación superficial en fosa
iliaca derecha, con signo de rebote positivo. La apendicectomia inmediata revelo apendicitis
grado II (2). Recibe ampicilina sulbactam por vía intravenosa, dosis única transoperatoria.
¿Cual es la mejor conducta a seguir?
a. Asociación de gentamicina y metronidazol por 7 dias.
b. No requiere otro tratamiento antibiotico adicional
c. Metronidazol intravenoso cada 8 horas por 3 dias
d. Asociación de ciprofloxacina y clindamicina por 7 dias.

Pregunta 311
Un varón de 66 años de edad, sin antecedentes relevantes presenta molestias urinarias desde
hace varios meses, que consisten en sentir que su vejiga no se vacia completamente luego de
terminar la micccion, en ocasiones requiere volver a orinar luego de una o dos horas después
de haberlo hecho, la micción a veces en entre cortada y el chorro es débil. El examen físico
general es normal para su edad y el examen de orina no tiene alteraciones. Hasta obtener una
evaluación especializada, en caso de requerir alivio de síntomas. ¿Cual medicamento
prescribiría usted?
a. Finasteride
b. Biperidem
c. Amitriptilina
d. Dimenhidrinato

Pregunta 312
Maritza de 50 años, presenta desde hace doce horas dolor tipo colico en cuadrante superior
derecho, 30 minutos después de ingesta de alimentos grasos. El dolor se irradia hacia la región
subescapular derecha. Se acompaña de síntomas neurovegetativos. Desde hace 8 horas se
nota coloracion amarillenta de ojos y sensación de alza térmica y escalofríos. Al examen físico:
frecuencia respiratoria 16 por minuto, frecuencia cardiaca 90 por minuto, tensión arterial
120/80 mmHg, temperatura 39,3 0C. Facies: algica, sudorosa, fría, con palidez generalizada.
Ojos: escleras ictericas. Abdomen: doloroso a la palpación superficial y profunda en epigastrio
e hipocondrio derecho. En exámenes: leucocitos 18000 /uL, neutrófilos 80%, bilirrubina total 8
mg/dl, bilirrubina directa 6,5 mg/dl, bilirrubina indirecta 1,5 mg/dl, AST 248 UI, ALT 285 UI,
fosfatasa alcalina 280 UI/dl, amilasa de 440 U/dl. El reporte de ecosonografia indica: vesicula
con multiples litos en su interior y de diferente tamaño, paredes de 3 mm de espesor;
colédoco de 16 mm de diamentro y no se visualiza cálculos. De acuerdo al cuadro clínico,
indique el diagnostico y tratamiento mas acertado según lo expuesto en los siguientes
literales:
a. Terapia hidroeletrolitica, antibióticos, colecistectomía por colecistitis crónica
reagudizada.
b. Terapia hidroelectrolítica, antibióticos, colecistectomía con exploración de vias biliares
por colecistitis aguda mas colangitis aguda.
c. Antibióticos y colecistectomía con exploración de vias biliares por coledocolitiasis mas
colangitis aguda.
d. Terapia hidroelectrolítica, antibióticos, colecistectomía con exploración de vias biliares
por coledocolitiasis mas colangitis aguda.
Pregunta 313
Un paciente de 65 años de edad, luego de un examen de fondo de ojo presento cefalea, dolor
ocular unilateral, alteración de la percepción visual y de colores en ese mismo ojo y el tono
ocular esta elevado. ¿Cual de los siguientes fármacos esta indicado?
a. Acetazolamida
b. Sulfato de zinc
c. Fenilefrina
d. Omatropina

Pregunta 314
Una paciente de 78 años de edad fue sometida a un examen de fondo de ojo, con dilatación
pupilar pocas horas mas tarde una hija le llama a usted porque la paciente se queja de intenso
dolor en su ojo izquierdo, tiene visión borrosa con el mismo ojo, nausea y vomito. Al
examinarla el ojo derecho esta normal en forma y función. En el ojo izquierdo hay
enrojecimiento intenso, disminución severa de la agudeza visual y visión solo de luz y la
palpación refleja ojo duro y doloroso. El resto del examen físico no presenta particularidades.
Hasta que el oftalmologo se haga cargo del paciente, ¿Cual de los siguientes fármacos debe
utilizarse como parte imprescindible del tratamiento emergente de primera elección?
a. Acetazolamina 250 mg, una tableta cada 6 horas.
b. Amitriptilina 25 mg una tableta cada 12 horas.
c. Carbamazepina 100 mg 1 tableta cada 12 horas.
d. Difenhidramina 25 mg una tableta cada 8 horas.

Pregunta 315
Un varón de 15 años de edad, hace pocas horas mientras se hallaba participando en una
carrera de motocicleta en la modalidad de enduro, sufrió un accidente, a consecuencia del cual
presenta intenso dolor e impotencia funcional en su pierna izquierda. En el examen físico
encontramos una herida de aproximadamente 1 cm, localizada en el tercio medio de la pierna
sin contusión local, con poca cantidad de tierra en los alrededores, por la cual se advierte un
sangrado moderado. ¿Cual es el diagnostico mas probale?
a. Herida de partes blandas
b. Fractura cerrada de tibia o perone.
c. Fractura expuesta de tibia o perone, tipo I
d. Fractura expuesta de tibia o perone tipo II.

Pregunta 316
¿Cuál de las siguientes NO es un factor de riesgo para pancreatitis aguda?
a. Dabetes mellitus tipo II
b. Consumo exagerado de alcohol
c. Hipertrigliceridemia familiar
d. Colangiopancreatografia endoscópica

Pregunta 317
Un varón de 66 años de raza blanca, acude a control medico de salud y se documenta un
antigeno prostatico (PSA) de 9,6 ng/ml. En la anamnesis refiere polaquiuria durante el dia, y
dismuncion del chorro urinario. El medico urólogo luego del tacto rectal decide realizar una
biopsia de próstata y un estudio radiologico oseo en todo el cuerpo. Las siguientes
aseveraciones respecto al cancer prostatico son frecuentes, excepto.
a. La mayoría de los canceres localizados son clinicamente silentes y se detectan por
vigilancia de antigeno prostatico (PSA)
b. La metastasis son frecuentes en los canceres prostáticos avanzados, y la localización
ósea es habitual en los mismos.
c. Las radiaciones ionizantes son causa de lesiones en los genes y contribuyen en la
fisiopatología del cancer de próstata.
d. El antigeno prostatico (PSA) puede estar elevado en enfermedades no neoplásicas,
como la hiperplasia nodular benigna.

Pregunta 318
Mujer de 63 años acude a emergencia por presentar dolor abdominal agudo tipo colico e
incapacidad para eliminar flatos. La paciente sufre de constipación crónica y toma laxantes
diariamente. Al examen físico: frecuencia cardiaca 97/ min, presión arterial 140/95, fiebre 38,1
0C, deshidratada, abdomen distendido, doloroso y timpánico, ausencia de peristaltismo
intestinal. El medico a cargo se plantea el diagnostico de vólvulo de colon. ¿Cuál de los
siguientes hallazgos es más sugerente de necesidad de intervención quirúrgica?
a. Leucocitosis con neutrofilia en biometría hemática.
b. Aumento de creatinina serica en examen de sangre
c. Proctorragia con eliminación de mucosa colonica oscura.
d. Distensión masiva de colon sigmoide en radiografia.

Pregunta 319
Varón de 17 años de edad, consulta por presentar 4 horas antes traumatismo directo en ojo
izquierdo, ocasionado por un objeto contundente. En el examen físico encontramos diplopía,
limitación de los movimientos oculares, equimosis, enoftalmos y dolor intenso. Cual es el
diagnostico?
a. Fractura orbitaria
b. Laceración conjuntival
c. Contusion periocular
d. Lesión de parpado

Pregunta 320
Carolina de 24 años de edad presenta fiebre de 39 0C, tres días después de haber sido
sometida a cesárea, su hijo nació en buenas condiciones, sin signos de enfermedad y se
encuentra en buen estado de salud. Se palpa el fondo uterino blando y doloroso y se aprecian
loquios de mal olor. Al tacto vaginal: dolor en cuello que se incrementa con los movimientos.
Se solicitan biometría hemática que reporta: Hemoglobina 11 g/dl, leucocitos 12.500
(segmentados 75%, linfocitos 25%); análisis de orina normal. ¿Cual de los siguientes esquemas
antibióticos es el mas adecuado para el tratamiento de la endometritis?
a. Gentamicina – Ampicilina
b. Ampicilina- sulbactan
c. Clindamicina – gentamicina
d. Metronidazol- clindamicina.

Pregunta 321
Mujer de 26 años, ingresada a sala de alto riesgo obstétrico, por embarazo a termino
complicado con preeclampsia severa y manejada con sulfato de magnesio a dosis de
impregnación de 6 g y de mantenimiento al 2 g /hora. Desde su ingreso recibe hidratación
intravenosa, una dosis de misoprostol intravaginal de 25 mcg y una de hidralazina de 5 mg I.V.
al momento presenta sensación de calor y eritema en cara y tronco, leve somnolencia,
hiporreflexia patelar y disminución de la variabilidad de la frecuencia cardiaca fetal. ¿Cuál de
los medicamentos recibidos explicaría las manifestaciones actuales de la paciente?
a. Misoprostol
b. Sulfato de magnesio
c. Hidralazina
d. Fluidos intravenosos

Pregunta 322
Paciencia de 39 años con antecedentes de hipertensión arterial crónica controlada con
enalapril 20 mg diarios, hasta la fecha actual. La presión arterial atual se ha mantenido entre
130/90 mmHg y 140/90 mmHg. Cursa su primer embarazo de 10 semanas con evolución
normal. Acude a su primer control prenatal. Al examen físico se constata presión arterial de
150/98 mmHg. Cuál es su recomendación en relación a la terapia antihipertensiva?

a. Cambiar el fármaco
b. Aumentar la dosis
c. Disminuir la dosis
d. Mantener la terapia

Pregunta 323
Paciente de 21 años, con tumoración de 2 cm, móvil, liso, sin fijación a piel, localizado en
cuadrante superexterno de la mama izquierda, no se palpa adenopatías. Luego de los
exámentes complementarios se le propone su extirpación quirúrgica. Indique el diagnóstico
más probable:

a. Papiloma intraductal
b. Cáncer in situ
c. Necrosis grasa
d. Fibroadenoma

Pregunta 324
Mujer de 35 años, cursa con embarazo de 32 semanas, acude al servicio de emergencia por
presentar desde hace dos horas, sangrado vaginal rojo de moderada a abundante cantidad,
que apareció luego de relación sexual. Antecedentes Gineco-obstétricos: gesta 5, partos 2,
cesáreas 0, abortos 4, sin control prenatal. Al examen físico: signos vitales normales, fondo
uterino corresponde a amenorrea, feto único, transverso, frecuencia cardíaca fetal 150 lpm,
tono uterino normal. ¿Cuál de los siguientes procedimientos diagnósticos estaría
contraindicado?

a. Monitoreo fetal electrónico


b. Ecografía obstétrica
c. Tacto vaginal.
d. Resonancia magnética

Pregunta 325
Mujer de 35 años con embarazo de 28 semanas, acude al control prenatal, preocupada por
antecedentes de dos partos prematuros a las 30 y 32 semanas, saldados con la muerte de uno
de ellos a los pocos días del nacimiento por membrana hialina. La historia clínica y el examen
físico son normales y compatibles con la amenorrea. ¿Cuál de los siguientes exámenes o
evaluaciones puede predecir mejor la posibilidad de parto antes de las 35 semanas?

a. Longitud cervical
b. Calcular el puntaje de Bishop
c. Monitorizar actividad uterina
d. Dosificación de fibronectina fetal

Pregunta 326
Mujer de 31 años de edad, embarazada de 34 semanas de gestación (G:1, P:0) Acude a la
consulta el centro de salud por presentar dolor de cabeza, visión borrosa y dolor a nivel
epigástrico. Niega antecedentes patológicos personales. Durante el examen físico tenemos:
P:82/min; PA:190/125 mmHg; FR: 18/min; y T:37 oC. Dolor a la palpación superficial y
profunda a nivel epigástrico, no se palpan masas, ni órganomegalia. Extremidades inferiores:
edema, reflejos tendinosos 3/4 bilateral, resto del examen físico normal. Se ordena exámenes
de laboratorio: HB:12 g/dL; Hct: 36%; leucocitos: 12000/mm3; BUN:24mg/dL; Cr:1.3mm/dL;
Glucosa: 99mg/dL; HEMO: sangre negativo; glucosa negativo; leucocitos 1-2/campo; proteínas
+++. Elija la conducta adecuada con base en el caso:

a. Tiene eclampsia y debe ser manejada con sulfato de magnesio


b. Tiene hipertensión inducida por el embarazo y debe ser internada
c. Tiene pre-eclampsia y manejamos con reposo y control semanal
d. Tiene pre-eclampsia y referimos de inmediato para el manejo integral parto

Pregunta 327
Al realizar las maniobras de Leopold, en una paciente con embarazo a término y labor de
parto. ¿En qué maniobra usted determina la posición y situación fetal?

a. Primera
b. Segunda
c. Tercera
d. Cuarta

Pregunta 328
Mujer de 19 años, soltera, acude a la consulta por presentar aumento de secreción vaginal
desde hace 24 horas, asociada a leve sintomatología urinaria. Antecedentes gineco-obstétrica:
Gesta 0. FUM: 20 días antes. Relación sexual hace 6 días con pareja ocasional. El examen
especular revela vulva y vagina normales, cérvix posterior con ligero eritema, por cuyo orificio
externo se exterioriza secreción amarillo purulenta. El tacto vaginal evidencia útero de tamaño
normal, doloroso a la movilización y anexos no palpables pero sensibles. ¿Cual de las
siguientes opciones es el diagnóstico más probale?

a. Candidiasis
b. Gonorrea
c. Vaginosis
d. Tricomoniasis

Pregunta 329
De acuerdo a las estadísticas de la Organización Mundial de la Salud, ¿Cuál es la principal causa
de muerte materna?
a. Preeclampsia
b. Infección
c. Sangrado
d. Multiparidad

Pregunta 330
La inducto conducción de la labor de parto con oxitocina es uno de los procedimientos as
frecuentes en obstetricia. ¿cuál de los siguientes conceptos NO es correcto en relación al uso
de oxitocina?
a. La respuesta del utero es variable
b. Produce efecto de taquifilaxis
c. La vida media es de 30 minutos
d. Tiene efecto antidiurético

Pregunta 331
Usted atiende un parto a termino sin complicaciones. Se produce alumbramiento completo. Al
controlar una hora después se constata hemorragia vaginal profusa, y útero blando, no
contraído. A demás de indicar medidas para corrección de hipovolemia: ¿Cuál es el orden
adecuado de procedimientos médicos para controlar la hemorragia vaginal?
1. Compresión bimanual del útero
2. Taponamiento uterino
3. Masaje uterino
4. Agentes uterotónicos
5. Intervención quirúrgica
a. 2, 1, 4, 5
b. 3, 2, 4, 5
c. 3, 4, 1, 5
d. 4, 3, 1, 5

Pregunta 332
Mujer de 33 años, gestas 5, para 4, aborto 1, acude a la consulta a los 4 meses después del
ultimo parto en el que preento hemorragia importante, con signo de choque hemodinámico,
que preciso transfusión de sangre. Consultapor presentar sintomatología variada, en la que se
incluye: perdida de peso, caída importate de vello pubico, disminución significativa de
volumen de las mamas, decaimiento, piel seca. Le preocupa también, que a pesar de que no
pudo dar de lactar, hasta el momento no ha menstruado. ¿Cuál de los siguientes síndromes es
el mas probable?
a. Rokitansky
b. Kallman
c. Sheehan
d. Addison

Pregunta 333
Paciente de 25 años con dos partos normales en antecedentes, FUM: 10 dias antes. Acude a la
consulte para control periódico y realización de citología cervi vaginal. ¿Cuáles son las mejores
opciones para el sitio anatomico de la toma y el insumo usado para ello?
a. Exocervix, espátula de Ayre
b. Endocérvix, cepillo citológico
c. Unión escamocilindrica, espátula y cepillo
d. Fondos de saco vaginales, espátula de Ayre
Pregunta 334
Paciente de 51 años, acude a la consulta por presentar durante los últimos dos años episodios
de sensación de calor intenso seguidos de sudoración profusa, localizados en cara cuello y
torax. Son de corta duración (minutos). Se asocian a palpitaciones y angustia. Tienen una
frecuencia de diez al dia y en la noche le impiden el sueño. También refiere sequedad vaginal
que provoca dispareunia. La fecha de la última menstruación es hace 7 meses y sus ciclos
menstruales han sido irregulares desde hace 18 meses. Si usted sospecha de síndrome pre
menopausico, en el que hay insuficiencia ovárica. ¿Cuál de los siguientes exámenes
evidenciaría mejor el feedback negativo que se produce?
a. Hormona foliculoestimulante
b. Determinación de estrógeno
c. Determinación de progesterona
d. Hormona liberadora de gonadotropina

Pregunta 335
Mujer De 21 años, soltera, acude a consulta por presentar secreción vaginal de ligero mal olor
desde hace 24 horas, asociada a leve sintomatología urinaria. Fecha de última menstruación:
15 días antes. Relación sexual hace 1 semana, con pareja ocasional. El examen especular revela
vulva con escurrimiento por comisura posterior de consistencia líquida, amarillento, en
moderada cantidad. En vagina se constata leucorrea amarillo-verdosa, de ligero mal olor, con
pocas burbujas. Cervix posterior eritematoso y punteado rojo. El tacto vaginal es normal. El
examen de fresco y gram de secreción vaginal reporta test de KOH positivo levemente,
polimorfonuleares aumentados y presencia de microorganismos móviles. ¿Cuál es el mejor
tratamiento para esta paciente?

a. Metronidazol 500 mg VO BID por 7 días.


b. Isoconazol 600 mg intravaginal, una dosis.
c. Penicilina benzatínica 2.400.000 UI, una dosis
d. Fluonazol 150 mg VO por semana, por tres semanas

Pregunta 336
Paciente femenina de 20 años, acude por secreción vaginal verdosa de ligero mal olor de tres
días de evolución, teniendo como causa aparente relación sexual hace 9 días. Se asocia a
sintomatología urinaria. La secreción descrita se observa en el orificio externo del cuello
uterino. El examen de fresco y Gram demuestra la presencia de diplococos Gram negativos
intra y extracelulares. ¿Cuál de los siguientes esquemas terapéuticos es el de primera
elección?

a. Ceftriaxona más azitromicina


b. Penicilina procaínica más gentamicina
c. Azitromicina en monodosis
d. Fluoroquinolona en monodosis

Pregunta 337
Mujer de 35 años, gestas 5, paras 4, abortos 1. Consulta por presentar sintomatología variada,
en la que se incluye: pérdida de peso, amenorrea secundaria de meses de duración, después
del último parto, a pesar de que no pudo dar de lactar, caída del vello púbico, disminución del
volumen mamario, decaimiento y piel seca. Tiene como antecedente personal hemorragias
post parto, que en el último fue exagerada, con signos de choque hipovolémico, el cual precisó
transfusión de sangre. ¿Cuál de las siguientes glándulas cree usted que es responsable del
cuadro clínico?
a. Ovarios
b. Tiroides
c. Hipófisis
d. Suprarrenales

Pregunta 338
Una mujer de 70 años, diabética e hipertensa, requiere bajo criterio médico una cirugía para
colocar prótesis de cadera derecha. La señora está lúcida y su salud se encuentra estable. Los
hijos se reúnen y deciden no autorizar la intervención quirúrgica porque han leído en internet
y familiares médicos les han informado que la convalescencia es dolorosa y dificultosa. ¿Cuál
de las siguientes debe ser la conducta médica recomendada?

a. No operar a la señora conforme a lo decidido en forma consensuada por los hijos


b. Convocar al comité de ética para decidir si se opera o no a la paciente
c. El galeno decide en base a evidencia médica si opera o no a la paciente
d. Conversar con la paciente y operar en base a la decisión de ella

Pregunta 339
En los ensayos clínicos y en general en la investigación que involucra a seres humanos, debe
haber un balance razonable entre los posibles riesgos que pueden derivar para los sujetos que
participan en la investigación y los beneficios que se espera obtener de ella. ¿Cuál principio de
la Bioética sustenta este enunciado?

a. Beneficencia
b. Información
c. Autonomía
d. No maleficencia

Pregunta 340
Mujer de 36 años de edad, casada, tiene un hijo de 8 años. Antecedentes de cuatro episodios
depresivos, el primero de los cuales empezó a los 20 años de edad. El episodio actual lleva ya
una evolución de 1 año y pese a que su psiquiatra ha ensayado algunos esquemas
terapéuticos, la sintomatología no cede. Ante esta situación el especialista sugiere
internamiento en una clínica para realizar un tratamiento de “activación cerebral” sin más
explicaciones- Luego de haber recibido durante una semana estos tratamientos, si bien el
estado de ánimo ha mejorado, la paciente se encuentra angustiada porque no recuerda los
acontecimientos de esta última semana. El médico explica que es un efecto secundario del
tratamiento y que “ya se le pasará”. El esposo de la paciente reclama por el hecho de no haber
sido informado de los efectos secundarios que podía ocasionar el tratamiento propuesto.
¿Cuál de los principios de bioética no respetó el médico?

a. Principio de justicia
b. Principio de beneficencia
c. Principio de no maleficencia
d. Principio de autonomía

Pregunta 341
Una mujer de 35 años de edad, sedentaria, tiene un índice de masa corporal de 35, se siente
deprimida y solicita consejo médico. Al examen físico tiene la presión arterial en 140/90 mm
Hg. En esta paciente. ¿Cuál de las siguientes medidas terapéuticas individualizadas, para el
control de la obesidad, tiene MENOS impacto sobre la reducción de morbilidad y mortalidad a
largo plazo?

a. Ejercicio diario, moderado, 30 minutos consecutivos


b. Reducción de las calorías diarias en la alimentación
c. Control de peso con fármacos como la fluoxetina
d. Incremento del consumo diario de frutas y legumbres

Pregunta 342
Una paciente de 66 años de edad acude por dolor pleurítico. Luego de verificar la existencia de
derrame pleural izquierdo, usted decide realizar un estudio del líquido. Los exámenes de
sangre de la paciente indican proteínas totales de: 6 g/dL, LDH: 200 UI/L. Los exámenes del
líquido reportan proteínas de 1,5 g/dL, LDH:100 UI/L. Como posibles causas de derrame
pleural, de acuerdo a los resultados obtenidos. ¿Cuál de los siguientes diagnósticos es MENOS
probable?

a. Insuficiencia cardíaca congestiva


b. Embolismo pulmonar
c. Nefrosis
d. Cirrosis

Pregunta 343
Durante el transcurso de una enfermedad crónica un paciente debe recibir fármacos que son
excretados por via renal. Clásicamente se utiliza la ecuación de Cockroft-Gault para calcular el
aclaramiento (clearance) de creatinina. Ese método permite obtener rápidamente un valor
aproximado del aclaramiento de creatinina. ¿Cuál de los siguientes parámetros NO toma en
cuenta la ecuación mencionada?

a. Edad del paciente


b. Creatinina sérica
c. Peso del paciente
d. Volumen urinario

Pregunta 344
Una paciente de 29 años de edad, sin antecedentes patológicos de importancia. Desde hace 5
días presenta fiebre, malestar genera, decaimiento, tos no productiva y dolor en punta de
costado del lado derecho. Acude a consulta y al examen físico tiene presión arterial de 120/80
mm Hg, frecuencia cardíaca de 98 latidos por minuto, frecuencia respiratoria de 22
respiraciones por minuto, temperatura bucal de 39oC. Tiene incremento del frémito y
broncofonía en región subescapular derecha, el hemitórax izquierdo es normal, ruidos
cardiacos normales, abdomen normal. En la radiografía de tórax se aprecia imagen radio opaca
con broncograma aéreo limitado al lóbulo inferior derecho. La oximetría de pulso de 91% al
aire ambiente. En la biometría hemática se encuentran 8.500 leucocitos/mm3 con 70% de
segmentados y 30% de linfocitos. La conducta más apropiada será:

a. Reposo en casa, administrar amoxicilina clavulanato vía oral y reevaluar a las 48 horas
b. Reposo en casa, administrar tratamiento sintomático vía oral y reevaluar a las 48 horas
c. Hospitalizar, tomar cultivos y no administrar antibióticos hasta tener resultados
d. Hospitalizar, administrar un beta lactámico y un macrólido, vía intravenosa
Pregunta 345
Un paciente joven, sin antecedentes de importancia. Vice en zona tropical selvática y acude
por presentar una lesión en la piel de la mejilla, provocada por una picadura de insecto. Desde
hace un par de meses, la pápula tomó un color violeta, tiende a crecer y a ulcerarse, Al examen
físico tiene una lesión papular indurada, color rojo vinoso, con halo eritematoso, de unos 6
centímetros de diámetro, cubierta de una capa escamosa y en el centro tiene una costra
redondeada de 4 centímetros de diámetro, no es dolorosa. El resultado del frotis encuentra
macrófagos y escasos microorganismos en formas amastigotes. ¡cual de los siguientes
fármacos está indicado como tratamiento de primera línea para esta lesión?

a. Ceftriaxona
b. Estreptomicina
c. Antimonio pentavalente
d. Ivermectina

Pregunta 346
Para inspeccionar y palpar la glándula tiroides, usted debe conocer la ubicación correcta de la
misma, con respecto a los cartílagos tiroides y cricoides. ¿Cuál es la posición normal de esta
glándula?

a. Junto al cartílago tiroides


b. Junto al cartílago cricoides
c. Entre los dos cartílagos
d. Bajo el cartílago cricoides

Pregunta 347
Una paciente de 42 años de edad, sin antecedentes clínicos relevantes. Acude por presentar
dolor abdominal difuso, recurrente, de moderada intensidad, concambios en los hábitos
defecatorios, que dura aproximadamente 1 semana. Esta sintomatología se ha repetido
durante los últimos 3 meses. Según los criterios internacionalmente aceptados (de Roma II).
¿Cuál de los siguientes síntomas o signos concomitantes confirmaría el diagnóstico de colon
irritable?

a. Vómito post prandial


b. Alivio luego de la defecación
c. Meteorismo
d. Diarrea nocturna

Pregunta 348
Se trata de un paciente de 35 años de edad, previamente sano, que se dedica a la venta
informal de quesos artesanales. Acude porque desde hace un par de semanas tiene sensación
de fiebre, decaimiento, sudoración nocturna, dolor corporal y algo de pérdida de peso. No
tiene tos. Al examen físico lo relevante es la temperatura de 37,9 grados centígrados. En los
exámenes e laboratorio las muestras e BAAR en orina son negativas, tiene leucocitos de
7000/mm3, con 50% de linfocitos y 45% de neutrófilos de características normales.
Aglutinaciones para brucella son positivas 1:640. Hasta que los resultados de los cultivos estén
listos. ¡que antibióticos debería prescribir?

a. Dicloxacilina y amikacina
b. Ampicilina y claritromicina
c. Estreptomicina y doxiciclina
d. Vancomicina y gentamicina

Pregunta 349
En un paciente con derrame pleural usted debe evaluar las características del liquido pleural y
de la sangre. Las proteínas en sangre son normales. La lactodeshidrogenasa (LDH) en sangre
está elevada; las proteínas y la LDH del líquido pleural tienen un valor mayor al 60% de los
valores en sangre. Con estos valores, de entre las siguientes causas de derrame pleural. ¿Cuál
es la más probable?

a. Insuficiencia cardíaca
b. Mixedema
c. Hipoalbuminemia
d. Neumonía

Pregunta 350
Paciente de 24 años de edad, sin antecedentes de importancia que sufre un traumatismo de
miembros inferiores con fractura de fémur, tibia y peroné del mismo lado. Al evaluarlo en la
emergencia, usted debe decidir si presenta signos de choque “shock”. En este contexto, de
entre los siguientes signos clínicos del estado de choque. ¿Cuál es el que con mayor frecuencia
ocurre más temprano?

a. Hipotensión
b. Confusión
c. Taquicardia
d. Bradicardia

Pregunta 351
Paciente varón de 70 años, portador de fibrilación auricular crónica, acude al servicio de
emergencias. Ha presentado de manera súbita, la perdida e movimientos y de la sensibilidad
en las extremidades superior e inferior del lado izquierdo y desviación de la mirada hacia el
lado derecho. ¿Cuál de las siguientes arterias está probablemente involucrada?

a. Cerebral posterior
b. Cerebral media
c. Cerebral anterior
d. Vértebro basilar

Pregunta 352
Mujer de 42 años de edad, dese hace 2 dias luego de ingerir alimentos, presenta dolor
epigástrico moderado, que se ha intensificado de manera paulatina, es continuo y se irradia
hacia la espalda, se acompaña de esporádico vómito de contenido alimentario que no calma el
dolor. Al examen físico, al tomar la presión arterial, se provoca la contractura espástica de la
musculatura extensora y aductora de los dedos de la mano correspondiente. La amilasa se
encuentra elevada 7 veces y la lipasa 3 veces, con respecto a sus valores normales. El reporte
ecográfico indica páncreas incrementado de volumen e hipoecogénico. ¿Qué hallazgo de
laboratorio podría explicar la contractura espástica descrita?

a. Hipopotasemia
b. Hiponatremia
c. Hipocalcemia
d. Hipocoremia
Pregunta 353
Paciente de 40 años refiere fiebre, sudoración nocturna, pérdida de peso y disnea de un
tiempo evolución de 2 meses. A la exploración física TA: 120/80, FC:80 lpm, Sat O2: 96%, Tº:
38 ºC, dolor torácico en hemitórax anterior derecho, murmullo vesicular disminuido en base
derecha. En la radiografía de tórax se observa radiopacidad homogénea unilateral derecha,
borramiento del ángulo costofrénico derecho de borde cóncavo sin presencia de
condensaciones. ¿Cuál es el diagnóstico?
a. Neumonía bacteriana.
b. Tuberculosis pleural.
c. Asma persistente.
d. Absceso pulmonar.

Pregunta 354
Paciente asintomático, que al examen físico presenta un aumento del tiroides grado II, sin
nódulos palpables. Los exámenes reportan: TSH y T3 normales, T4 total baja, colesterol
alto, anticuerpos contra TPO negativos, la gammagrafía tiroidea pone de manifiesto un
aumento en la captación. Señale el diagnóstico:
a. Bocio multinodular tóxico.
b. Bocio difuso no tóxico.
c. Tiroiditis de Hashimoto.
d. Bocio multinodular no tóxico.

Pregunta 355
¿Cuál de las siguientes causas de hipoglucemia se relaciona en el paciente no enfermo?
a. Medicamentos.
b. Deficiencia hormonal.
c. Tumores no pancreáticos.
d. Hiperinsulinismo endógeno.

Pregunta 356
Señale la patología que ocasiona anemia megaloblástica:
a. Síndrome de Mallory-Weiss.
b. Tumor esofágico.
c. Hernia del hiato.
d. Gastrectomía total.

Pregunta 357
Señale el procedimiento representativo que se relaciona con cirugía sucia (clase IV).
a. Traumatismo abdominal penetrante.
b. Enterotomía durante la obstrucción intestinal.
c. Diverticulitis perforada.
d. Reparación de hernia.

Pregunta 358
Paciente de 9 años con síndrome de Laron que ha sido seleccionado para entrar en un
ensayo clínico multicéntrico para probar un nuevo IGF-1 recombinante humano. El niño y
sus padres firman el consentimiento informado. ¿Qué principio bioético está ejerciendo el
paciente?
a. Privacidad.
b. Beneficencia.
c. Autonomía.
d. Justicia.

Pregunta 359
En pacientes con trauma, en la valoración primaria, en la sección de circulación.
Identifique la primera lesión más importante que pone en riesgo la vida:
a. Hemotórax masivo.
b. Hemoperitoneo masivo.
c. Fracturas pélvicas.
d. Taponamiento cardíaco.

Pregunta 360
¿Cuáles son complicaciones de la hemorragia subaracnoidea?
a. Vasoespasmo, encefalopatía hipertensiva e isquemia secundaria.
b. Hematoma intraparenquimatoso, hipernatremia y ceguera.
c. Hidrocefalia, vasoespasmo y meningitis.
d. Hidrocefalia, vasoespasmo y resangrado.

Pregunta 361
Señale la dosificación ideal (objetivo terapéutico) de HbA1c en pacientes ancianos con
múltiples enfermedades crónicas y dificultad para las actividades de la vida diaria:
a. HbA1c 7.0%
b. HbA1c 10.0%.
c. HbA1c 6.5 %.
d. HbA1c 8.0 %.

Pregunta 362
Paciente de 4 años de edad, quien está consciente, afebril, presenta estridor inspiratorio
leve y tos seca "perruna". Indique el diagnóstico correcto:
a. Neumonía.
b. Bronquiolitis.
c. Crup moderado.
d. Amigdalitis aguda.

Pregunta 363
Paciente de 30 años, refiere disuria y secreción uretral purulenta de 3 días de evolución.
En el examen físico se objetiva abundante secreción uretral purulenta e inflamación del
meato uretral. La tinción de Gram muestra abundantes leucocitos polimorfonucleares y
diplococos Gram-negativos. Señale el tratamiento empírico de elección para este paciente:
a. Ceftriaxona 250 mg intramuscular + Azitromicina 1 g vía oral, ambas en dosis única.
b. Ceftriaxona 250 mg intramuscular una sola dosis + Metronidazol 500 mg vía oral por
14 días.
c. Ceftriaxona 250 mg intramuscular en una sola dosis + Clindamicina 300 mg vía oral por
7 días.
d. Gentamicina 160 mg intramuscular una sola dosis + Azitromicina 500 mg vía oral por
tres días.

Pregunta 364
Paciente femenino de 18 años, sin antecedentes patológicos de importancia,
sexualmente activa acude a la consulta por presentar secreción vaginal, al examen físico se
detecta cervicitis con secreción vaginal abundante que es inodora, no irritante y de color
amarillento. ¿Cuál es el diagnóstico y tratamiento médico adecuado?
a. Infección por hongos - Azitromicina.
b. Vaginosis bacteriana - Clotrimazol.
c. Infección por herpes - Valaciclovir.
d. Infección por gonorrea - Doxiciclina.

Pregunta 365
Paciente de 65 años que acude a consulta médica de primer nivel, sexo masculino,
fumador, no tiene antecedentes de diabetes mellitus; se le toma la presión arterial sistólica
marca 160 mm Hg. Utilizando la siguiente tabla de estimación de riesgo cardiovascular
(OMS/ISH), determine cual es el RCV que presenta este paciente:

a ) RCV de 30% a menor a 40%


b ) RCV mayor o igual a 40%
c ) RCV de 20% a menor a 30%
d ) RCV de 10% a menor a 20%

Pregunta 366
Las siguientes son anomalías de pared abdominal, EXCEPTO:
a ) Onfalocele.
b ) Hematoma de la vaina del recto.
c ) Hernia umbilical.
d ) Quiste epiploico.

Pregunta 367
La forma de prescripción de dosis de impregnación de Sulfato de Magnesio, como
tratamiento para prevenir la eclampsia es:
a ) 4 gramos de Sulfato de Magnesio, vía intravenosa a pasar en 20 minutos.
b ) 8 gramos de Sulfato de Magnesio, vía intravenosa a pasar en 10 minutos.
c ) 6 gramos de Sulfato de Magnesio, vía intravenosa a pasar en 20 minutos.
d ) 3 gramos de Sulfato de Magnesio, vía intravenosa a pasar en 20 minutos.
Pregunta 368
¿Cuál es el manejo inicial de un varón con retención aguda de orina causada por
hematuria con coágulos?
a ) Colocar una sonda suprapúbica.
b ) Colocar un catéter de tres vías.
c ) Colocar una sonda vesical Foley.
d ) Realizar cistoscopia con dilatador uretral.

Pregunta 369
Señale en qué caso se respeta la objeción de conciencia del médico:
a ) Suicidio médicamente asistido.
b ) Petición expresa y reiterada del paciente.
c ) Padecimientos graves insoportables.
d ) Enfermedad clínica irreversible e incurable.

Pregunta 370
Señale un signo que represente déficit agudo de volumen extracelular:
a ) Hiperazoemia.
b ) Aumento de peso.
c ) Edema intestinal.
d ) Soplo.

Pregunta 371
Señale la complicación transfusional retardada:
a ) Anafilaxia.
b ) Hepatitis.
c ) Hemólisis.
d ) Edema pulmonar.

Pregunta 372
Miriam dio a luz hace 3 horas por cesárea, usted le recomienda que alimente al recién
nacido con leche materna, pero la madre le dice que no tiene leche. Escoja la opción
correcta en relación a la lactancia post cesárea.
a ) Le pide a Miriam que mezcle leche infantilizada con leche del banco y espera a que la
producción propia aumente.
b ) Le da leche de fórmula para complementar hasta que Miriam produzca más leche materna.
c ) Le solicita acudir a un banco de leche humana para que el niño reciba leche materna.
d ) Le tranquiliza y le dice que ahora tiene calostro, verificando la posición correcta para dar de
lactar.

Pregunta 373
Las siguientes son factores relacionados al desarrollo de dermatitis seborreica.
EXCEPTO:
a ) Infección por virus de inmunodeficiencia humana (VIH).
b ) Enfermedades neurológicas como Parkinson.
c ) Infección por Staphylococcus aureus.
d ) Infección por Malassezia furfur.

Pregunta 374
Señale cuál es la manifestación clínica de la escabiosis reincidente:
a ) Máculas azuladas.
b ) Sarpullido generalizado.
c ) Sarpullido local.
d ) Pseudoliendres.

Pregunta 375
En el segundo trimestre del embarazo. ¿Qué valores de hemoglobina y hematocrito se
consideran anemia?
a ) Hb mayor a 11 g/dL y Hcto 33%
b ) Hb mayor a 12 g/dL y Hcto 36%
c ) Hb menor a 10.5 g/dL y Hcto 32%
d ) Hb menor a 10 g/dL y Hcto 30%

Pregunta 376
Señale el objetivo terapéutico para control de hiperlipidemia que debe tener un paciente
adulto con diabetes:
a ) Triglicéridos < 150mg/dl (1.7mmol/L).
b ) Lípidos LDL < 100mg/dL (5.6 mmol/L)
c ) Triglicéridos < 100mg/dL (2.6 mmol/L)
d ) Lípidos HDL < 180mg/dL (10mmol/L)

Pregunta 377
¿Qué se observa en la obesidad central?
a ) Triglicéridos disminuidos.
b ) Colesterol HDL elevado.
c ) Resistencia a la insulina.
d ) Colesterol LDL disminuido.

Pregunta 378
Los siguientes enunciados corresponden a los factores causales de la distanasia
(obstinación terapéutica), EXCEPTO:
a ) Deseo del paciente crítico por eliminar su propia existencia o de sus representantes legales,
para evitar la prolongación del sufrimiento del paciente.
b ) Adopción de medidas terapéuticas que contemplan más los aspectos científicos de la
enfermedad que al enfermo afectado de un proceso irreversible.
c ) Desconocer el derecho del paciente o de sus representantes legales, a rechazar el inicio o
continuación de tratamientos médicos que prolonguen el sufrimiento del enfermo crítico o su
agonía.
d ) Convencimiento de algunos médicos de que la vida biológica es un bien por el que luchar, al
margen de consideraciones sobre la calidad de esa vida.

Pregunta 379
En un sangrado uterino anormal de tipo anatómico, usamos la ecografía con infusión de
solución salina como método diagnóstico. ¿En qué caso está contraindicado su uso?
a. Infección pélvica.
b. Pólipos endometriales.
c. Diabetes.
d. Leiomiomas submucosos.

Pregunta 380
Indique, ¿Cuándo un lavado peritoneal diagnóstico (LPD) es positivo?
a. Extracción de 5 cc de sangre inmediatamente a la punción.
b. Un lavado negativo excluye cualquier tipo de lesión abdominal.
c. Retorno mayor del 30% del líquido infundido en la cavidad abdominal.
d. Más de 100.000 eritrocitos por mm3.

Pregunta 381
A su consulta general acude un paciente de 2 meses, sexo masculino es llevado para
vacunación, el lactante ha sido vacunado con la BCG y con Hepatitis B pediátrica al
nacimiento. Indique el esquema que corresponde a la edad actual del lactante
a. Vacuna antipoliomielítica, Pentavalente, Influenza, Neumococo pediátrica.
b. Vacuna antipoliomielítica, Pentavalente, Rotavirus, Influenza.
c. Vacuna antipoliomielítica, Pentavalente, Fiebre Amarilla, Neumococo pediátrica
d. Vacuna antipoliomielítica, Pentavalente, Rotavirus, Neumococo pediátrica.

Pregunta 382
Con respecto a la urticaria en pacientes pediátricos es correcto que:
a. La urticaria puede ser causada por varios factores, siendo lo más común los estímulos
físicos.
b. En la urticaria crónica las lesiones duran más de dos días a la semana durante 3
semanas.
c. El habón característico es una roncha elevada, pruriginosa y eritematosa que palidece
a la
presión.
d. En la urticaria aguda las lesiones suelen durar más de 24 horas, con tendencia a
confluir.

Pregunta 383
Paciente de 7 años procedente de Machala. Desde hace 10 días presenta fiebre cada 72
horas, cuantificada en 40º C. Se acompaña de sudoración, cefalea, diarrea y palidez cutáneo
mucosa. Estos episodios alternan con periodos de buen estado general. Indique el
diagnóstico más probable:
a. Malaria.
b. Fiebre amarilla.
c. Fiebre tifoidea.
d. Dengue.

Pregunta 384
Paciente masculino de 33 años con trabajo extenuante que le ocasiona estrés, con
antecedentes de enfermedad ulcerosa péptica, presenta vómitos de contenido no bilioso y
alcalosis metabólica intensa con hipopotasemia. Indique a que complicación de la
enfermedad ulcerosa péptica se refiere el enunciado.
a. a ) Úlcera péptica perforada.
b. b ) Obstrucción pilórica.
c. c ) Gastritis crónica.
d. d ) Síndrome de Zollinger - Ellison.

Pregunta 385
El enunciado correcto en referencia al parto pretérmino es:
a. a ) El incremento del pH vaginal previene el riesgo de parto pretérmino.
b. b ) La proteína C reactiva en frotis vaginal previene el riesgo de parto pretérmino.
c. c ) El uso de progesterona aumenta la incidencia de parto pretérmino.
d. d ) Las modificaciones cervicales valoradas por ecografía, son un marcador predictor.

Pregunta 386
Complete el siguiente enunciado: Los diámetros oblicuos del plano de entrada de la
pelvis se extienden desde las…………… a ………………….
a. Sincondrosis sacroiliacas, eminencia iliopectínea ipsilateral.
b. Sindesmosis sacroiliacas, eminencia iliopectínea contralateral.
c. Sincondrosis sacroiliacas, borde lateral de sínfisis del pubis.
d. Sindesmosis sacroiliacas, espina ciática contralateral.

Pregunta 387
Paciente de 68 años, diagnosticado con diabetes se hospitaliza por descompensación
de valores glicémicos. Señale la opción que refleja un modelo autonomista en la relación
clínica de este paciente.
a. El paciente menciona que dejará que el médico tome las decisiones por él.
b. Establecer con el paciente los objetivos que se quieren lograr y que él quiera lograr.
c. Paciente debe seguir estrictamente indicaciones médicas aún cuando tiene opiniones
contrarias.
d. El médico toma todas las decisiones porque el paciente no sabe nada de medicina.

Pregunta 388
Un niño de 6 años de edad, que vive en una región tropical del país, acude a consulta
porque en los últimos 10 días presenta tos y esputo sanguinolento. Además, dolor
abdominal ocasional, así como deposiciones diarreicas abundantes y con moco, que se
producen por temporadas, desde hace un año. Ocasionalmente presenta también vómitos.
En este año, por tres ocasiones presentó lesiones irregulares, longilíneas, a manera de
trayectos indurados, eritematosos y pruriginosos en tórax, abdomen, glúteos y miembros
inferiores. Estas lesiones desaparecieron en uno o dos días. Su peso está en el percentil 5
para la edad. Su talla en el percentil 10. La biometría hemática reporta únicamente una
marcada eosinofilia, con 600 eosinófilos/ml de sangre. Se realiza un coproparasitario que
reporta la presencia de larvas parasitarias. Seleccionar el literal que contenga el parásito
más probable de acuerdo con el cuadro clínico de este niño, y el medicamento con el que
debería realizarse el tratamiento:
a. Enterobius vermicularis; mebendazol, 100 mg dos veces diarias, por 7 días.
b. Ascaris lumbricoides; albendazol, 200mg en una sola toma, por una sola vez.
c. Strongyloides stercoralis; ivermectina, una tableta de 3 mg en ayunas, una vez.
d. Giardia lamblia; metronidazol, 250 mg, 1 c/día, luego de una comida, 10 días.

Pregunta 389
Seleccione los criterios que corresponden a un caso de dengue sin signos de alarma:
a. IgM positiva.
b. Exantema.
c. Sangrado de mucosas.
d. Leucopenia.
e. Elevación de AST mayor o igual a 1 000.
f. Prueba de torniquete positivo.
a. b, d, e.
b. a, d, f.
c. c, d, f.
d. b, d, f.

Pregunta 390
A su consulta general acude un paciente de 15 meses de edad masculino con su madre
para vacunación, el paciente ha cumplido con el esquema de vacunación vigente. ¿Cuál
vacuna es la siguiente a administrarse según el Ministerio de Salud Pública del Ecuador?
a. Vacuna contra virus del papiloma humano (HPV).
b. Vacuna de difteria y tétano (Td).
c. Vacuna de poliomielitis (OPV).
d. Vacuna contra varicela.

Pregunta 391
¿Cuál es el principal pilar de tratamiento en la infección por Cólera?
a. Acortar el período de la excreción bacteriana.
b. Restablecer los fluidos y electrolitos.
c. Administrar antibióticos como terapéutica inicial.
d. Asegurar una nutrición adecuada.

Pregunta 392
Una niña de 3 años de edad, pesa 15 kg. Desde hace 12 horas presenta una erupción de
aparición abdominal, con vesículas y pápulas eritematosas, con intenso prurito, que se han
extendido hacia el tórax. No ha tenido fiebre. Se mantiene activa y come bien. T°: 37,5 °C;
FC: 70 x 1´; FR: 24 x 1´; Al examen físico las máculas, pápulas y vesículas se han extendido
al tronco anterior y posterior. Seleccione el diagnóstico y tratamiento correctos:
a. Escarlatina, administrar penicilina benzatínica 600 000 UI IM por una sola vez.
b. Varicela, administrar aciclovir suspensión de 200 mg / 5 ml, 7.5 ml cada 6 horas
durante 5 días.
c. Roséola o exantema súbito, paracetamol en gotas de 100 mg / ml, 2 ml cada 6 horas, 3
días.
d. Rubéola, administrar paracetamol en jarabe de 150 mg / 5 ml, 4 ml cada 6 horas, 3
días.

Pregunta 393
En la pelvis materna los límites del estrecho superior incluyen:
a. a ) Atrás limitado por el promontorio del sacro y a los lados por la cresta pectínea.
b. b ) A los lados limitado por las alas del sacro y adelante por la cresta pectínea.
c. c ) A los lados limitado por promontorio del sacro y adelante por la sínfisis del pubis.
d. d ) Atrás limitado por la cresta pectínea y adelante por la sínfisis del pubis.

Pregunta 394
Todos son cambios en el estilo de vida de un paciente con Diabetes Mellitus tipo 2.
EXCEPTO:
a. Se sugiere una disminución del 7 % de grasas en la dieta.
b. Reducción de peso entre el 5 y 10 % en pacientes con DM2 con sobrepeso u obesidad.
c. Fraccionar el total de la alimentación habitual del día en 5 o 6 porciones.
d. Mantener una dieta fraccionada con tres ingestas diarias.

Pregunta 395
RN de 30 semanas de edad gestacional. Membranas íntegras. Nace por cesárea. Líquido
amniótico claro, sin grumos. APGAR 6-8. Al nacer presenta taquipnea, quejido, cianosis y
retracciones costales y subcostales. Se le suministra oxígeno y una hora después presenta
pausas de apnea. Seleccione el diagnóstico correcto:
a. Síndrome de aspiración de meconio.
b. Taquipnea transitoria del recién nacido.
c. Neumonía neonatal.
d. Enfermedad de membrana hialina.
Pregunta 396
Paciente masculino de un año de edad acude a la consulta porque sus padres indican
que en ocasiones no sienten el testículo del lado derecho, sobre todo cuando lo bañan. El
examen físico revela que el testículo derecho se encuentra un poco elevado, pero con
facilidad es llevado a la bolsa escrotal y permanece en ella. El manejo apropiado del
paciente es:
a. Realizar una ecografía diagnóstica.
b. Inducción hormonal para descenso testicular.
c. Referirlo a urología pediátrica.
d. Evaluar cada año la evolución del cuadro

Pregunta 397
Señale qué tasa de filtración glomerular indica falla renal en el paciente diabético tipo 2:
a. 89-90 ml/min.
b. 15-29 ml/min.
c. 14-15 ml/min.
d. 30-44 ml/min.

Pregunta 398
Camila tiene 2 años 2 meses, usted realiza el test de Denver y evidencia que dice su
nombre y apellidos, arma torre de 6 cubos, se puede poner y sacar la chompa, salta y se
separa de la madre con facilidad. Escoja el literal correcto relacionado a la consejería que
dará a los padres de Camila.
a. Sugiere a los padres que Camila sea evaluada por la psicóloga para un examen
completo, ya
que en el primer nivel no se aplica el test de Denver II.
b. Alerta a los padres porque Camila tiene un evidente retraso del desarrollo psicomotor
por lo
que la refiere al servicio de rehabilitación física.
c. Comunica a los padres que Camila puede tener un retraso del desarrollo pero por falta
de
estimulación por lo que la refiere al servicio de estimulación.
d. Elogia a los padres y les comunica que Camila tiene un desarrollo psicomotor
adecuado y
brinda información sobre más actividades de estimulación.

Pregunta 399
RN de 36.3 semanas por FUM, nació por cesárea sin labor, con membranas íntegras.
Líquido amniótico claro con grumos. Crecimiento adecuado para su edad gestacional. Al
nacimiento presenta taquipnea, quejido, cianosis, retracciones costales y subcostales. Se
le administra oxígeno y una hora después presenta mejoría. Seleccione el diagnóstico
correcto:
a. Síndrome de aspiración de meconio.
b. Neumonía neonatal.
c. Taquipnea transitoria del recién nacido.
d. Enfermedad de membrana hialina.
Pregunta 400
Sobre las parasitosis en pediatría, seleccione los enunciados correctos:
1. Entamoeba histolytica – Dolor abominal, diarrea, tenesmo – metronidazol.
2. Giardia lambia – dolor pélvico, meteorismo y evacuaciones liquidas claras – nitaxoznida.
3. Tenia soliun – obstrucción intestinal – citrato de piperazina.
4. Trichuris trichiura – prolapso rectal, anemia y retardo del crecimiento – Albendazol.
5. Enterobius vermicularis – prurito anal nocturno, bruxismo y vulvovaginitis – Albendazol.
6. Estrongyloides stercoralis – esteatorrea y tos - ivermectina.
a. 1, 2, 6.
b. 2, 3, 5.
c. 1, 4, 5.
d. 1, 2, 5.

También podría gustarte